93
Pharmacology 1. Duration of action of flumazenil is: A. 5 minute B. 10 minute C. 20 minute D. 30 minute Key: D 2. Which of the following compounds acts as a benzodiazepine antagonist? A. Flumazenil B. Naloxone C. Furazolidone D. Naltrexone Key: A 3. The effect of thiopentone on the CNS is quickly terminated because of: A. Rapid metabolism in the CNS B. Quick first-pass elimination C. Redistribution D. Rapid metabolism in systemic circulation Key: C 4. All of the following benzodiazepines can be used in elderly and those with liver disease EXCEPT: A. Lorazepam B. Oxazepam C. Triazolam D. Diazepam Key: D 5. Which of the following agents binds to GABA receptor chloride channel complex? A. Ethanol B. Alphaxolone C. Zolpidem D. Buspirone Key: C 6. In which of the following disorders, administration of barbiturates is contraindicated? A. Anxiety disorders B. Acute intermittent porphyria C. Kernicterus D. Refractory status epilepticus Key: B 7. Drugs that can be used safely in porphyria are: A. Phenobarbitone B. Ketamine C. Sodium valproate D. Midazolam Key: D 8. Flumazenil can reverse the respiratory depression caused by which of the following? A. Fentanyl B. Ketamine C. Midazolam D. Propofol Key: C 9. Which of the following hypnotic drugs facilitates the inhibitory actions of GABA but lacks anticonvulsant or muscle relaxing properties and has minimal effect on sleep architecture? A. Buspirone B. Diazepam C. Phenobarbital D. Zaleplon Key: D 10. A very potent and short acting benzodiazepine was given to a patient Kallu for the purpose of causing hypnosis but the drug caused psychiatric disturbances in him. Which of the following can be the hypnotic used? A. Flurazepam B. Nitrazepam C. Temazepam D. Triazolam Key: D 11. A 40 years old patient with liver dysfunction is scheduled for a surgical procedure. Lorazepam can be used for pre-anaesthetic medication in this patient without concern for excessive CNS depression because the drug is: A. Selective anxiolytic like buspirone B. Conjugated directly C. Reversible by administration of naloxone D. Forming several active metabolites Key: B 12. Which of the following drugs is contra-indicated in acute intermittent porphyria? A. Thiopentone B. Midazolam

7. Sodium valproate

  • Upload
    others

  • View
    17

  • Download
    0

Embed Size (px)

Citation preview

Page 1: 7. Sodium valproate

Pharmacology

1. Duration of action of flumazenil is:

A. 5 minute

B. 10 minute

C. 20 minute

D. 30 minute

Key: D

2. Which of the following compounds acts as a

benzodiazepine antagonist?

A. Flumazenil

B. Naloxone

C. Furazolidone

D. Naltrexone

Key: A

3. The effect of thiopentone on the CNS is quickly

terminated because of:

A. Rapid metabolism in the CNS

B. Quick first-pass elimination

C. Redistribution

D. Rapid metabolism in systemic circulation

Key: C

4. All of the following benzodiazepines can be used in

elderly and those with liver disease EXCEPT:

A. Lorazepam

B. Oxazepam

C. Triazolam

D. Diazepam

Key: D

5. Which of the following agents binds to GABA

receptor chloride channel complex?

A. Ethanol

B. Alphaxolone

C. Zolpidem

D. Buspirone

Key: C

6. In which of the following disorders, administration of

barbiturates is contraindicated?

A. Anxiety disorders

B. Acute intermittent porphyria

C. Kernicterus

D. Refractory status epilepticus

Key: B

7. Drugs that can be used safely in porphyria are:

A. Phenobarbitone

B. Ketamine

C. Sodium valproate

D. Midazolam

Key: D

8. Flumazenil can reverse the respiratory depression

caused by which of the following?

A. Fentanyl

B. Ketamine

C. Midazolam

D. Propofol

Key: C

9. Which of the following hypnotic drugs facilitates the

inhibitory actions of GABA but lacks anticonvulsant

or muscle relaxing properties and has minimal effect

on sleep architecture?

A. Buspirone

B. Diazepam

C. Phenobarbital

D. Zaleplon

Key: D

10. A very potent and short acting benzodiazepine was

given to a patient Kallu for the purpose of causing

hypnosis but the drug caused psychiatric disturbances

in him. Which of the following can be the hypnotic

used?

A. Flurazepam

B. Nitrazepam

C. Temazepam

D. Triazolam

Key: D

11. A 40 years old patient with liver dysfunction is

scheduled for a surgical procedure. Lorazepam can be

used for pre-anaesthetic medication in this patient

without concern for excessive CNS depression

because the drug is:

A. Selective anxiolytic like buspirone

B. Conjugated directly

C. Reversible by administration of naloxone

D. Forming several active metabolites

Key: B

12. Which of the following drugs is contra-indicated in

acute intermittent porphyria?

A. Thiopentone

B. Midazolam

Page 2: 7. Sodium valproate

C. Propofol

D. Etomidate

Key: A

13. Action of flumazenil on benzodiazepine receptor is:

A. Agonist

B. Partial agonist

C. Inverse agonist

D. Antagonist

Key: D

14. Which of the following drug is not metabolized by

liver:

A. Flunitrazepam

B. Diazepam

C. Oxazepam

D. Nitrazepam

Key: C

15. Shortest acting benzodiazepine is:

A. Midazolam

B. Alprazolam

C. Lorazepam

D. Diazepam

Key: A

16. Which of the following drug is used as Transcranial

patch for Parkinson’s disease?

A. Levodopa

B. Rotigotine

C. Selegiline

D. Carbidopa

Key: B

17. Which of the following if combined with

rivastagmine, decreases its efficacy?

A. Selective serotonin reuptake inhibitors

B. Reversible inhibitor of MAO-A

C. Tricyclic antidepressants

D. Atypical antidepressants

Key: C

18. Which of the following agents enhances the

bioavailability of levodopa in patients with

Parkinson’s disease:

A. Amantadine

B. Ropinirole

C. Entacapone

D. Selegiline

Key: C

19. All the following statements regarding levodopa are

correct EXCEPT:

A. In Parkinsonism, phenothiazines reduce its

efficacy

B. It is a prodrug

C. Pyridoxine reduces effect of levodopa in

Parkinsonism

D. Domperidone blocks levodopa induced emesis

and its therapeutic potential.

Key: D

20. A patient of Parkinsonism is managed with levodopa.

If Vitamin B complex is administered concurrently to

the patient:

A. The action of levodopa in brain will be

potentiated

B. Decarboxylation of l-dopa in brain will be

decreased

C. Side effects will be ameliorated

D. Decreased efficacy will result

Key: D

21. Ropinirole is most useful for the treatment of:

A. Parkinson’s disease

B. Wilson’s disease

C. Hoffmann syndrome

D. Carpal tunnel syndrome

Key: A

22. Drugs used for treatment of Parkinson’s disease

include:

A. Levodopa

B. Bromocriptine

C. Selegiline

D. All

Key: D

23. Drugs causing Parkinsonism include:

A. Bromocriptine

B. Phenothiazine

C. Haloperidol

D. Amantadine

E. Carbidopa

Key: B

24. Entacapone may be useful in patients being treated

with levodopa-carbidopa combination because it:

A. Activates COMT

B. Decreases formation of 3-OMD

C. Inhibits monoamine oxidase type B

Page 3: 7. Sodium valproate

D. Inhibits dopamine uptake

Key: B

25. Which of the following adverse effects of levodopa is

not minimized even after combining it with

carbidopa?

A. Involuntary movements

B. Nausea and vomiting

C. Cardiac arrhythmia

D. ‘On-off’ effect

Key: A

26. Which of the following antiparkinsonian drugs

directly activates dopaminergic D2 receptors in the

striatum?

A. Pramipexole

B. Entacapone

C. Benserazide

D. Selegiline

Key: A

27. Entacapone is an anti-Parkinsonism drug. It acts by:

A. Agonism at dopamine receptors

B. Antagonism at dopamine receptors

C. Monoamine oxidase inhibition

D. Cathecol-o-methyl transferase inhibition

Key: D

28. A patient of Parkinsonism, Mr. Khan noticed that the

therapeutic effect of levodopa decreased when he was

given another drug by his physician but no interaction

was seen when he switched over to levodopa-

carbidopa combination. The possible drug prescribed

by his physician can be:

A. Metoclopramide

B. Vitamin B complex

C. Chlorpromazine

D. Isoniazid

Key: B

29. Which of the following drug should not be given

along with levodopa:

A. Carbidopa

B. MAO inhibitors

C. Vitamin B complex

D. Benserazide

Key: C

30. Drug of choice in drug induced Parkinsonism is:

A. Levodopa

B. Benzhexol

C. Amantadine

D. Carbidopa

Key: B

31. In treatment of Parkinsonism, L-Dopa is combined

with carbidopa mainly:

A. To decrease the treatment duration

B. To decrease central side effects of L-Dopa

C. To decrease effectiveness of L-Dopa

D. To increase crossing of L-Dopa through BBB

Key: D

32. Mechanism of action of donepezil is:

A. Centrally acting reversible anticholinesterase

B. Centrally acting irreversible anticholinesterase

C. Irreversible cholinergic action

D. Reversible anticholinesterase

Key: A

33. Anti-Parkinsonism drug that is a selective COMT

inhibitor:

A. Entacapone

B. Ropinirole

C. Pergolide

D. Pramipexole

Key: A

34. A 72-year-old patient with Parkinsonism presents

with swollen feet. They are red, tender and very

painful. You could clear up these symptoms within a

few days if you tell the patient to stop taking:

A. Amantadine

B. Benztropine

C. Bromocriptine

D. Levodopa

Key: A

35. All of the following adverse effects are associated

with carbamazepine except:

A. Teratogenicity

B. Neurotoxicity

C. Decrease in antidiuretic hormone

D. Hypersensitivity

Key: C

36. Which is a treatment of juvenile myoclonic epilepsy

in pregnancy?

A. Levetiracetam

B. Carbamazepine

Page 4: 7. Sodium valproate

C. Vigabatrin

D. Phenytoin

Key: A

37. A 65 years old male presented to a hospital with focal

seizures. His renal function was normal. Which of the

following is the drug of choice for this patient?

A. Valporate

B. Pregbalin

C. Levetiracetam

D. Oxcarbazepine

Key: D

38. Which among the following is an early sign of

magnesium toxicity?

A. Depression of deep tendon reflexes

B. Respiratory depression

C. Cardiac arrest

D. Decreased urine output

Key: A

39. All of the following are used for myoclonic seizures

except:

A. Sodium valproate

B. Zonisamide

C. Carbamazepine

D. Topiramate

Key: C

40. Fetal hydantoin syndrome is seen if following drug is

used in pregnancy?

A. Phenytoin

B. Alcohol

C. Ethosuximide

D. Phenobarbitone

Key: A

41. Ethosuximide can be used for the treatment of:

A. Generalized tonic clonic seizures

B. Absence seizures

C. Complex seizures

D. Myoclonic seizures

Key: B

42. The drug of choice for prevention of seizures in a

patient with severe preeclampsia is:

A. Phenytoin

B. Magnesium sulphate

C. Diazepam

D. Nifedipine

Key: B

43. All of the following are adverse effects of sodium

valproate EXCEPT

A. Weight gain

B. Alopecia

C. Liver damage

D. Osteomalacia

Key: D

44. First drug to be used in absence seizures is:

A. Phenytoin

B. Benzodiazepines

C. Valproate

D. Carbamazepine

Key: C

45. Which antiepileptic drug does not act via inhibition of

sodium channels?

A. Vigabatrin

B. Carbamazepine

C. Lamotrigine

D. Phenytoin

Key: A

46. Granulocytopenia, gingival hyperplasia and facial

hirsutism are all possible side effects of one of the

following anticonvulsant drugs?

A. Phenytoin

B. Valproate

C. Carbamazepine

D. Phenobarbitone

Key: A

47. Drug of choice for myoclonic epilepsy in pregnancy

is:

A. Carbamazepine

B. Sodium valproate

C. Phenobarbitone

D. Phenytoin

Key: B

48. Prolonged use of one of the following anticonvulsant

drugs can produce weight loss:

A. Gabapentin

B. Oxcarbazepine

C. Topiramate

D. Valproic acid

Key: C

Page 5: 7. Sodium valproate

49. A patient with recent-onset primary generalized

epilepsy develops drug reaction and skin rash due to

phenytoin sodium. The most appropriate course of

action is:

A. Shift to clonazepam

B. Restart phenytoin sodium after 2 weeks

C. Shift to sodium valproate

D. Shift to ethosuximide

Key: C

50. On chronic treatment with a drug, a patient presents

with gingival hyperplasia and facial hirsutism. The

drug most likely to cause these side effects is:

A. Phenytoin

B. Carbamazepine

C. Valproic acid

D. Phenobarbitone

Key: A

51. The most common adverse effect particularly seen in

young children because of the use of sodium

valproate is:

A. Hepatitis

B. Loss of hair

C. Anorexia

D. Tremor

Key: A

52. A patient, Rama was diagnosed to be having febrile

convulsions in the paediatric emergency. Which of

the following can be used for the treatment of this

patient?

A. Intramuscular phenobarbitone

B. Intravenous phenytoin

C. Rectal diazepam

D. Oral sodium valproate

Key: C

53. Status epilepticus is managed best with the use of

which of the following drugs?

A. Intravenous diazepam

B. Intravenous phenytoin sodium

C. Intramuscular phenobarbitone

D. Rectal diazepam

Key: A

54. Antiepileptic drug implicated in causing toxic

epidermal necrolysis/Steven Johnson syndrome is:

A. Felbamate

B. Gabapentin

C. Lamotrigine

D. Vigabatrin

Key: C

55. A young male, Farman suffers from a seizure disorder

which is characterized by tonic rigidity of limbs

followed in 20-30 sec by tremors progressing to

massive jerking of the body. This clonic phase lasts

for 1-3 min. The anti-seizure drug of choice for this

patient is:

A. Clonazepam

B. Ethosuximide

C. Fosphenytoin

D. Valproic acid

Key: D

56. An antiepileptic drug ‘A’ can also be used for the

treatment of post-herpetic neuralgia and pain due to

diabetic neuropathy. Which of the following can be

the agent ‘A’?

A. Carbamazepine

B. Gabapentin

C. Lamotrigine

D. Primidone

Key: B

57. Which of the following will you like to give to a

pregnant patient to decrease the risk of neural tube

defects in the offspring, if your patient is receiving

antiepileptic drugs?

A. Folic acid

B. Vitamin A

C. Vitamin E

D. Pyridoxine

Key: A

58. Carbamazepine in elderly causes:

A. Hypernatremia

B. Hyponatremia

C. Hyperkalemia

D. Hypokalemia

Key: B

59. Pseudolymphoma can result from long-term use of:

A. Phenytoin

B. Carbamazepine

C. Sodium valproate

D. Phenobarbital

Key: A

Page 6: 7. Sodium valproate

60. Management of typical febrile seizures include all the

following except:

A. Tepid sponging

B. Paracetamol and ibuprofen

C. Intermittent diazepam

D. Prophylactic phenobarbitone.

Key: D

61. The drug of choice in treatment of infantile spasms is:

A. Phenobarbitone

B. Carbamazepine

C. Phenytoin

D. None

Key: A

62. Which of the following is not a side effect of

phenytoin?

A. Hypoglycemia

B. Osteomalacia

C. Gum hypertrophy

D. Lymphadenopathy

Key: A

63. Drug of choice for Trigeminal neuralgia is:

A. Carbamazepine

B. Phenobabitone

C. Phenytoin

D. Valproic acid

Key: A

64. Myoclonus in children is best treated by:

A. Clonazepam

B. Sodium Valproate

C. Phenobarbitone

D. Ethosuccimide

Key: B

65. DOC for myoclonic seizure is:

A. Phenobarbitone

B. Ethosuximide

C. Lamotrigine

D. Valproic acid

Key: D

66. All are the side effects of prolonged phenytoin

therapy EXCEPT:

A. Osteomalacia

B. Gynaecomastia

C. Megaloblastic anemia

D. Gum hyperplasia

Key: B

67. Antiepileptic drug that can cause folate deficiency

anemia is:

A. Valproate

B. Phenytoin

C. Phenobarbitone

D. Carbamazepine

Key: B

68. Which one of the following drugs is used to treat

status epilepticus?

A. Primidone

B. Carbamazepine

C. Diazepam

D. Sodium valporate

Key: C

69. The antiepileptic drug which does not produce

enzyme induction is:

A. Phenobarbitone

B. Sodium valproate

C. Phenytoin sodium

D. Carbamazepine

Key: B

70. Osteomalacia is adverse effect of:

A. Primidone

B. Phenytoin

C. Carbamazepine

D. Valproic acid

Key: B

71. Gum hypertrophy is an adverse effect of the following

drug when used at therapeutic levels:

A. Phenobarbitone

B. Phenytoin

C. Carbamazepine

D. Sodium evaporate

Key: B

72. Drug of choice in complex partial seizure is:

A. Phenytoin

B. Valproate

C. Carbamazepine

D. Phenobarbitone

Key: C

73. Which drug should be avoided in pregnancy?

Page 7: 7. Sodium valproate

A. Phenytoin

B. Insulin

C. Heparin

D. Methyle dopa

Key: A

74. Neural tube defect is an adverse effect of:

A. Valproate

B. Phenytoin

C. Diazoxide

D. None

Key: A

75. Lamotrigine has common side effects of:

A. Rash

B. Irritability

C. Nephrotoxicity

D. Behavioral disturbances

Key: A

76. Carbamazepine has drug interaction with all of the

following except:

A. Erythromycin

B. Phenytoin

C. Doxycycline

D. Barbiturates

Key: C

77. The drug of choice to control convulsions in

eclampsia is:

A. Pethidine

B. Diazepam

C. Magnesium sulphate

D. Phenytoin

Key: C

78. Best agent for premenstrual syndrome management

is?

A. Progesterone

B. Anxiolytic

C. SSRI

D. Vitamin E

Key: C

79. Antidepressant drug that can be used in nocturnal

enuresis is?

A. Imipramine

B. Fluvoxamine

C. Phenelzine

D. Bupropion

Key: A

80. A schizophrenic patient started on haloperidol 2 days

back, comes with complaints of torticollis and

orofaciolingual movements. What is the diagnosis?

A. Acute dystonia

B. Tardive dyskinesia

C. Parkinsonism

D. Akathisia

Key: A

81. A woman treated with lithium during pregnancy, the

fetus should be tested for:

A. Neural tube defects

B. Cardiac malformations

C. Urogenital abnormalities

D. Scalp defects

Key: B

82. What is the drug of choice for Obsessive

CompulsiveDisorder?

A. Imipramine

B. Fluoxetine

C. Benzodiazepines

D. Alprazolam

Key: B

83. Which of the following has highest potential to cause

metabolic syndrome?

A. Clozapine

B. Risperidone

C. Quetiapine

D. Aripiprazole

Key: A

84. Which of the following is not a side effect of

paroxetine (SSRIs)?

A. Premature ejaculation

B. Erectile dysfunction

C. Decreased libido

D. Diarrhea

Key: A

85. Which of the following is not a mood stabilizer?

A. Lithium

B. Valproate

C. Carbamazepine

D. Fluoxetine

Key: D

Page 8: 7. Sodium valproate

86. Buspirone is used as a/an:

A. Anxiolytic

B. Sedative

C. Muscle relaxant

D. Anticonvulsant

Key: A

87. Risperidone is most commonly used to treat which of

the following disorders?

A. Dementia

B. Depression

C. Schizophrenia

D. Obsessive-compulsive disorder

Key: C

88. Which of the following is the most common side

effect seen with fluoxetine therapy?

A. Seizure

B. Anxiety

C. Hypotension

D. Loose stools

Key: B

89. Antipsychotic drug induced Parkinsonism is treated

by:

A. Anticholinergics

B. Levodopa

C. Selegiline

D. Amantadine

Key: A

90. Prolactin secretion is inhibited by:

A. Dopamine antagonist

B. GABA

C. Neurophysin

D. Bromocriptine

Key: D

91. Whichof the following drugs has a high affinity for 5-

HT2 receptors in the brain, does not cause

extrapyramidal dysfunction or hematotoxicity, and is

reported to increase the risk of significant QT

prolongation?

A. Chlorpromazine

B. Clozapine

C. Olanzapine

D. Ziprasidone

Key: D

92. A 45-year-old male, Sanjeev was brought to the

emergency with severe agitation and aggressive

behavior. He was started on haloperidol and the

patient became responsive and cooperative. After 8

days of treatment, he developed high grade fever,

diarrhea, confusion and muscle rigidity. Which of the

following should be used for the treatment of this

condition?

A. Diazepam

B. Benzhexol

C. Dantrolene

D. High dose of haloperidol

Key: C

93. Drug of choice in intractable hiccups is:

A. Metoclopramide

B. Fluoxetine

C. Selegiline

D. Chlorpromazine

Key: D

94. Dryness of mouth caused by antipsychotic drug is

caused by blockade of:

A. Muscarinic ACh receptors

B. GABA receptors

C. Serotonergic receptors

D. Dopaminergic receptors

Key: A

95. Depression is not a side effect of:

A. Propanolol

B. Oral contraceptives

C. Reserpine

D. Flupenthixol

Key: D

96. Which of the following drug treatment increases thirst

and causes dilute diuresis?

A. Phenobarbitone

B. Lithium

C. Chlorpromazine

D. Clozapine

Key: B

97. Extrapyramidal symptoms are a complication of

treatment with following drugs:

A. Antipsychotics

B. Anti anxiety drugs

C. Anti depressants

D. Anti malarial drugs

Page 9: 7. Sodium valproate

Key: A

98. Drug useful in malignant hyperthermia is:

A. Halothane

B. Succinyl choline

C. Dantrolene

D. Haloperidol

Key: C

99. Risperidone acts on which receptor:

A. D2

B. 5 HT2

C. Both

D. NA

Key: C

100. Neuroleptic malignant syndrome is caused by:

A. Carbamazepine

B. Clonazepam

C. Haloperidol

D. Fluoxetine

Key: C

101. Drug used to treat extrapyramidal syndrome due to

phenothiazines:

A. Diphenhydramine

B. Benzhexol

C. Clonidine

D. Promethazine

Key: B

102. Coarse tremors, dysarthria and ataxia are side effects

of

A. Lithium

B. Haloperidol

C. Imipramine

D. None

Key: A

103. A young man is with known heroin addiction is

brought in the emergency in unconscious state. On

examination, the patient has decreased bowel sounds,

depressed respiration and pin point pupil. The

treatment of choice for this patient is

A. Oral natrexone

B. IV naloxone

C. Oral diazepam

D. Oral Buprenorphine

Key: B

104. Which of the following is a phase II drug

metabolizing reaction?

A. Acetylation

B. Deamination

C. Hydrolysis

D. Oxidation

Key: A

105. Which of the following drugs may inhibit the hepatic

microsomal P450 responsible for warfarin metabolism

A. Cimetidine

B. Ethanol

C. Phenobarbital

D. Rifampin

Key: A

106. Tricyclic antidepressant side effect

A. Generalized tonic clonic seizures

B. Urinary retention

C. Hypotension

D. Diarrhea

Key: A

107. Which of the following terms best describes an

antagonist that interacts directly with the agonist and

not at all or only incidentally, with the receptor?

A. Pharmacological antagonist

B. Partial agonist

C. Physiological antagonist

D. Chemical antagonist

Key: D

108. Which of the following terms best describes a drug

that blocks the action of epinephrine at its receptors

by occupying those receptors without activating

them?

A. Pharmacological antagonist

B. Partial agonist

C. Chemical antagonist

D. Noncompetitive antagonist

Key: A

109. Phenylephrine causes

A. Constriction of vessels in the nasal mucosa

B. Increased gastric secretion and motility

C. Increased skin temperature

D. Miosis

Key: A

Page 10: 7. Sodium valproate

110. Which among the following is the best inotrope drug

for use in right heart failure?

A. Dobutamine

B. Digoxin

C. Dopamine

D. Milrinone

Key: D

111. All of the following are seen in digitalis toxicity

except:

A. Ventricular bigeminy

B. Paroxysmal atrial tachycardia with fast

ventricular rate

C. Regularization of atrial fibrillation

D. Bidirectional ventricular tachycardia

Key: B

112. Digoxin toxicity is increased by all except:

A. Renal impairment

B. Hyperkalemia

C. Hypercalcemia

D. Hypomagnesemia

Key: B

Precipitating factors for digoxen toxicity

» classically: hypokalaemia*

» increasing age, renal failure, myocardial

ischaemia, hypomagnesaemia, hypercalcaemia,

hypernatraemia, acidosis, hypoalbuminaemia,

hypothermia, hypothyroidism

» Drugs: amiodarone, quinidine, verapamil,

diltiazem, spironolactone (competes for secretion

in distal convoluted tubule therefore reduce

excretion), ciclosporin. Also drugs which cause

hypokalaemia e.g. thiazides and loop diuretics

113. Digoxin is contraindicated in

A. Supraventricular tachycardia

B. Atrial fibrillation

C. Congestive heart failure

D. Hypertrophic obstructive cardiomyopathy

Key: D

114. Digoxin toxicity increases with

A. Digoxin with quinidine

B. Digoxin with hyperkalemia

C. Digoxin with hypercalcemia

D. None

Key: B

115. Digoxin toxicity is caused by

A. Hypokalemia

B. Hyperkalemia

C. Hyponatremia

D. Hyperuricemia

Key: A, Digoxin toxicity is caused by Hypokalemia

and Digoxin toxicity causes hyperkalemia

116. Digoxin is not indicated in:

A. Atrial flutter

B. Atrial fibrillation

C. High output failure

D. PSVT

Key: C

117. Digoxin toxicity causes?

A. Hypokalemia

B. Hyperkalemia

C. Hypercalcemia

D. All

Key: A, thiazides and loop diuretics causes

hypokalemia so lead to digoxen toxicity

118. Adverse effect of Digoxin is

A. Hyperkalemia

B. Hypokalemia

C. Hypercalcemia

D. Hypomagnesemia

Key: A

119. Digoxin action is not affected in:

A. Hepatic disease

B. Electrolyte disturbances

C. Renal failure

D. MI

Key: A

120. Digoxin toxicity is caused by

A. Hydrochlorothiazide

B. Furosemide

C. Acetazolamide

D. All

Key: A

121. All of the following increase chance of digitalis

toxicity except:

A. hypokalemia

B. hypoglycemia

C. hyperkalemia

D. hypothyroidism

Page 11: 7. Sodium valproate

Key: A

122. BNP is degraded by:

A. Neutral endopeptidase

B. Elastase

C. Omapatrilat

D. ACE

Key: A

123. Digoxin toxicity is aggravated by:

A. Hypokalemia

B. Hyperkalemia

C. Hypocalcemia

D. Hypermagnesemia

Key: A, Hypokalemia and Hypercalcemia aggravated

digoxin toxicity

124. Which of the following conditions increases the risk

of digoxin toxicity?

A. Administration of quinidine

B. Hyperkalemia

C. Hypermagnesemia

D. Hypocalcemia

Key: A

125. Drugs that reduce myocardial remodelling in CHF

include all of the following except:

A. Carvedilol

B. Digoxin

C. Enalapril

D. Spironolactone

Key: B

126. Which of the following has been shown to prolong

life in patients with chronic congestive heart failure

but has a negative ionotropic effect on cardiac

contractility?

A. Carvedilol

B. Digoxin

C. Enalapril

D. Furosemide

Key: A

127. Which of the following is the drug of choice in

treating suicidal overdose of digitoxin?

A. Digibind antibodies

B. Lignocaine

C. Magnesium

D. Potassium

Key: A

128. Angiotensin converting enzyme inhibitors are useful

in congestive heart failure as:

A. First choice drugs unless contraindicated

B. An alternative to diuretics

C. A substitute for digitalis

D. Adjuncts only in resistant cases

Key: A

129. The most important channel of elimination of digoxin

is:

A. Glomerular filtration

B. Tubular secretion

C. Hepatic metabolism

D. Excretion in bile

Key: A

130. Which of the following drugs can prolong survival in

patients with CHF?

A. Furosemide

B. Inamrinone

C. Losartan

D. Digoxin

Key: C

131. The diuretic of choice for rapid relief of congestive

symptoms in a patient of CHF is:

A. Hydrochlorothiazide

B. Furosemide

C. Metolazone

D. Amiloride

Key: B

132. Mechanism of action of digitalis in atrial fibrillation

is:

A. By decreasing cardiac contractility

B. Na+ K+ ATPase inhibition

C. Increase in refractoriness of AV nodal tissue

D. By causing bradycardia

Key: C

133. Sodium-nitroprusside acts by activation of:

A. Guanylate cyclase

B. K+ channels

C. Ca++ channels

D. Cyclic AMP

Key: A

134. Best treatment of severe digitalis toxicity is:

A. Potassium supplements

Page 12: 7. Sodium valproate

B. Diphenyl hydantoin

C. Quinidine

D. Fab fragments of digitalis antibodies

Key: D

135. Which of the following drug reduces blood pressure

primarily by directly decreasing heart rate alone:

A. Propanolol

B. Prazosin

C. Alpha methyl dopa

D. Nitroprusside sodium

Key: A

136. The antihypertensive which causes decreased libido

and impotence is

A. Atenolol

B. Enalapril

C. Prazosin

D. Diltiazem

Key: A

137. Alpha methyldopa is primarily used for:

A. Pregnancy induced hypertension

B. Renovascular hypertension

C. First line agent in hypertension

D. Refractory hypertension

Key: A

138. A 50 year old male presents with malignant

hypertension. The drug of choice is:

A. Sodium nitroprusside

B. Sublingual nifedipine

C. Furosemide

D. Enalapril

Key: A

139. Which drug should not be given in pregnancy?

A. Labetalol

B. ACE inhibitors

C. Hydralazine

D. Methyl dopa

Key: B

140. A 60-year old hypertensive patient on angiotensin II

receptor antagonist (losartan) is posed for hernia

repair surgery. The antihypertensive drug should be:

A. Continued till the day of operation

B. Discontinued 24 hrs preoperatively

C. Discontinued one week preoperatively

D. Administered in an increased dosage on the day

of operation

Key: A

141. An elderly hypertensive has diabetes mellitus and

bilateral renal artery stenosis. The best management

is:

A. Enalapril

B. Verapamil

C. Beta blockers

D. Thiazides

Key: B

142. Postural hypotension is the common side effect of

which of the following?

A. ACE inhibitors

B. Alpha receptor blockers

C. Arteriolar dilators

D. Selective β1 blockers

Key: B

143. An anti-hypertensive drug that causes positive

Coomb’s test is:

A. Methyldopa

B. Clonidine

C. Hydralazine

D. Sodium-nitropruside

Key: A

144. The drug of choice in digitalis induced ventricular

arrhythmias is:

A. I.V. Lignocaine

B. Phenytoin

C. Quinidine

D. Procainamide

Key: A

145. Drug of choice in pregnancy induced hypertension is

A. Amlodipine

B. Losartan

C. Diuretic

D. Methyldopa

Key: D

146. Centrally acting sympatholytic agent used as

antihypertensive agent is:

A. Propanolol

B. Clonidine

C. Prazosin

Page 13: 7. Sodium valproate

D. Phenoxybenzamine

Key: B

147. Treatment of choice in hypertension with diabetes

mellitus is:

A. Beta-blockers

B. Thiazides

C. ACE inhibitors

D. Calcium channel blockers

Key: C

148. Which of the following antihypertensives causes

sedation:

A. Clonidine

B. Hydralazine

C. Losartan

D. Amlodipine

Key: A

149. Nitrates are used for all of the following conditions

except?

A. Congestive heart failure

B. Cyanide poisoning

C. Esophageal spasm

D. Renal colic

Key: D

150. Nitroglycerine causes all except:

A. Hypotension and bradycardia

B. Methemoglobinemia

C. Hypotension and tachycardia

D. Vasodilation

Key: A

151. Coronary steal phenomenon is seen with:

A. Dipyridamole

B. Diltiazem

C. Propanolol

D. Verapamil

Key: A

152. You decide not to prescribe sildenafil in a patient

because the patient told you that he is taking an

antianginal drug. Which of the following can it be?

A. Calcium channel blockers

B. β adrenergic blockers

C. Organic nitrates

D. Angiotensin converting enzyme inhibitors

Key: C

153. Verapamil is associated with all of the following

except?

A. Constipation

B. Bradycardia

C. Hyperglycemia

D. Increased PR interval

Key: C

154. Which of the following drugs has been used in the

treatment of angina by inhalation and has a very rapid

onset and brief duration of action?

A. Amyl nitrite

B. Isosorbide mononitrate

C. Nitroglycerine

D. Propanolol

Key: A

155. The drug effective for treatment as well as

prophylaxis of angina pectoris is:

A. Isosorbide dinitrate

B. Pentaerythritol tetranitrate

C. Diltiazem

D. Dipyridamole

Key: A

156. Which of the following drugs is most likely to

precipitate angina?

A. Amlodipine

B. Nifedipine

C. Diltiazem

D. Verapamil

Key: B

157. Propanolol is contra-indicated in a patient of angina

pectoris who is already receiving:

A. Nifedipine

B. Aspirin

C. Verapamil

D. Isosorbide mononitrate

Key: C

158. Calcium channel blockers with predominant

peripheral effects is:

A. Verapamil

B. Diltiazem

C. Amlodipine

D. None of these

Key: C

Page 14: 7. Sodium valproate

159. Longest acting nitroglycerine preparation is:

A. Glyceryl trinitrate

B. Amyl nitrite

C. Penta erythrital tetranitrate

D. Isosorbide dinitrate

Key: C

160. A drug lacking vasodilatory properties that is

effective in angina is:

A. Isosorbide dinitrate

B. Metoprolol

C. Nifedipine

D. Verapamil

Key: B

161. A patient of acute myocardial infarction being treated

in intensive care unit developed left ventricular failure

with raised central venous pressure. It was decided to

use nitroglycerine. Which route of administration

would be most suitable?

A. Sublingual

B. Oral

C. Intravenous bolus injection

D. Slow intravenous infusion

Key: D

162. Glyceryl trinitrate is given by sublingual route

because of:

A. Short t1/2 in plasma

B. High hepatic first pass metabolism

C. High bioavailability by oral route

D. Extensive protein binding

Key: B

163. Calcium channel blockers are useful in all, EXCEPT:

A. Angina

B. Supraventricular arrhythmia

C. Sick sinus syndrome

D. Hypertension

Key: C

164. Drug not to be given in ischemic heart disease is:

A. Atenolol

B. ACE inhibitor

C. Isoproterenol

D. Streptokinase

Key: C

165. Potassium channel opener with anti-anginal activity

is:

A. Nicorandil

B. Dipyridamole

C. Trimetazidine

D. Oxyphedrine

Key: A

166. All of the following drugs act by blocking calcium

channels except:

A. Dantrolene

B. Nicardipine

C. Diltiazem

D. Verapamil

Key: A

167. Amyl nitrite is used by which route?

A. Oral

B. Inhalation

C. IV

D. IM

Key: B

168. Propanolol should not be given to a patient on

treatment with which of the following drug?

A. Nifedipine

B. Nitrates

C. ACE inhibitors

D. Verapamil

Key: D

169. Verapamil is contraindicated in

A. Hypertesion

B. Complete heart block

C. Paroxysmal supraventricular tachycardia

D. Angina pectoris

Key: B

170. Enalapril increases the levels of which of the

following?

A. Bradykynin

B. Interferon

C. PAF

D. TNF

Key: A

171. The major clinical use of nimodipine is in:

A. Hypertension

B. Angina pectoris

C. Subarachnoid haemorrhage

D. Raynaud’s phenomenon

Page 15: 7. Sodium valproate

Key: C

172. When nitrates are combined with calcium channel

blockers:

A. Arterial pressure will decrease

B. Heart rate will increase

C. Ejection time will decrease

D. End-diastolic volume will increase

Key: A

173. Calcium channel blocker with maximum effect on

conduction in heart is:

A. Phenylamine

B. Nifedipine

C. Diltiazem

D. Verapamil

Key: D

174. In a patient of congenital prolonged QT syndrome

and intermittent Torsades de pointes, which of the

following should be prescribed:

A. Magnesium sulphate

B. Metoprolol

C. Cardiac pacing

D. Isoprenaline

Key: A

175. The anti-arrhythmic drug of choice in most of the

cases of acute paroxysmal supraventricular

tachycardia is:

A. Adenosine

B. Amiodarone

C. Propanolol

D. Quinidine

Key: A

176. Characteristic adverse effect of quinidine includes:

A. Lupus erythematosis

B. Cinchonism

C. Increase in digoxin clearance

D. Precipitation of hyperthyroidism

Key: B

177. A drug effect that is produced by therapeutic doses of

both timolol and amiodarone is blockade of:

A. Cardiac Na+ channels

B. Cardiac K+ channels

C. Beta-1 adrenoceptors

D. Alpha-adrenoceptors

Key: C

178. Aslam malik developed acute CHF and was put on

digitalis therapy. ECG of this patient revealed the

presence of ventricular extrasystoles. Which of the

following drugs can be administered safely to this

patient in order to counteract this arrhythmia?

A. Lignocaine

B. Quinidine

C. Atropine

D. Amiodarone

Key: A

179. The drug of choice for rapid correction of PSVT in

known asthmatic is:

A. Adenosine

B. Esmolol

C. Neostigmine

D. Verapamil

Key: D

180. Which of the following anti-arrythmic drug decreases

the action potential duration in purkinje fibers?

A. Quinidine

B. Flecainide

C. Amiodarone

D. Lignocaine

Key: D

181. Drug of choice for ventricular arrhythmias due to

myocardial infarction (MI) is:

A. Quinidine

B. Amiodarone

C. Xylocaine

D. Diphenylhydantoin

Key: C, DOC for ventricular arrhythmias after MI is

lignocaine (lidocaine; xylocaine). DOC for

supraventricular arrhythmias after MI is beta blockers

if cardiac function is adequate.

182. Drug of choice for ventricular premature beats (VPC)

due to digitalis toxicity is

A. Diphenylhydantoin

B. Quinidine

C. Amiodarone

D. Verapamil

Key: A

183. Arrhythmias refractory to the treatment of lignocaine

can be treated by:

A. Sotalol

B. Diltiazem

Page 16: 7. Sodium valproate

C. Amiodarone

D. Quinidine

Key: C

184. Quinidine exerts its action on heart by:

A. Ca2+ channel blockade

B. Na+ channel blockade

C. K+ channel opening

D. Cl– channel opening

Key: B

185. Mechanism of action of statins is:

A. Inhibition of HMG-CoA synthase

B. Stimulation of HMG-CoA reductase

C. Indirect increase of LDL receptors synthesis

D. Inhibition of intestinal cholesterol absorption

Key: C

186. In a patient with hypertriglyceridemia and low HDL,

which of the following drug will be best without risk

of myopathy as side effect?

A. Fibric acid derivatives

B. Nicotinic acid

C. Atrovastatin

D. Clofibrate

Key: B

187. HDL is specifically increased by:

A. Lovastatin

B. Niacin

C. Gemfibrozel

D. Probucol

Key: B

188. Which of the following drugs is best for reducing

proteinuria in a diabetic patient?

A. Metoprolol

B. Enalapril

C. Chlorthiazide

D. Clonidine

Key: B

189. Vagal stimulation of heart causes?

A. Increased heart rate

B. Increased RR interval in ECG

C. Increased cardiac output

D. Increased force of contraction

Key: B

190. ACE inhibitors (Captopril, Enalapril) causes:

A. Persistent cough & Taste changes

B. First dose hypotension

C. Angioedema

D. All

Key: D

191. Drugs causing constipation are:

A. Verapamil & Ferrous sulphate

B. MAO-Inhibitors

C. Tricyclic antidepressants

D. All

Key: D

192. Drug is absorbed fast if given in

A. Lipid form

B. Aqueous form

C. Charged form

D. Oxidized form

Key: B

193. Beta blockers are not used in

A. Migraine

B. Peripheral vascular disease

C. Angina pectoris

D. Hypertension

Key: B

194. Dipyridamol decrease platelet aggregation by:

A. Inhibiting thromboxane A2

B. Inhibiting degradation of adenosine

C. Inhibiting adenosine uptake

D. Potentiation of PGI1

Key: C

» Dipyridamole inhibits phosphodiesterase enzyme and

blocks uptake of adenosine. This increase platelet

CAMP and potentiates prostaglandin I2. This

interferes with platelet aggregation.

195. GABA agonist is:

Page 17: 7. Sodium valproate

A. Carbamazepine

B. Phenytoin

C. Sodium valporate

D. Lamotrigine

Key: C

196. Which drug causes SLE like syndrome?

A. Albutolol

B. Proctalol

C. Haloperidol

D. Phenybutazone

Key: B

197. SLE is caused by:

A. Rifampicin

B. Procainamide

C. Digitalis

D. Acetazolamide

Key: B

198. The analgesics that shouldnot be used in acute gout is:

A. Aspirin

B. Indomethacin

C. Naproxen

D. Diclofenac

Key: A

199. Mechanism of action of rifampicin

A. Inhibits RNA synthesis

B. Inhibit DNA synthesis

C. Inhibit protein synthesis

D. Activate DNA- dependent RNA polymerase

Key: A, Rifampicin binds strongly to the beta subunit

of bacterial DNA-dependent RNA polymerase and

thereby inhibits RNA synthesis

200. Which of the following is true about muscarinic

receptors?

A. M1- bronchus

B. M2-heart

C. M3-nerves

D. M1 endothelium

Key: B

201. Captopril causes cough by

A. Inhibit bradykinin metabolism

B. Histamine release

C. Local irritation in the throat

D. Stimulation of cough centre

Key: A, The action of ACE inhibitors to inhibit

bradykinin metabolism contributes significantly to

their hypotensive action and is apparently responsible

for some adverse side effects, including cough and

angioedema.

202. Which of the antimicrobial drug doesnot act on cell

wall :

A. Griseofulvin

B. Cycloserine

C. Penicillin

D. Bacitracin

Key: A

203. The drug that causes obstructive jaundice is:

A. Chlorpromazine

B. Carbon tetrachloride

C. Tetracycline

D. Methyldopa

Key: A

204. The drug that is not effective in UTI caused by

pseudomonas is:

A. Nalidixic acid

B. gentamycin

C. Ceftazidime

D. Norfloxacin

Key: A

205. Osteoporosis is a complication of the long term use

of:

A. Glucocorticoids

B. Estrogen

C. Progesterone

D. Warfarin

Key: A, Drugs causing osteoporosis

Glucocorticoids and Heparin

206. Tachyphylaxis means

A. Responsiveness gradually decreases as a

consequence of continued drug administration

B. Responsiveness gradually increases as a

consequence of continued drug administration

C. Responsiveness diminishes rapidly after

administration of a drug

D. Responsiveness increases rapidly after

administration of a drug

Key: C, with some drugs, the intensity of

response to a given dose may change during the

course of therapy; in these cases, responsiveness

usually decreases as a consequence of continued

Page 18: 7. Sodium valproate

drug administration, producing a state of relative

tolerance to the drug’s effects. When

responsiveness diminishes rapidly after

administration of drug, the response is said to be

subject to tachyphylaxis.

207. The anesthetic agent that raises the blood pressure and

pulse rate is

A. Ketamine

B. Halothane

C. Ether

D. Enflurane

Key: A

208. Grey baby syndrome is the adverse effect of:

A. Aspirin

B. Tetracycline

C. Chloramphenicol

D. Erythromycin

Key: C

209. In PSVT, the drug of choice is:

A. Adenosine

B. Propanolol

C. Lignocaine

D. Epinephrine

Key: A

» Management of PSVT:

Perform carotid sinus massage

IV adenosine (6 to 12 mg) or verapamil (2.5 to

10mg)

210. Which of the following is not a catecholamine:

A. Epinephrine

B. Norepinephrine

C. Dopamine

D. Phenylephrine

Key: D

211. AV block is not caused by:

A. Hydralazine

B. Digoxin

C. Clonidine

D. Verapamil

Key: A

212. Digoxin toxicity is increased by

A. Hyperkalemia

B. Hypokalemia

C. Hypercalcemia

D. Hypocalcemia

Key: B

213. What is not true about digoxin?

A. Doesnot cross placenta

B. Hyperkalemia may be caused by acute digitalis

overdose or severe poisoning

C. Vomiting is common in patients with digitalis

overdose

D. Atropine is effective for digoxin induced

bradycardia or AV block

Key: A

214. Cell wall synthesis inhibitor is

A. Vancomycin

B. Amphotericin B

C. Octreotide

D. Tetracycline

Key: A, Vancomycin inhibits cell wall synthesis by

binding firmly to the D-Ala-D-Ala terminus of

nascent peptidoglycan penta-peptide, preventing

further elongation of peptidoglycan and cross-linking.

The peptidoglycan is thus weakened and the cell

becomes susceptible to lysis. The cell membrane is

also damaged, which contributes to the antibacterial

effect.

215. Which of the following has least glucocorticoid

activity:

A. Fludrocortisone

B. Dexamethasone

C. Triamcinolone

D. Betamethasone

Key: C

216. Flumazenil is a/an:

A. Benzodiazepine antagonist

B. Benzodiazepine agonist

C. Adrenergic blocking agent

D. Opiate antagonist

Key: A

217. Antidote of diazepam is

A. Flumazenil

B. Naltrexone

C. Naloxane

D. N acetylcysteine

218. Potassium loss is least with

A. Frusemide

B. Acetazolamide

Page 19: 7. Sodium valproate

C. Spironolactone

D. Thiazide

Key: C

219. Which of the following drug does not act on

Pseudomonas:

A. Ofloxacine

B. Azithromycin

C. Imipenam

D. Ceftazidine

Key: B

220. NSAID induced peptic ulcer is treated with:

A. Misoprostol

B. Omeprazol

C. Famotidine

D. Sucralfate

Key: A

221. Which of the following drug cannot be given in renal

failure:

A. Doxycycline

B. Vancomycin

C. Clindamycin

D. Chloramphenicol

Key: B

222. Which of the following is not a feature of ethambutol

toxicity:

A. Hypercalcemia

B. Retrobulber neuritis

C. Color vision defect

D. Hyperuricemia

Key: A

223. All are the long acting bronchodilators except:

A. Salbutamol

B. Adrenaline

C. Salmeterol

D. Terbutaline

Key: B

224. Which of the following is not used in petit mal

epilepsy?

A. Benzodiazepam

B. Phenytoin

C. Sodium valporate

D. Ethosuximide

Key: B

225. Pancreatitis is a side effect of:

A. Asparaginase

B. Glucocorticoids

C. Valporic acid

D. All of the above

Key: D

226. In coronary artery disease the role of beta blocker in

decreasing the symptoms is by:

A. Decreasing the heart rate

B. Increasing the coronary blood flow

C. Decreasing cardiac force

D. Preventing exercise induced tachycardia

Key: D

227. Cyclosporin acts by inhibiting the production of:

A. Interleukin-1

B. Interleukin -2

C. Interleukin 6

D. Macrophages

Key: B

228. Which of the following anti-malarial drugs can also

be used in nocturnal leg cramps?

A. Quinine

B. Chloroquine

C. Proguanil

D. Artemether

Key: A

229. Severe acne is treated with:

A. Topical retinoic acid

B. Topical steroid

C. Oral tretinoin

D. Systemic steroid

Key: C

230. Which of the following is not a second line ATT

drug:

A. Kanamycin

B. Cycloserine

C. 5 flucytosine

D. Ciprofloxacin

Key: C

231. Which of the following drug acts on motilin receptor:

A. Erythromycin

B. Tetracycline

C. Norfloxacin

Page 20: 7. Sodium valproate

D. Chloramphenicol

Key: A

232. Milk –alkali syndrome is caused by:

A. Calcium carbonate

B. Magnesium sulfate

C. Aluminum trisilicate

D. Aluminum hydroxide

Key: A

233. Most common cause of interstitial nephritis is:

A. Cloxacillin

B. Methicillin

C. Carbenicilline

D. Penicillin

Key: B, Use of Methicillin leads to interstitial

nephritis in about 33% of cases if treatment continues

for more than 10 days.

234. Serum level should be looked for drugs with:

A. Hit and run drugs

B. Wide therapeutic index

C. More side effects

D. Narrow therapeutic index

Key: D

235. Albendazole is used in all except:

A. Ascariasis

B. Trichuris trichiura

C. Cysticercosis

D. Schistosomiasis

Key: D

236. Single-strand breaks in DNA during DNA replication

is mediated by:

A. DNA ase I

B. Ligase

C. Topoisomerase I

D. Restricition endonuclease

Key: C, Topoisomerase I is an enzyme which produce

reversible single-strand breaks in DNA during DNA

replication.

237. Clindamycin inhibits :

A. Protein synthesis

B. DNA gyrase

C. Cell wall

D. Lysosomal enzyme

Key: A, Clindamycin inhibits protein synthesis in

susceptible bacteria by binding to 50 S subunits of

bacterial ribosomes and preventing peptide bond

formation.

238. Which of the following drugs acts on u receptors in

central nervous system:

A. Pentazocine

B. Pethidine

C. Buprenorphine

D. Morphine

Key: D

239. Advantages of parenteral route:

A. Hepatic first pass effect is minimized

B. Maximize concentration at the site of action

C. Convenience for use

D. Side effects are minimised

Key: A

240. Which type of pain does morphine remove

A. Visceral

B. Somatic

C. Both

D. None

Key: C

241. Reflex bradycardia can occur with

A. Reserpine

B. Methyldopa

C. Verapamil

D. Nimodipine

Key: A

242. Glucagon is given in the poisoning of

A. Propanolol

B. Atropine

C. Carbimazole

D. OP

Key:

A

243. Long term effect of steroid are all except

A. Hypertension

B. Hypotension

C. Cushingoid state

D. Diabetes mellitus

Key: B

244. Tapering of steroid is done in case of long term

therapy

Page 21: 7. Sodium valproate

A. To prevent adrenal hyper secretion

B. To prevent hypertension

C. To prevent activation of HPA axis

D. To prevent adrenal suppression

Key: D

245. Which of the following is not an immunosuppressant

being used in chemotherapy:

A. Cyclophosphamide

B. Cyclosporine

C. Prednisolone

D. Methotrexate

Key: B, Among following immunosuppressant

cyclosporine is not used in cancer chemotherapy.

246. Amlodipine is

A. K+ channel blocker

B. Na+ channel blocker

C. Ca+ channel blocker

D. None

Key: C

247. Mechanism of action of Bisphosphate in osteoporosis:

A. Increased release of Ca+ from bone

B. Increase blood Ca+ level

C. Increase Ca+ deposition on bones

D. Inhibit the osteoclastic resorption of bone

Key: D,

248. Active form of folic acid is:

A. Folvin

B. Folate

C. Folinic acid

D. Folvate

Key: C

249. Clarithromycin is a

A. Aminoglycoside

B. Beta lactam antibiotic

C. Cephalosporin

D. Macrolide

Key: D

250. Drug of choice for acute attack of Migraine:

A. Ergotamine

B. Sumatriptan

C. Fluphenaline

D. Propanolol

Key: B

251. The drug of choice in AML is

A. Cytarabine and daunorubicin

B. Methotrexate

C. Cyclosporine and cyclophosphamide

D. Azathioprine and levamisol

Key: A

252. The antagonist of lorazepam is:

A. Flumazenil

B. Domperidone

C. Bromocriptine

D. Propanolol

Key: B

253. Therapeutic index of a drug is an indicator of:

A. Potency

B. Safety

C. Toxicity

D. Efficacy

Key: B

254. The drug of choice for schiostosmiasis is

A. Levimasole

B. Praziquental

C. Piperazine

D. Albendazole

Key: B

255. Which of the following causes hypoglycemia:

A. Acute alcohol intoxication

B. Diazoxide

C. Thiazide

D. Domperidone

Key: A

256. Which of the following is false regarding ipratropium

bromide:

A. Increased intraocular pressure

B. Used by inhalation

C. Scratching in trachea

D. Dryness of mouth

Key: A

257. Which of the drug is excreted in urine?

A. Digoxin

B. Streptomycin

C. Cefotaxime

D. Ciprofloxacin

Page 22: 7. Sodium valproate

Key: A

258. Which of the following drug doesnot cause

peripheral neuropathy:

A. Ethambutol

B. INH

C. Rifampicin

D. Streptomycin

Key: C

259. Which of the following is not a side effects of

haloperidol:

A. Hypomania

B. Akathesia

C. Dystonia

D. Dyskinesia

Key: A

260. Which of the following is not a side effects of

clofazimine:

A. Anemia

B. Skin pigmentation

C. GI disturbance

D. Icthyosis

Key: A

261. Most cardiotoxic :

A. Procaine

B. Bupivacaine

C. Ropivacaine

D. Lidocaine

Key: B

262. Oral contraceptives failure is seen with the concurrent

use of:

A. Ethambutol

B. Propanolol

C. Cimetidine

D. Rifampicin

Key: D

263. Dimethyl polysiloxane acts as :

A. Defoaming agent

B. Local antacids

C. Systemic antacids

D. Ulcer healing agents

Key: A, Dimethyl polysiloxane is a silicon polymer

which reduces the surface tension and collapses the

froth (antifoaming agent). It is not absorbed from

intestine and is pharmacologically inert.

264. Which of the following is not used in

organophosphate poisoning:

A. Atropine

B. Pralidoxime

C. Physostigmine

D. Hospitalization

Key: C

265. In organophosphate poisoning all are used except:

A. Charcoal

B. Neostigmine

C. Atropine

D. PAM

Key: B

266. Which of the following is a ‘specific’ opiod

antagonist:

A. Naltrexone

B. Methadone

C. Nalorphine

D. Naloxone

Key: D, Naloxone is a competitive antagonist of all

types of opioid receptors and has no agonist activity

even at high doses. Usual dose is 0.4 to 0.8 mg IV

267. The action of d- tubocurarine is potentiated by:

A. Streptomycin

B. Cephalexine

C. Tetracycline

D. Doxycycline

Key: A, action of competitive blockers( d-

tubocurarine) is potentiated by aminoglycoside

antibiotics particularly streptomycin.

268. The drug of choice in myasthenia gravis is :

A. Atropine

B. Neostigmine

C. Physostigmine

D. Rocuronium

Key: B

269. Discoloration of urine is not seen with the use of:

A. Quinidine

B. Methylene blue

C. Rifampicin

D. Phensuximide

Key: A

Page 23: 7. Sodium valproate

270. Which ATT drug is completely avoided in pregnancy:

A. Streptomycin

B. Pyrazinamide

C. INH

D. Rifampicin

Key: A

271. Which of the following anti-tubercular drug is not

hepatotoxic:

A. Streptomycin

B. Rifampicin

C. Ethionamide

D. Pyrazinamide

Key: A

272. Antiplatelet drug

A. Warfarin

B. Streptokinase

C. Low dose aspirin

D. Tranexamic acid

Key: C

273. Drug of choice for pulmonary edema:

A. Frusemide

B. Steroid

C. LMW heparin

D. Nitrates

Key: A

274. Ototoxicity is caused by

A. Loop diuretics

B. Thiazides

C. Triamterine

D. Acetazolamide

Key: A

275. True about HIV in pregnancy:

A. Deformity in baby are more common

B. Postpartum infection is common

C. Nevirapine significantly reduces transmission

D. Cesarean section has no role.

Key: C

276. Cardiomimetic action of catecholamines through

A. Stimulation of beta receptors

B. Acetylcholine

C. Atimulation of alpha receptors

D. Activation of sodium channels

Key: A

277. Multi drug resistance in TB means resistance to at

least

A. All primary drugs

B. Rifampin+ pyrazinamide+ethambutol

C. Rifampin+ pyrazinamide+isoniazid

D. Rifampin+ isoniazid

Key: D

278. Propanolol is not used in:

A. Hypertension

B. Migraine

C. Varient angina

D. Thyrotoxicosis

279. Propranolol is contraindicated in

A. Hyperthyroidism

B. Asthma

C. Cardiac neurosis

D. GI bleeding

Key: B, Propranolol is a nonselective beta blocker .

Beta 2 blocking may exacerbate shortness of breath

by causing bronchoconstriction.

280. Proton pump inhibitors are most effective when:

A. Given after prolonged fasting

B. Along with an H2 blocker

C. Given with meals

D. Given before meals

Key: D

281. A 25 years old male experienced severe flushing, fall

in blood pressure after consumption of alcohol. This

attack can be precipitated by simultaneous intake of

all of the following drugs along with alcohol except:

A. Cefamandole

B. Metronidazole

C. Dexamethasone

D. Chlorpropamide

Key: C

282. Acetylcysteine is used in the treatment of:

A. Paracetamol toxicity

B. Chloroform toxicity

C. Carbon tetrachloride toxicity

D. All of the above

Key: A

283. Which of the following drugs causes gingival

hyperplasia, hirsutism and osteomalacia as side

effects?

Page 24: 7. Sodium valproate

A. Carbamazepine

B. Valproate

C. Phenytoin

D. Furosemide

Key: C

284. All are cell wall synthesis inhibitors except:

A. Vancomycin

B. Cefotaxim

C. Tetracycline(protein synthesis inhibitor)

D. Imipenem

Key: C

285. All of the following are used in AIDS except

A. Nevirapine

B. Zidovudine

C. Saquinavir

D. Gancyclovir ( it is antiviral for CMV not for

HIV)

Key: D

286. Drug of choice for MRSA(Methicillin-resistant

Staphylococcus aureus ):

A. Neomycin

B. Vancomycin

C. Streptomycin

D. Clindamycin

Key: B

287. Drug used for radical cure for malaria is:

A. Chloroquine

B. Primaquine

C. Sulphamethoxazole

D. Pyrimethamine

Key: B, Primaquine- radical cure and terminal

prophylaxis of infections with P vivax and P ovale

288. Alfa glucosidase inhibitor is

A. Metformin

B. Sulphonylurea

C. Acarbose

D. Rosiglitazone

Key: C

289. Renal toxicity is not seen in

A. Chlortetracycline

B. Doxycycline

C. Cephadroxil

D. Gentamicin

Key: B, Tetracycline given alone with diuretics may

produce nitrogen retention. Tetracyclines other than

doxycycline may accumulate to toxic level in patients

with impaired kidney function.

290. The potency of the steroid in the decreasing order is:

A. dexamethasone- prednisolone – hydrocortisone –

cortisone

B. dexamethasone- hydrocortisone- prednisolone –

cortisone

C. dexamethasone- prednisolone- cortisone-

hydrocortisone

D. Prednisolone-dexamethasone- hydrocortisone –

cortisone

Key: C

291. Bradycardia is the sideeffect of all the following

except:

A. Verapamil

B. Nifedipine

C. Digoxin

D. Propanolol

Key: B, Nifedipine is vascular selective calcium

channel blocker vasodilation and reflexly

tachycardia

292. Mechanism of action of aspirin is:

A. COX inhibition

B. Anticholinergic action

C. DPA agonist action

D. Selective COX-2 inhibition.

Key: A

293. Mechanism of action of trimethoprim is:

A. Dihydrofolate reductase inhibition

B. Protein synthesis inhibition

C. Cell wall damage

D. G-6 PD inhibition

Key: A

294. Which of the following drugs is not a dihydrofolate

reductase inhibitor?

A. Pyrimithamine

B. Sulfamethoxazole

C. Trimethoprim

D. Methotrexate

Key: B

295. Which of the following is not given in falciparum

malaria?

A. Pyrimethamine

Page 25: 7. Sodium valproate

B. Fansidar

C. Quinine

D. Primaquin

296. Which of the following is a somatostatin analogue?

A. Vasopressin

B. Leuprolide

C. Prolactin

D. Octreotide

Key: D

297. Drug of choice for T. solium is:

A. Mebendazole

B. Niclosamide

C. Pyrantel pamoate

D. Levimasole

Key: B

298. Which of the following is not an anti-cancer drug?

A. Cyclophosphamide

B. Carmustine

C. Mechlorethamine

D. Ondansetron

Key: D

299. Which of the following is a nephrotoxic drug?

A. Aminoglycosides

B. Amphotericin

C. Tetracycline

D. All

Key: D

300. All the following are used in absence seizure except

A. Ethosuximide

B. Phenytoin

C. Sodium valporate

D. Clonazepam

Key: B, Ethosuximide and Sodium valporate are drug

of choice for absence seizure. Clonazepam is a long-

acting drug with documented efficacy against absence

seizure.

301. The drugs causing ototoxicity among the following

drugs are :

A. Nalidixic acid

B. Rifampicin

C. Streptomycin (aminoglycoside)

D. Dapsone

Key: C

302. The drug not excreted through kidney among the

following is:

A. Digoxin

B. Ofloxacin

C. Streptomycin

D. Digitoxin

Key: D, Digitoxin is excreted through liver and

digoxin through kidney.

303. Which class of antiarrythmic drugs does verapamil

belong to?

A. Class I

B. Class II

C. Class III

D. Class IV

Key: D

304. All are the sideeffects of pyrazinamide except:

A. Drug fever

B. Hepatic failure

C. Arthalgia

D. Optic neuritis

Key: D

305. The drug of choice in cholera is

A. Doxycycline

B. Tetracycline

C. Amoxycillin

D. Azithromycin

Key: B

306. The usual dose of tinidazole in giardiasis is

A. 500 mg TDS for 5 days

B. 300 mg BD for 3 days

C. 2gm single dose

D. 500mg TDS for 7 days

Key: C

307. Which of the following is not used as an

antiparkinsonian drug?

A. Phenothiazine

B. Entacapone

C. Levodopa

D. Chlorpropmide

Key: D

308. Which is not used for the treatment of kalaazar?

A. Amphotericin

B. Pentamidine

C. Sodium stibogluconate

Page 26: 7. Sodium valproate

D. Quinidine

Key: D

309. During metabolism, alcohol is reduced to

A. Acetaldehyde

B. Acetone

C. Formaldehyde

D. Ether

Key: A

310. Which of the following is the protease inhibitor

antiviral drug:

A. Zidovudin

B. Lamivudin

C. Indinavir

D. Nevirapine

Key: C

311. The mechanism of action of aminoglycosides is:

A. Inhibition of 50s ribosome

B. Inhibition of 30s ribosome

C. Inhibition of cellwall synthesis

D. a and b

Key: B

312. The first dose effect is seen with

A. Enalapril

B. Nifedipine

C. Verapamil

D. Propanolol

Key: A, Severe hypotension can occur after initial

doses of any ACE inhibitor in patients who are

hypovolemic due to diuretics, salt restriction, or

gastrointestinal fluid loss.

313. Which of the following drugs used in peptic ulcer

causes black coating of tongue?

A. Sucralfate

B. Bismuth

C. Ranitidine

D. Omeprazole

Key: B, Bismuth can cause black tongue and black

stools in some users of the drug, when it combines

with trace amount of sulfur in saliva and the GI tract.

314. Which of the following drug is the most appropriate

for the treatment of a patient with diabetes and

hypertension?

A. Beta-blocker

B. Calcium channel blocker

C. ACE inhibitor

D. None

Key: C, ACE inhibitors, antihypertensives, are

important in controlling nephropathy in diabetes.

315. Ciprofloxacin acts by inhibiting

A. DNA gyrase

B. Cell wall synthesis

C. 50s ribosome

D. Mycolic acid synthesis

Key: A, Fluoroquinolones are DNA gyrase inhibitors.

316. Which of the following benzodiazepines is a short

acting one?

A. Oxazepam

B. Lorazepam

C. Diazepam

D. Nitrazepam

Key: A

317. The shortest half-life among benzodiazepines is of

A. Midazolam

B. Diazepam

C. Clonazepam

D. Lorazepam

Key: A, Shortest acting benzodiazepines –TOM

(triazolam, oxazepam, midazolam)

318. Which of the following is not given during

pregnancy?

A. Warfarin

B. Insulin

C. Heparin

D. Nalidixic acid

Key: A, Warfarin is teratogenic and heparin is safe

319. Most potent steroid among the following is:

A. Hydrocortisone

B. Budesonide

C. Dexamethasone

D. Beclomethasone

Key: C

320. The long acting anti-protozoal is:

A. Secnidazole

B. Metronidazole

C. Tinidazole

D. Omeprazole

Key: A

Page 27: 7. Sodium valproate

321. Pralidoxime is used in the treatment of poisoning of

A. Organophosphate

B. Morphine

C. TCA

D. Paracetamol

Key: A

322. OCP failure occurs in all except

A. Rifampin

B. Barbiturates

C. Ketoconazole

D. Chloroquine

Key: D, Chloroquine is not an enzyme inducer

323. Which drug is the hepatic enzyme inducer?

A. Propanolol

B. Phenylephrine

C. Phenytoin

D. Pefloxacin

Key: C

324. Corticosteroid doesnot cause

A. Myopoathy

B. Avascular necrosis

C. Cataract

D. Peptic ulcer

Key: D

325. Which drug induces its own metabolism?

A. Phenytoin

B. IV diazepam

C. Carbamazepine

D. Lorazepam

Key: C

326. Coronary steal syndrome is caused by

A. Enflurane

B. Isoflurane

C. Sevoflurane

D. Halothane

Key: B, Isoflurane relaxes previously constricted

isolated porcine coronary artery segments, causes

coronary steal. Coronary steal (with its symptoms

termed coronary steal syndrome or cardiac steal

syndrome) is a phenomenon where an alteration of

circulation patterns lead to a reduction in the blood

directed to the coronary circulation.

327. Mechanism of action for local anesthetics

A. Sodium channel inactivation

B. GABA mediated CL-channel activation

C. GABA mimetic action

D. Prolongation of sodium channel activation

Key: A, Local anesthetics reversibly block impulse

conduction along nerve axons and other excitable

membranes that utilize sodium channels as the

primary means of action potential generation. This

action can be used clinically to block pain sensation

from- or sympathetic vasoconstrictor impulses to-

specific areas of the body.

328. Which drug is contraindicated in a child with

shortness of breath

A. Ipratopium

B. Nidocromil

C. Propanolol

D. Beclomethasone

Key: C, Propanolol is a nonselective beta blocker.

Beta 2 blocking may exacerbate shortness of breath

by causing bronchoconstriction.

329. Gum hypertrophy is the adverse effect of

A. Phenobarbitone

B. Phenytoin

C. Reserpine

D. Methyldopa

Key: B, Phenytoin toxicity: Nystagmus, Diplopia,

Ataxia, Gingival hyperplasia, Hirsutism

330. Which is NRT I(Nucleoside/Nucleotide Reverse

Transcriptase Inhibitors ):

A. Efavirenz

B. Delaverdine

C. Enfuvertide

D. Didanosine

Key: D

331. Morphine for pain is not to be used in:

A. MI

B. Postoperative pain

C. Terminal cancer pain

D. Biliary colic

Key: D, Morphine can aggravate conditions like

biliary colic, diverculitis and pancreatitis and is

therefore contraindicated in these conditions.

332. Which of the following drug is not used for managing

status epilepticus:

A. Phenytoin

B. Diazepam

Page 28: 7. Sodium valproate

C. Thiopentone sodium

D. Carbamazepine

Key: D, Carbamazepine is effective in the

management of generalized tonic clonic and partial

seizure types. It is not effective in status epilepticus.

333. Which of the following drug is contraindicated to

treat hypertension with pregnancy?

A. Enalapril

B. Methyldopa

C. Nifedipine

D. Labetalol

Key: A, In humans, ACE inhibitors (enalapril) can

cause fetal and neonatal morbidity and mortality when

administered to pregnant women. ACE inhibitors

should be discontinued as soon as possible when

pregnancy is detected. Captopril and enalapril are

associated with transient anuric renal failure and

oligohydraminos.

334. Which of the following is a fourth generation

cephalosporin?

A. Ceftriaxone

B. Cefaclor

C. Cefepime

D. Cefuroxime

Key: C, Cefepime and cefpirome are the fourth

generation cephalosporin.

335. Dobutamine increases:

A. Heart rate

B. Cardiac output

C. Blood pressure

D. Plasma volume

Key: B, Dobutamine directly stimulates beta-1

receptors of the heart to increase myocardial

contractility and stroke volume, resulting in increased

cardiac output.

336. Ingestion of which of the following drug by mother

may cause the infant to have cleft lip, spinal bifida

and ASD:

A. Isotretinoin

B. Valporate

C. Phenytoin

D. Carbamazepine

Key: B, Fetal valporate syndrome

337. Ciprofloxacin / Fluoroquinolones are contraindicated

in children because?

A. They cause arthropathy

B. They cause mental retardation

C. They cause agranulocytosis

D. All of the above

Key: A

338. Most common cardiac defect caused due to lithium is:

A. Ebstein’s anomaly

B. HOCM

C. Aortic aneurysm

D. Eisenmenger syndrome

Key: A, Use of lithium during pregnancy increases

incidence of neonatal goiter and congenital

cardiovascular malformations, especially Ebsteins

anomaly

339. Half-life of IV plain insulin is ….. Minutes:

A. 5 to 9 minutes

B. 15 to 30 minutes

C. 30 to 45 minutes

D. 60 minutes

Key: A

Drug of choice in cold urticaria is:

A. Cyproheptadine

B. Terfinadine

C. Fexofinadine

D. Astemizole

Key: A, Cyproheptadine is an antihistaminic used in

cold urticaria and as an appetite stimulant

340. Chloroquine is most effective against which stage of

malarial parasite:

A. Merozoites

B. Tissue schizonts

C. Blood schizonts

D. Gametocytes

Key: C, Chloroquine inhibits the erythrocytic stage of

Plasmodium. It interrupts erythrocytic schizogony

(blood schizonts).

341. Ipratropium bromide is useful in bronchial asthma

because it is

A. Anticholinergic

B. Vasodilator

C. Antiallergic

D. Mast cell stabilizer

Page 29: 7. Sodium valproate

Key: A, It is an anticholinergic drug used as a

bronchodilator inhalation spray for management of

bronchial asthma. Dryness of mouth and irritation of

trachea are common side effects.

342. For legionella infection, the drug of choice is:

A. Erythromycin

B. Cefuroxime

C. Tetracycline

D. Sulfamethoxazole

Key: A

343. Which one of the following drugs increases

gastrointestinal motility?

A. Glycopyrrolate

B. Atropine

C. Neostigmine

D. Fentanyl

Key: C

344. Antipsychotic drug induced parkinsonism is treated

by

A. Anticholinergic

B. Levodopa

C. Selegiline

D. Amantadine

345. The drug not avoided in G6PD?

A. chloramphenicol

B. quinidine

C. nitrofurantoin

D. streptomycin

Key: D

346. The aspirin is used in MI because?

A. decreases thromboxane synthesis

B. has analgesic effect

C. reduces prostaglandin synthesis

D. reduces cardiac work

Key: a

347. OCP produces which of the following in the liver?

A. cholangiosarcoma

B. adenoma

C. cirrhosis

D. atropic changes

Key: b

348. Which of the following drug is contraindicated in

pregnancy?

A. clonidine

B. Enalapril

C. methyldopa

D. hydralazine

Key: B

349. The side effect of phenytoin are?

A. Gum hypertrophy

B. alopecia

C. subungual exostosis

D. onycholysis

Key: a

350. Drug used for treatment of Methicillin Resistant

Staphylococcus Aureus

A. Teicoplanin

B. Vancomycin

C. Both

D. None

Key: c

351. Mechanism of action of Aspirin is inhibition of

A. Thromboxane A2 Synthesis

B. Phosphodiesterase

C. HMG- CoA Reductase

D. Pancreatic Lipase

Key: A

352. MAO of local anesthetics

A. Blocking Na and Preventing depolarisation

B. Inhibiting Aldose reductase

C. Increasing degradation of Adrenaline

D. None of the above

Key: A

353. Ceftriaxone is

A. II Generation short acting

B. III generation long acting

C. active against beta lactamase

D. IV generation used orally

Key: B

354. Which of the following is a Prodrug

A. Enalapril

B. Dopamine

C. Ampicillin

D. Prednisolone

Key: A

355. Drug of choice for Herpes

Page 30: 7. Sodium valproate

A. Acyclovir

B. Zidovidine

C. Indinavir

D. Ribavirin

Key:

356. Treatment of bacterial vaginosis. Drug of choice is

A. Ampicillin

B. Metronidazole

C. Ciprofloxacin

D. Fluconazole

Key:

357. Drug with high degree of Photosensitivity is

A. Tetracycline

B. Doxycycline

C. Minocycline

D. Methacycline

Key: B

358. The Drug to be avoided in the first trimester of

pregnancy?

A. Warfarin

B. INH

C. Quinine

D. Methyldopa

Key: A

359. Inverse agonist of benzodiazepine receptor is

A. Phenobarbitone

B. Flumazenil

C. Beta-carboline

D. Gabapentin

Key: C, Beta-carboline: An inverse agonist is an agent

that binds to the same receptor as an agonist but

induces a pharmacological response opposite to that

agonist.

360. All of the following are hallucinogens except

A. LSD

B. Phencyclidine

C. Mescaline

D. Methylphenidate

Key: D, Methylphenidate is a psychostimulant drug

used for treatment of attention-deficit hyperactivity

disorder.

361. One of the following is not pencillinase susceptible

A. Amoxicillin

B. Pencillin G

C. Piperacillin

D. Cloxacillin

Key: D, Cloxacillin is a penicillinase resistant

penicillin.

362. Which one of the following is used in therapy of

Toxoplasmosis?

A. Artensunate

B. Thiacetazone

C. Ciprofloxacin

D. Pyrimethamine

Key: D

363. Which one of the following drugs is "Topoisomerase

I" inhibitor?

A. Doxorubicin

B. Irinotecan

C. Etoposide

D. Vincristine

Key: B

364. All of the following drugs cause definite hemolysis in

a glucose-6-phosphate dehydrogenase deficient

patient except?

A. primaquine

B. nitrofurantoin

C. naphthalene

D. ciprofloxacin

Key: D

365. Digoxin is not indicated in?

A. atrial flutter

B. atrial fibrillation

C. high output failure

D. PSVT

Key: C

366. SLE like syndrome is most commonly associated with

the administration of the following drug?

A. rifampicin

B. procainamide

C. digitalis

D. phenytoin

Key: B

367. Volume of distribution of drugs is altered in all

except?

A. athletes

B. pregnancy

C. older age

Page 31: 7. Sodium valproate

D. neonate

Key: B

368. Digoxin is eliminated from the body by?

A. tubular filtration

B. hepatic conjugation

C. glomerular filtration

D. oxidation

Key: C

369. Chills, fever, and muscle aches are common reactions

to which one of the following antiviral drugs?

A. Acyclovir.

B. Ganciclovir.

C. Oseltamivir.

D. Interferon.

Key: D

370. A 57-year-old man is being treated for an atrial

arrhythmia. He complains of headache, dizziness, and

tinnitus. Which one of the follo wing antiarrhythmic

drugs is the most likely cause?

A. Amiodarone.

B. Procainamide.

C. Propranolol.

D. Quinidine.

Key: D, The clustered symptoms of headache,

dizziness, and tinnitus are characteristic of

cinchonism

371. Drug of choice for cisplatin induced emesis is

A. Metoclopramide

B. Domperidone

C. Ondansetron

D. Octreotide

Key: C

372. Drug used for the Rx of malignant hyperthermia due

to inhaled anesthesia interaction with Neuruscular

blockers

A. Baclofen

B. Diazepam

C. Dantrolene

D. Tubocurarine

Key: C

373. Long acting insulin is ?

A. Insulin Detemir

B. Insulin Lispro

C. Insulin aspart

D. Insulin glulisine

Key: A

374. Which of the following steroids can be administered

for maturity of surfactant in preterm labor.

A. Beclomethasone

B. Betamethasone

C. Prednisolone

D. Hydrocortisone

Key: A

375. Digoxin toxicity

A. Increase K

B. Increase Mg

C. Decrease Ca

D. Alkalosis

Key: D

376. Which drug DO NOT cause gynaecomastia

A. Digoxin

B. Griseofulvin

C. Cimetidine

D. Androgen

Key: D

377. Acetazolamide is:

A. Competitive and reversible carbonic anhydrase

inhibitor

B. Non-competitive and reversible carbonic

anhydrase inhibitor

C. Competitive and irreversible carbonic anhydrase

inhibitor

D. Non-competitive and irreversible carbonic

anhydrase inhibitor

Key: B

378. Thiazides can cause:

A. Hyperkalemic paralysis

B. Hypouricemia

C. Hypolipidemia

D. Impotence

Key: D

379. Free water clearance is decreased by?

A. Vincristine

B. Vinblastine

C. Chlorpropamide

D. Furosemide

Key: D

Page 32: 7. Sodium valproate

380. Which diuretic could be considered appropriate for

combining with ACE inhibitors?

A. Spironolactone

B. Eplerenone

C. Hydrochlorothiazide

D. Amiloride

Key: C

381. Spironolactone is contraindicated with which of the

following drugs?

A. Enalapril

B. Atenolol

C. Verapamil

D. Chlorthiazide

Key: A

382. Regarding furosemide true statement is:

A. Acute pulmonary edema is an indication

B. Acts on PCT

C. Mild diuresis

D. Given only by parenteral route

Key: A

383. Intravenous furosemide is used for rapid control of

symptoms in acute left ventricular failure. It provides

quick relief of dyspnoea by:

A. Producing bronchodilation

B. Causing rapid diuresis and reducing circulating

blood volume

C. Causing venodilation

D. Stimulating left ventricular contractility

Key: C

384. Most potent loop diuretic is:

A. Furosemide

B. Bumetanide

C. Torsemide

D. Ethacrynic acid

Key: B

385. Which of the following diuretics can result in

metabolic acidosis?

A. Indapamide

B. Furosemide

C. Hydrochlorthiazide

D. Acetazolamide

Key: D

386. Long-term use of which diuretic agent can result in

gynaecomastia?

A. Amiloride

B. Spironolactone

C. Triamterene

D. Acetazolamide

Key: B

387. A 50-year-old man has a history of frequent episodes

of renal colic with high calcium renal stones. The

most useful diuretic in the treatment of recurrent

calcium stones is:

A. Furosemide

B. Spironolactone

C. Hydrochlorthiazide

D. Acetazolamide

Key: C

388. A 46-year-old male, Jorawar Singh presented to the

emergency with muscle weakness and cramping. He

has been taking hydrochlorothiazide for recently

diagnosed hypertension. Which of the following is the

most likely cause of his symptoms?

A. Hypocalcemia

B. Hyponatremia

C. Hypokalemia

D. Hypoglycemia

Key: C

389. Furosemide should not be administered with NSAIDs

because latter:

A. Prevent platelet aggregation

B. Inhibit prostacyclin synthesis

C. Decrease sodium reabsorption

D. Increase the secretion of furosemide in urine

Key: B

390. All of the following diuretics inhibit Na+ – K+ – 2Cl

symporter, EXCEPT:

A. Furosemide

B. Thiazide

C. Ethacrynic acid

D. Mersalyl

Key: B

391. Which of the following diuretics cause Hypercalcemia

and can be used in recurrent renal calcium stones?

A. Spironolactone

B. Furosemide

Page 33: 7. Sodium valproate

C. Chlorthiazide

D. Mannitol

Key: C

392. Which one is a mineralocorticoid antagonist?

A. Thiazide

B. Cyproterone acetate

C. Furosemide

D. Spironolactone

Key: D

393. In cirrhotic ascites, which diuretic is preferred?

A. Furosemide

B. Acetazolamide

C. Spironolactone

D. Any of the above

Key: C

394. Aldosterone action is on:

A. Proximal tubule

B. Distal tubules

C. Loop of Henle

D. Collecting duct

Key: D

395. Thiazide diuretic does not cause:

A. Hyper calcaemia

B. Hypo magnesemia

C. Hyperkalemia

D. Hyperuricemia

Key: C

396. Potassium sparing diuretics acts on:

A. Na+ K+ pump

B. Aldosterone receptor

C. Carbonic anhydrase

D. Na+ Cl– symporter

Key: B

397. Drug causing deafness is:

A. Thiazide

B. Spiranolactone

C. Ethacrynic acid

D. Triamterene

Key: C

398. Drug that can be used for producing alkalinization of

urine is?

A. Hydrochlorthiazide

B. Furesemide

C. Acetazolamide

D. Spironolactone

Key: C

399. In a patient on cisplatin therapy, which of the

following diuretics would be preferred?

A. Mannitol

B. Acetazolamide

C. Thiazide

D. Furosemide

Key: A

400. Spirolactone is contraindicated with enalapril because

it causes

A. Hyperkalemia

B. Hypercalcemia

C. Hypernatremia

D. Hypokalemia

Key: A

401. Loop diuretics acts on:

A. PCT

B. DCT

C. Thick ascending limb of loop of Henle

D. Collecting duct

Key: C

402. Which of the following is the drug of choice for the

treatment of Syndrome of Inappropriate Antidiuretic

Hormone secretion?

A. Demeclocycline

B. Vasopressin

C. Thiazide diuretics

D. Chlorpropamide

Key: A

403. Selective V2 receptor agonist useful for the treatment

of central diabetes insipidus is:

A. Arginine vasopressin

B. Desmopressin

C. Lypressin

D. Terlipressin

Key: B

404. Drug of choice for central diabetes insipidus is:

A. Desmopressin

B. Leuprolide

C. Thiazide diuretics

D. Insulin

Page 34: 7. Sodium valproate

Key: A

405. A 33-year-old male, Ali presents to OPD your office

with fatigue, muscle weakness and headache. His

blood pressure is 170/120 mmHg and his heart rate is

100/min. Laboratory evaluation reveals hypokalemia,

metabolic alkalosis and decreased plasma renin

activity. On CT scan, a mass was noted on left

suprarenal gland. Patient was prescribed a drug for

few weeks and the symptoms subsided. Laboratory

values and blood pressure returned to normal values.

The likely drug given to this patient is?

A. Clonidine

B. Propanolol

C. Hydrochlorothiazide

D. Spironolactone

Key: D

406. A 37-year-old woman presents to the emergency

room complaining of general tingling sensation all

over her body and spasm of her right hand for 1 hour.

Her spasm is worsening and she feels as her body

becomes more rigid. She had undergone subtotal

thyroidectomy 2 weeks ago and the incision is healed

and clean. Blood tests reveal very low serum calcium

level. What should be the immediate therapy to

reverse her symptoms?

A. Vitamin D

B. Calcitonin

C. Calcium gluconate

D. Levothyroxine

Key: C

407. A 62-year-old man presents to the clinic with rashes

all over his body and painful joints. Last month, he

was diagnosed with arrhythmia. He is taking one

medication for it. Laboratory tests reveal anti-histone

antibodies. Which of the following drugs is most

likely responsible for this condition?

A. Hydralazine

B. Metoprolol

C. Propranolol

D. Verapamil

Key: A

408. A 4-year old girl is brought to the clinic by her mother

complaining of fever and abdominal pain since

yesterday. Today, she looks uncomfortable during

micturition. On examination, her temperature is 99.4,

her pulse is 100/min. and her suprapubic area is soft

and tender. Her blood analysis shows increased

neutrophils. Her urinalysis is positive for organisms

and leukocytes. What is the drug of choice for this

condition?

A. Ciprofloxacin

B. Chloramphenicol

C. Co-trimoxazole

D. Rifampin

Key: C

409. A 56-year-old man with a history of type 2 diabetes

presents to his physician for a follow up. He

complains of recurrent hypoglycemic reactions to his

medications. Which of these medications is most

likely responsible for this patient's hypoglycemic

reactions?

A. Glyburide

B. Metformin

C. Acarbose

D. Rosiglitazone

Key: A

410. A 75-year-old man presents to the office for

evaluation of new symptoms. He has a history of

cancer and takes chemotherapeutic treatment. Now he

complains of dyspnea and mild fatigue since last

week. Examination reveals temperature of 98.3 F,

pulse of 80/min, blood pressure of 140/86 and

respiratory rate of 22/min. On chest auscultation,

there are crackles on both lung bases with no

wheezes. A chest x-ray shows bilateral diffuse

interstitial markings. Which of the following agents is

responsible for this patient’s new features?

A. Bleomycin

B. Cisplatin

C. Cyclophosphamide

D. Etoposide

Key: A

411. A 24-year-old man presents to the clinic with a

complaint of grey-colored penile discharge and

burning micturition for 2 days. His medical, surgical

and family histories are unremarkable. Examination

reveals a purulent discharge that can be seen by

squeezing his penis. Penile swab shows Gram-

negative diplococci within neutrophils. What is the

most appropriate treatment?

A. Oral doxycycline

B. IV ceftriaxone

C. IV ceftriaxone and oral doxycycline

D. IV penicillin

Key: C

Page 35: 7. Sodium valproate

412. A 22-year-old female with a history of acquired

immunodeficiency syndrome (AIDS) develops low

grade fever, headache and blurred vision.

Cerebrospinal fluid (CSF) testing for Cryptococci

antigen is positive. Which of these drugs can be given

orally to treat Cryptococcal meningitis?

A. Amphotericin B

B. Caspofungin

C. Fluconazole

D. Metronidazole

Key: C

413. A 19-year-old man with a history of

immunodeficiency disease develops fever.

Prophylactic measures including some medications

are initiated. A few days later, the patient develops

acute renal failure. Which of these drugs is most

likely responsible?

A. Acyclovir

B. Amphotericin B

C. Ampicillin

D. Ciprofloxacin

Key: B

414. A 62-year old woman with a history of type 2

diabetes complains of nausea and bloating for the last

three months. Gastric emptying studies confirm the

diagnosis of diabetic gastroparesis. What is the first

line therapy for this patient?

A. Clindamycin

B. Metoclopramide

C. Omeprazole

D. Ondansetron

Key: B

415. A 45 year old man is admitted to the hospital for a

scheduled surgery. Two days later, he develops fever,

chills, cough and chest pain. This hospital has

reported several cases of methicillin-resistant

Staphylococcus aureus (MRSA) in the last 3 months.

What should have been given to this patient to prevent

his infection with MRSA?

A. Ampicillin

B. Ciprofloxacin

C. Gentamicin

D. Vancomycin

Key: D

416. A 72 year old woman presents to her physician

complaining of abdominal pain, frequent and burning

micturition. After physical examination and

laboratory tests, she is diagnosed with urinary tract

infection. She is prescribed gentamicin.

What is the most likely complication attributed to this

antibiotic?

A. Hepatic toxicity

B. Hearing loss

C. Thrombocytopenia

D. Agranulocytosis

Key: B

417. A 45 year old man who was diagnosed with malaria

and received anti-malarial medication presents to the

office with progressive fatigue and loss of appetite.

He also mentions that he noticed that his urine has

become dark lately. Urine analysis is positive for both

bilirubin and urobilinogen. Blood studies show serum

total bilirubin of 4.3 mg/dL, hemoglobin

concentration of 8.5 g/dL and hematocrit of 30%.

Which medication is used to treat this patient?

A. Atovaquone

B. Chloroquine

C. Doxycycline

D. Primaquine

Key: D

418. A 9-year-old boy presents to his physician with a

complaint of headache, sore throat and fever. After

taking history, the physician warned the parents

recommending that they should never administer

aspirin to the boy for the risk of having a disease that

is characterized by which of the following?

A. Bronchospasm

B. Gastritis

C. Chronic diarrhea

D. Seizures

Key: D, Reye's syndrome is a rare side effect of

aspirin especially in those who are recovering from a

flu or chickenpox. It affects mainly the liver and the

brain. It causes seizures and hepatomegaly.

419. MAO of amphotericin-B

A. Acts on DNA

B. Acts on RNA

C. Acts on sterols in membrane

D. Alters protein synthesis

Key: C

420. A 52-year old woman with a history of type 2

diabetes mellitus comes to the physician because of

facial swelling around her lips and eye. On her last

visit 3 weeks ago she was diagnosed with

Page 36: 7. Sodium valproate

hypertension and appropriate pharmacotherapy was

initiated. An image of the woman’s eyes is shown.

Which of the following agents could account for the

adverse drug reaction in this patient?

A. Clonidine

B. Atenolol

C. Hydrochlorothiazide

D. Lisinopril

Key: D

421. Postural (Orthostatic) hypotension is a significant

problem with all of the following agents except?

A. phenoxybenzamine

B. mecamylamine

C. guanethidine and prazosin

D. it is a problem with all of the above

Key: D

422. Which of the following antiarrhythmic agents has the

longest duration of action?

A. quinidine

B. amiodarone

C. verapamil

D. Lidocaine

Key: B

423. A 56-year old man at risk of symptomatic atrial

fibrillation/flutter is placed on prophylactic

pharmacotherapy. The ECG tracing below shows the

response before (solid line) and 2-weeks after (broken

line) commencement of treatment. Which of the

following agents was most likely prescribed for this

patient?

A. Verapamil

B. Metoprolol

C. Quinidine

D. Flecainide

Key: C

424. The vasodilator action of nitrates involves the

formation of ______ and an increase in the amount of

_____ in the vascular smooth muscle cell?

A. ATP, calmodulin

B. NO, troponin

C. NO, cGMP

D. disulfide bonds, calcium

Key: C

425. With respect to the treatment of angina, ALL

members of this drug class would cause reflex

tachycardia?

A. calcium blockers

B. beta blockers

C. nitrates

D. all of the above

426. Compensatory increases in heart rate and renin release

that occur in heart failure may be alleviated by which

of the following agents?

A. Milrinone

B. Lisinopril

C. Atenolol

D. Dobutamine

Key: C

427. A 60-year-old man is admitted to the hospital with

acute heart failure and pulmonary edema. Which of

the following drugs would be most useful in treating

the pulmonary edema?

A. Lisinopril

B. Dobutamine

C. Furosemide

D. Spironolactone

Key: C

428. Which of the following reduces myocardial oxygen

demand by decreasing contractility and heart rate?

A. Atenolol

B. Nifedipine

C. Prazosin

D. Hydrochlorothiazide

Key: A

429. A 54-year-old man is admitted to the emergency

department. He has taken more than 50 digoxin

tablets (0.25 mg each), ingesting them about 2 hours

before admission. His pulse is 54 b/min, and the ECG

shows third-degree heart block. Which of the

following is the most important therapy to initiate in

this patient?

A. SC cardioversion

B. Digoxin immune Fab

C. Lidocaine

D. Potassium salts

Key: B

430. A 54-year-old woman had a myocardial infarction.

Which of the following would be appropriate

prophylactic antiarrhythmic pharmacotherapy?

A. Quinidine

Page 37: 7. Sodium valproate

B. Procainamide

C. Verapamil

D. Metoprolol

Key: D

431. A 58-year-old woman is being treated for chronic

suppression of a ventricular arrhythmia. After 2

months of pharmacotherapy, she complains about

feeling tired all the time. Examination reveals a

resting heart rate of 10b/min lower than her previous

rate. Her skin is cool and clammy. Laboratory test

results indicate low thyroxin and elevated thyroid-

stimulating hormone levels. Which of the following

agents is most likely to have caused these signs and

symptoms?

A. Amiodarone

B. Procainamide

C. Verapamil

D. Quinidine

Key: A

432. A 54-year-old woman is being treated for an atrial

arrhythmia. She complains of headache, dizziness and

tinnitus. Which of the following antiarrhythmic drugs

is the most likely cause of these symptoms?

A. Amiodarone

B. Procainamide

C. Verapamil

D. Quinidine

Key: D

433. A 56-year-old woman complains of chest pain

following any sustained exercise. She is diagnosed

with atherosclerotic angina. She is prescribed

sublingual nitroglycerin for treatment of acute chest

pain. Which of the following adverse effects is likely

to be experienced by this patient?

A. Hypertension

B. Throbbing headache

C. Bradycardia

D. Sexual dysfunction

Key: B

434. A 56-year-old woman complains of chest pain

following any sustained exercise. She is diagnosed

with atherosclerotic angina. She is prescribed

sublingual nitroglycerin for treatment of acute chest

pain. Which of the following adverse effects of

sublingual nitroglycerin can be prevented by atenolol?

A. Dizziness

B. Throbbing headache

C. Methemoglobinemia

D. Reflex tachycardia

Key: D

435. A 68-year-old man has recently been diagnosed with

hypertension and started on monotherapy designed to

reduce peripheral resistance and prevent sodium and

water retention. Since commencing treatment he has

developed a persistent cough. Which of the following

drugs would have the same benefits but would not

cause the cough?

A. Lisinopril

B. Nifedipine

C. Prazosin

D. Losartan

Key: D

436. A 68-year-old man has been successfully treated for

exercise-induced angina for several years. He recently

has been awakened at night with chest pain. Which of

the following drugs would be useful in preventing this

patient’s nocturnal angina?

A. Amyl nitrite

B. Nitroglycerin (sublingual)

C. Nitroglycerin (transdermal)

D. Esmolol

Key: C

437. A diabetic woman with CHF and asthma.

A. Alpha-blocker i.e. Phenoxybenzamine

B. beta-blocker or Ca++-blocker

C. diuretic

D. cardiac glycoside or ACE inhibitor

.

Key: D

438. An obese 76 year old hypertensive male.

A. Alpha-blocker i.e. Phenoxybenzamine

B. beta-blocker or Ca++-blocker

C. diuretic

D. cardiac glycoside or ACE inhibitor

Key: C

439. A woman with diffuse Pheochromocytoma.

A. Alpha-blocker i.e. Phenoxybenzamine

B. beta-blocker or Ca++-blocker

C. diuretic

D. cardiac glycoside or ACE inhibitor

Key: A

Page 38: 7. Sodium valproate

440. A hypertensive 43 yr old male with supraventricular

tachycardia.

A. Alpha-blocker i.e. Phenoxybenzamine

B. beta-blocker or Ca++-blocker

C. diuretic

D. Cardiac glycoside or ACE inhibitor

Key: B

441. Parentally administered antiarrhythmic with an

extremely short half-life (15-20 secs).

A. Adenosine

B. Flecainide

C. Metoprolol

D. Verapamil

Key: A

442. Class IC agent producing marked slowing of AP

conduction.

A. Adenosine

B. Phenytoin

C. Flecainide

D. Verapamil

Key: C

443. Orally effective agent with actions similar to

Lidocaine

A. Adenosine

B. Phenytoin

C. Flecainide

D. Metoprolol

Key: B

444. Useful for suppressing the activity of the SA node,

may cause constipation

A. Phenytoin

B. Flecainide

C. Metoprolol

D. Verapamil

Key: D

445. Useful for suppressing the activity of the SA node,

may cause depression and sleep disturbances.

A. Adenosine

B. Phenytoin

C. Flecainide

D. Metoprolol

Key: D

446. The efficacy of beta blockers in hypertension:

A. is almost entirely due to inhibition of renin

secretion

B. is lower in whites than in blacks

C. can be enhanced by concurrent administration of

a diuretic

D. Compromised by adverse CNS effects ie.

stimulation & convulsions

Key: C

447. Minoxidil (Loniten) lowers vascular resistance

by_____ and is also used in the treatment of______.

A. opening K+ channels, baldness

B. opening K+ channels, angina

C. closing K+ channels, baldness

D. increasing cellular NO and cGMP, angina

Key: A

448. Problems associated with the use of ACE inhibitors

include all of the following except:

A. Hypertension

B. Potassium retention

C. Glossitis

D. Dry cough

Key: A

449. An agent whose cardiac actions include a strong

inotropic effect, but a relatively weak chronotropic

effect is:

A. Dobutamine

B. norepinephrine

C. epinephrine

D. isoproterenol

Key: A

450. Which of the following agent(s) are associated with

gingival hyperplasia?

A. calcium blockers

B. ACE inhibitors

C. Phenytoin

D. none

Key: C

451. A 53-year-old woman is being treated for atrial flutter

subsequent to a myocardial infarction. After

commencing treatment she experiences prolonged

depression, bradycardia and wheezing. Which of the

following agents was the patient prescribed?

A. propranolol

B. phenytoin

C. Flecainide

Page 39: 7. Sodium valproate

D. verapamil

Key: A

452. Which of the following is an undesired effect

associated with the use of Cholestyramine?

A. increased VLDL

B. increased LDL

C. decreased HDL

D. decreased triglycerides (TGs)

Key: A

453. The primary action of which of the following agents

is to increase the activity of lipoprotein lipase to

decrease VLDL levels?

A. Niacin

B. Cholestyramine

C. Atorvastatin

D. Gemfibrozil

Key: D

454. Which of the lipoproteins listed below is

antiatherogenic?

A. VLDL

B. IDL

C. HDL

D. LDL

Key: C

455. Which of the following agents can cause the greatest

decrease in triglyceride levels?

A. Niacin

B. Atorvastatin

C. Gemfibrozil

D. Ezetimibe

Key: D

456. Which of the following agents can precipitate a

hypertensive crisis following abrupt cessation of

therapy?

A. Hydrochlorothiazide

B. Diltiazem

C. Losartan

D. Clonidine

Key: D

457. A 48-year old hypertensive man has been successfully

treated with a thiazide diuretic for the past 4 years.

Over the last 3 months his diastolic pressure has

steadily increased and he has been started on an

additional antihypertensive medication. He now

complains of sleep disturbances and he is no longer

able to complete three sets of tennis. The second

antihypertensive medication is most likely which of

the following agents?

A. Metoprolol

B. Lisinopril

C. Nifedipine

D. Losartan

Key: A

458. Adverse effects associated with the use of amiodarone

include all of the following except:

A. Cardiac depression & Increased QT interval

B. Pulmonary fibrosis

C. Thyroid dysfunction

D. All of the above can occur

Key: D

459. A 38-year old woman who is a marathon runner is

diagnosed with hypertension. Which of the following

agents would be the most suitable to treat her

hypertension and accommodate her lifestyle?

A. Metoprolol

B. Lisinopril

C. Nifedipine

D. Clonidine

Key: D

460. A 36-year old woman who is pregnant is diagnosed

with hypertension. Which of the following agents

would be the most suitable to treat her hypertension?

A. Atenolol

B. alpha-methyl dopa

C. Nifedipine

D. Clonidine

Key: B

461. Which of the following drug combinations has been

found to be very effective as a pharmacotherapy for

heart failure in Afro-Americans?

A. Hydralazine and Lisinopril

B. Isosorbide dinitrate and Hydralazine

C. Metoprolol and Lisinopril

D. Metoprolol and Hydralazine

Key: B

462. The development of tolerance is a major concern with

which of the following agents?

A. Clonidine

B. Hydralazine

C. Metoprolol

Page 40: 7. Sodium valproate

D. Nitroglycerin

Key: D

463. Which of the following agents would be appropriate

pharmacotherapy for an individual with hypertension

and benign prostrate hypertrophy?

A. Clonidine

B. Atenolol

C. Prazosin

D. Phenoxybenzamine

Key: C

464. Which of the following agents would decrease the

therapeutic action of digoxin for heart failure?

A. Dobutamine

B. Atenolol

C. Furosemide

D. Lisinopril

Key: D

465. A 54-year old woman with a history of pulmonary

embolism controlled with warfarin comes to the

physician because of bleeding gums and easy

bruising. On her last visit 2 weeks ago she was

diagnosed with atrial fibrillation and an

antiarrhythmic agent was prescribed. Which of the

following agents could account for the increased

anticoagulant action of warfarin?

A. Amiodarone

B. Quinidine

C. Flecainide

D. Sotalol

Key: A

466. Beneficial effects of ______ in CHF may include

decreased afterload, and alpha1-adrenoceptor

blocking action.

A. Milrinone

B. carvedilol

C. hydrochlorothiazide

D. isosorbide dinitrate

Key: B

467. Drug not used in H. pylori is:

A. Metronidazole

B. Omeprazole

C. Mosapride

D. Amoxicillin

Key: C

468. Which of the following drugs is not used for H. pylori

treatment?

A. Oxytetracycline

B. Bismuth compounds

C. Amoxicillin

D. Omeprazole

Key: A

469. Proton pump inhibitors are most effective when they

are given:

A. After meals

B. Shortly before meals

C. Along with H2 blockers

D. During prolonged fasting periods

Key: B

470. Gynaecomastia can occur as a side effect of:

A. Bromocriptine

B. Cimetidine

C. Famotidine

D. Levodopa

Key: B

471. Ranitidine differs from cimetidine in the following

respect:

A. It is less potent

B. It is shorter acting

C. It does not have anti-androgenic action

D. It produces more CNS side effects

Key: C

472. Drug of choice for the treatment of peptic ulcer

caused due to chronic use of NSAIDs is:

A. Pirenzepine

B. Loxatidine

C. Misoprostol

D. Esomeprazole

Key: D, Proton pump inhibitors are the drugs of

choice for peptic ulcer disease due to any etiology.

Misoprostol is the MOST SPECIFIC drug for the

treatment of PUD due to chronic NSAID use because

it is a PGE1 analog.

473. Most specific drug for the treatment of peptic ulcer

disease due to chronic use of aspirin is:

A. Omeprazole

B. Misoprostol

C. Pirenzipine

D. Ranitidine

Key: B

Page 41: 7. Sodium valproate

474. Choose the antiulcer drug that inhibits gastric acid

secretion, stimulates gastric mucus and bicarbonate

secretion and has cytoprotective action on gastric

mucosa:

A. Misoprostol

B. Sucralfate

C. Carbenoxolone sodium

D. Colloidal bismuth subcitrate

Key: A

475. Cimetidine inhibits the metabolism of all of the

following drugs EXCEPT:

A. Phenytoin

B. Warfarin

C. Ketoconazole

D. Diazepam

Key: C

476. Drug used in the treatment of gastric ulcer due to

H.pylori is:

A. Anticholinergics

B. Carbenoxolone sodium

C. Bismuth sub citrate

D. Corticosteroid

Key: C

477. Which one of the following is not an antacid?

A. Magnesium sulfate

B. Magaldrate

C. Magnesium carbonate

D. Magnesium phosphate

Key: A

478. NSAID induced ulcer are treated by:

A. Antacids

B. H2 blockers

C. Misoprostol

D. PPI (proton pump inhibitors)

Key: D

479. Esomeprazole acts by inhibiting:

A. H+K+ ATPase pump

B. H+Na+ ATPase pump

C. H+ pump

D. None of the above

Key: A

480. Anti-peptic ulcer drug that can be given in patients

with chronic renal failure

A. Aluminium Hydroxide

B. Magnesium Hydroxide

C. Sucralfate

D. None

Key: C

481. Antacid drug that typically causes diarrhea?

A. Sodium bicarbonate

B. Magnesium hydroxide

C. Calcium bicarbonate

D. Aluminium hydroxide

Key: B

482. A patient presents with Zollinger-Ellison syndrome

due to gastrinoma. He has two bleeding ulcers and

diarrhoea. A drug that irreversibly inhibits the H+/K+

ATPase in gastric parietal cells is:

A. Cimetidine

B. Cisapride

C. Glycopyrrolate

D. Omeprazole

Key: D

483. Which of the following is an antagonist of a peptide

and is used to reduce chemotherapy induced nausea

and vomiting?

A. Atrial natriuretic peptide

B. Aprepitant

C. Bradykinin

D. Enalapril

Key: B

484. Drug given for metoclopramide induced dystonic

reaction is:

A. Pheniramine

B. Promethazine

C. Chlorpromazine

D. Prochlorperazin

Key: B

485. Regarding aprepitant all are true except:

A. Agonist at NK1 receptors

B. Crosses Blood Brain Barrier

C. Ameliorate nausea and vomiting induced by

chemotherapy

D. Metabolized by CYP450 enzymes

Key: A

Page 42: 7. Sodium valproate

486. An anti-emetic drug that also decreases acid secretion

due to its action on H

1 receptors is:

A. Promethazine

B. Domperidone

C. Metoclopramide

D. Ondansetron

Key: A

487. Metoclopramide

A. Inhibit cholinergic smooth muscle stimulation in

the gastrointestinal tract

B. Decrease lower esophageal sphincter pressure

C. Stimulate D2 receptor

D. Enhance colonic motility

Key: D

488. Drug implicated in prolonging QT interval is:

(a) Domperidone

(b) Metoclopramide

(c) Cisapride

(d) Omeprazole

Key: C

489. For chemotherapy induced vomiting, 5HT3 antagonist

having maximum potency is:

A. Ondansetron

B. Granisetron

C. Dolasetron

D. Palonosetron

Key: D

490. Which of the following drugs is not an antiemetic?

A. Ondansetron

B. Domperidone

C. Metoclopramide

D. Cinnarizine

Key: D

491. Anti-emetic action is produced through:

A. Decreased CTZ stimulation

B. H1 agonistic action

C. D1 antagonistic action

D. 5 HT4 agonistic action

Key: A & D

492. Ondansetron acts by:

A. Acting on CTZ & 5-HT3 antagonism

B. D1 and D2 receptor antagonism

C. Increasing GIT motility

D. Blocking cholinergic receptors

Key: A

493. In case of hill journey, antimotion sickness drugs are

best administered at:

A. Twelve hours before commencing journey

B. One hour before commencing journey

C. Immediately after commencing journey

D. At the first feeling of motion sickness

Key: B

494. Metoclopramide has the following actions EXCEPT:

A. Increase lower esophageal sphincter tone

B. Prokinetic action is blocked by atropine

C. Increase gastric peristalsis

D. Increase large intestinal peristalsis

Key: A

495. Which of the following prokinetic drugs produces

extrapyramidal side effects?

A. Metoclopramide

B. Cisapride

C. Domperidone

D. All of the above

Key: A

496. Which antiemetic drug selectively blocks levodopa

induced vomiting without blocking its anti-

Parkinsonian action?

A. Metoclopramide

B. Cisapride

C. Domperidone

D. Ondansetron

Key: C

497. The most effective antiemetic for controlling

Cisplatin induced vomiting is:

A. Prochlorperazine

B. Ondansetron

C. Metoclopramide

D. Aprepitant

Key: B

498. Which of the following prokinetic drugs has been

implicated in causing serious ventricular arrhythmias,

particularly in patients concurrently receiving

erythromycin or ketoconazole?

A. Domperidone

B. Cisapride

C. Mosapride

Page 43: 7. Sodium valproate

D. Metoclopramide

Key: B

499. Ondansetron acts by inhibiting which of the following

receptors?

A. 5-HT1

B. 5-HT2

C. 5-HT3

D. 5-HT

Key: C

500. Drug used in irritable bowel syndrome with

constipation is:

A. Lubiprostone

B. Loperamide

C. Alosetron

D. Clonidine

Key: A

501. Which of the following agents are useful in medical

treatment of variceal bleeding?

A. Octreotide

B. Pantoprazole

C. Somatotropin

D. Dexamethasone

Key: A

502. Which of the following laxatives lowers blood

ammonia level in hepatic encephalopathy?

A. Bisacodyl

B. Liquid paraffin

C. Lactulose

D. Magnesium sulfate

Key: C

503. The preferred drug for controlling an acute

exacerbation of ulcerative colitis is:

A. Prednisolone

B. Sulfasalazine

C. Mesalazine

D. Vancomycin

Key: A

504. A small amount of atropine is added to diphenoxylate

in order to:

A. Suppress associated vomiting of gastroenteritis

B. Augment the anti-motility action of

diphenoxylate

C. Block side effects of diphenoxylate

D. Discourage overdose and abuse of diphenoxylate

Key: D

505. Bisacodyl is:

A. Bulk forming

B. Stool softner

C. Stimulant purgative

D. Osmotic purgative

Key: C

506. Sulfa drug used in inflammatory bowel disease

includes:

A. Sulfasalazine

B. Sulfamethoxazole

C. Sulfinpyrazone

D. Sulphadoxine

Key: A

507. Which one of the following drugs has been shown to

offer protection from gastric aspiration syndrome in a

patient with symptoms of reflux?

A. Ondansetron

B. Metoclopramide

C. Sodium citrate

D. Atropine

Key: B

508. In a patient taking oral contraceptive, the chance of

pregnancy increases after taking any of the following

drugs except :

A. Phenytoin

B. Griseofulvin

C. Ampicillin

D. Cimetidine

Key: D

509. A patient is taking 40mg Famotidine OD, Sucralfate

and antacid tablets TDS. This treatment is irrational,

because of:

A. Sucralfate decreases the absorption of famotidine

B. Sucralfate increases the toxicity of famotidine

C. Sucralfate decreases absorption of antacids

D. Sucralfate polymerises only when gastric pH is

less than 4

Key: D

510. Most potent anti-emetic is:

A. Ondansertron

B. Granisertron

C. Dolasetron

D. Palonosetron

Page 44: 7. Sodium valproate

Key: D

511. A patient is posted for elective surgery. Which of the

following drugs should be stopped on the day of

surgery:

A. Atenolol

B. Amlodipine

C. Statins

D. Metformin

Key: D

512. Anticoagulant effect of warfarin is increased by all of

the following except:

A. Cimetidine

B. Phytonadione

C. Amiodarone

D. Phenylbutazone

Key: B

513. Granulomatous hepatitis is caused by :

A. Allopurinol

B. Methyldopa

C. Furazolidone

D. Amiodarone

Key: A

514. Drug of choice for malaria during pregnancy is :

A. Chloroquine

B. Quinine

C. Primaquine

D. Mepacrine

Key: A

515. The treatment of contacts of meningococcal

meningitis is by :

A. Rifampicin

B. Erythromycin

C. Penicillin

D. Cephalosporins

Key: A

516. Mebeverine hydrochloride is useful in:

A. Ulcerative colitis

B. Irritable bowel syndrome

C. Peptic ulcers

D. Crohn’s disease

Key: B

517. Which of the following antibiotics acts by inhibiting

cell wall synthesis?

A. Cefepime

B. Aminoglycosides

C. Erythromycin

D. Doxycycline

Key: A

518. Which antibiotic acts by inhibiting protein synthesis?

A. Cefotetan

B. Doxycycline

C. Ciprofloxacin

D. Oxacillin

Key: B

519. Which of the followign drugs require dose adjustment

in renal failure?

A. Cefoperazone

B. Doxycycline

C. Streptomycin

D. Rifampicin

Key: C

520. Antimicrobial agent acting by inhibition of cell wall

synthesis is:

A. Erythromycin

B. Tetracycline

C. Lomefloxacin

D. Cefepime

Key: D

521. All of the following antibacterial agents act by

inhibiting cell wall synthesis EXCEPT:

A. Carbapenems

B. Monobactams

C. Cephalosporins

D. Nitrofurantoin

Key: D

522. A post-operative patient developed septicemia and

was empirically started on combination chemotherapy

by a new resident doctor. However, when the patient

did

not respond even after 10 days of antibiotics

treatment,

the review of the charts was done. It was found that

that the resident doctor had started the combination of

antibiotics which was mutually antagonistic in action.

Which of the following is the most likely combination

that was given?

A. Vancomycin and Amikacin

Page 45: 7. Sodium valproate

B. Cephalexin and Gentamicin

C. Ampicillin and Chloramphenicol

D. Ciprofloxacin and Piperacillin

Key: C

523. All of the following drugs act on cell membrane

EXCEPT:

A. Nystatin

B. Griseofulvin

C. Amphotericin B

D. Polymyxin B

Key: B

524. Bacitracin acts on:

A. Cell wall

B. Cell membrane

C. Nucleic acid

D. Ribosome

Key: A

525. All of the following antibiotics act by interfering with

cell wall formation EXCEPT:

A. Ceftriaxone

B. Vancomycin

C. Cycloserine

D. Clindamycin

Key: D

526. Which of the following drugs is NOT excreted in bile:

A. Erythromycin

B. Ampicillin

C. Rifampicin

D. Gentamicin

Key: D

527. Multiple drug resistance is transferred through:

A. Transduction

B. Transformation

C. Conjugation

D. Mutation

Key: C

528. Most common mechanism for transfer of resistance in

Staphylococcus aureus is:

A. Conjugation

B. Transduction

C. Transformation

D. Mutation

Key: B

529. Pneumococcal resistance to penicillin G is mainly

acquired by:

A. Conjugation

B. Transduction

C. Transformation

D. All of the above

Key: C

530. A bactericidal drug would be preferred over a

bacteriostatic drug in a patient with:

A. Neutropenia

B. Cirrhosis

C. Pneumonia

D. Heart disease

Key: A

531. Which of the following drugs acts by inhibiting cell

wall synthesis?

A. Erythromycin

B. Cephalosporins

C. Chloramphenicol

D. Sulfonamides

Key: B

532. In Staphylococci, plasmids encoding beta-lactamase

are transmitted by:

A. Conjugation

B. Transduction

C. Transposon

D. Transformation

Key: B

533. Which of the following drug is bactericidal?

A. Sulfonamides

B. Erythromycin

C. Chloramphenicol

D. Cotrimoxazole

Key: D

534. Superinfection is common in:

A. Narrow spectrum antibiotics

B. Immunocompromised host

C. Low spectrum antibiotics

D. Nutritional deficiency

Key: B

535. Which of the following antibiotic does not act by

inhibiting protein synthesis?

A. Vancomycin

Page 46: 7. Sodium valproate

B. Tetracycline

C. Streptomycin

D. Azithromycin

Key: A

536. Which of the following antimicrobial is effective

against an organism producing extended spectrum

beta lactamase?

A. Amoxicillin–Clavulinic acid

B. Cefepime

C. Piperacillin-Tazobactam

D. Ceftriaxone

Key: C

537. Which of the following statement about Penicillin G

is true?

A. It is commonly administered orally

B. It has a broad spectrum of antibacterial activity

C. It can be used for the treatment rate bite fever

D. Concomitant probenecid decreases its duration of

action

Key: C

538. Which of the following beta-lactam antibiotics can be

safely used in a patient with a history of allergy to

penicillins?

A. Aztreonam

B. Cefepime

C. Loracarbef

D. Ceftriaxone

Key: A

539. All of the following statements about penicillin

binding proteins are true EXCEPT:

A. Present on cell surface

B. Mutation in PBPs gives rise to resistance

C. These are target site of vancomycin

D. These are targeted by imipenem

Key: C

540. Cephalosporin that does not require dose reduction in

patient with any degree of renal impairment is:

A. Cefuroxime

B. Cefoperazone

C. Ceftazidime

D. Cefotaxime

Key: B, Ceftriaxone and cefoperazone are excreted

mainly in the bile, therefore do not require dose

reduction in any grade of renal failure.

541. Extended spectrum beta lactamases (ESBLs) are

characterized by activity against all except:

A. Penicillinases

B. Cephalosporinases

C. Oxyimino-cephalosporinases

D. Carbapenems

Key: D

542. All of the following statements about penicillin-G are

true EXCEPT

A. It is actively secreted in tubules

B. It is never administered orally

C. It is effective against gram positive as well as

some gram negative bacteria

D. It acts by inhibiting cell wall synthesis

Key: B

543. All of the following are the therapeutic uses of

penicillin-G EXCEPT:

A. Bacterial meningitis

B. Rickettsial infection

C. Syphilis

D. Anthrax

Key: B

544. Which one of the following drugs is an

antipseudomonal penicillin?

A. Cephalexin

B. Cloxacillin

C. Piperacillin

D. Dicloxacillin

Key: C

545. One of the following is not penicillinase susceptible?

A. Amoxicillin

B. Penicillin G

C. Piperacillin

D. Cloxacillin

Key: D

546. Which of the following antimicrobials has

antipseudomonal action?

A. Cefpodoxime proxetil

B. Cephradine

C. Cefotetan

D. Cefoperazone

Key: D

547. Which of the following is a fourth generation

cephalosporin?

Page 47: 7. Sodium valproate

A. Ceftriaxone

B. Cefaclor

C. Cefepime

D. Cefuroxime

Key: C

548. The mechanism of antibacterial action of

cephalosporins involves:

A. Inhibition of the synthesis of precursors of

peptidoglycan

B. Interference with the synthesis of ergosterol

C. Inhibition of transpeptidation reaction

D. Inhibition of beta-lactamase

Key: C

549. Which of the following statements about imipenem is

most accurate?

A. The drug has a narrow spectrum of antibacterial

action

B. It is used in fixed dose combination with

sulbactum

C. In renal dysfunction, dosage reductions are

necessary to avoid seizures

D. Imipenem is active against methicillin-resistant

staphylococci

Key: C

550. Methicillin resistant staphylococci do not respond to

β-lactam antibiotics because:

A. They produce a β-lactamase which destroys

methicillin and related drugs

B. They elaborate an amidase which destroys

methicillin and related drugs

C. They have acquired penicillin binding protein

which has low affinity for β-lactam antibiotics

D. They are less permeable to β-lactam antibiotics

Key: C

551. The penicillin G preparation with the longest duration

of action is:

A. Benzathine penicillin

B. Sodium penicillin

C. Potassium penicillin

D. Procaine penicillin

Key: A

552. Amoxicillin + clavulanic acid is active against the

following organisms EXCEPT:

A. Methicillin resistant Staph. aureus

B. Penicillinase producing Staph. aureus

C. Penicillinase producing N. gonorrhoea

D. β-lactamase producing E. coli

Key: A

553. The following is true of vancomycin EXCEPT:

A. It is a bactericidal antibiotic active primarily

against gram positive bacteria

B. It acts by inhibiting bacterial protein synthesis

C. It is an alternative to penicillin for enterococcal

endocarditis

D. It can cause deafness as a dose related toxicity

Key: B

554. TRUE statement regarding vancomycin is:

A. It is bacteriostatic

B. It has the advantage of high oral bioavailability

C. It is not susceptible to penicillinases

D. Staphylococcal enterocolits occurs commonly

with its use

Key: C

555. This drug has activity against many strains of P.

aeruginosa. However, when it is used alone,

resistance has emerged during the course of treatment.

The drug should not be used in penicillin-allergic

patients. Its activity against gram-negative rods is

enhanced if it is given in combination with

tazobactam. Which of the following drugs is being

described?

A. Amoxicillin

B. Aztreonam

C. Piperacillin

D. Vancomycin

Key: C

556. A 36 years old woman recently treated for leukemia is

admitted to the hospital with malaise, chills and high

fever. Gram stain of blood reveals the presence of

gram negative bacilli. The initial diagnosis is

bacteremia and parenteral antibiotics are indicated.

The record of the patient reveals that she had severe

urticarial rash, hypotension and respiratory difficulty

after oral pencillin V about 6 months ago. The most

appropriate drug should be:

A. Ampicillin plus sulbactum

B. Aztreonam

C. Cefazolin

D. Imipenem plus cilastatin

Key: B

557. Not true about cefepime is:

A. 4th generation cephalosporin

Page 48: 7. Sodium valproate

B. Useful in hospital acquired infection

C. Inhibits transpeptidase

D. Given twice daily orally

Key: D

558. Antipseudomonals are all, EXCEPT:

A. Cephalexin

B. Carbenicillin

C. Piperacillin

D. Ceftazidime

Key: A

559. Cilastatin is given along with:

A. Imipenem

B. Amoxicillin

C. Erythromycin

D. Ampicillin

Key: A

560. Which of the following cephalosporins is active

against

Pseudomonas aeruginosa?

A. Ceftriaxone

B. Cephalothin

C. Ceftazidime

D. Cefotaxime

Key: C

561. Which of the following is NOT true about penicillins?

A. Penicillin V is absorbed orally

B. Benzathine penicillin is short acting penicillin

C. Cloxacillin is β-lactamase and acid resistant

D. Ampicillin is not resistant to β-lactamases

Key: B

562. Mechanism of action of vancomycin is:

A. Inhibition of cell wall synthesis

B. Inhibition of protein synthesis

C. Leakage from cell membrane

D. Inhibition of DNA gyrase

Key: A

563. Carbenicillin:

A. Is effective in pseudomonas infection

B. Has no effect in Proteus infection

C. Is a macrolide antibiotic

D. Is administered orally

Key: A

564. A potent inhibitor of beta-lactamase is:

A. Carbenicillin

B. Clavulanic acid

C. Cefamandole

D. Idoxuridine

Key: B

565. All are true about cefuroxime except:

A. Inhibit cell wall synthesis

B. Third generation cephalosporin

C. Some acquired resistance with penicillin

D. More active against gram negative organisms

Key: B

566. Amoxycillin is better than ampicillin due to:

A. Better bioavailability if taken with food

B. Lesser bioavailability if taken with food

C. Incidence of diarrhea is higher

D. More active against Shigella and H. influenza

Key: A

567. Mechanism of action of penicillins and

cephalosporins is to inhibit:

A. Cell wall synthesis

B. Leakage from cell membrane

C. Protein synthesis

D. DNA gyrase

Key: A

568. The following organisms are known to develop

resistance to Penicillin except:

A. Staphylococcus

B. Streptococcus

C. Pneumococcus

D. Treponema

Key: D

569. Ceftriaxone is:

A. IInd generation short acting cephalosporin

B. Has activity against beta lactamase producing

bacteria

C. IVth generation long acting cephalosporin

D. IIIrd generation long acting cephalosporin

Key: D

570. Acid susceptible penicillin is:

A. Methicillin

B. Ampicillin

C. Amoxicillin

D. Cloxacillin

Page 49: 7. Sodium valproate

Key: A

571. All are first generation cephalosporins except:

A. Cefadroxil

B. Cefazolin

C. Cephalexin

D. Cefaclor

Key: D

572. Which is not a beta lactum antibiotic?

A. Penicillin

B. Carbepenem

C. Monobactum

D. Azithromycin

Key: D

573. Second generation cephalosporin that can be used

orally is:

A. Cefepime

B. Cefalothin

C. Cefaclor

D. Cefadroxil

Key: C

574. Third generation cephalosporin that can be given

orally is:

A. Cefixime

B. Cefpirome

C. Cefaclor

D. Cefadroxil

Key: A

575. Ampicillin is not given in Epstein-Bar virus (EBV)

infection due to:

A. Due to increased toxicity

B. Skin rash

C. Blindness

D. Convulsions

Key: B

576. Which among the following is not a beta lactamase

inhibitor?

A. Sulbactam

B. Calvulanic acid

C. Piperacillin

D. None

Key: C

577. Oral cephalosporin among these is:

A. Cefotaxime

B. Ceftriaxone

C. Cefaclor

D. Ceftazidime

Key: C

578. Beta lactam antibiotics act by inhibiting

A. Cell wall synthesis

B. Protein synthesis

C. RNA synthesis

D. DNA synthesis

Key: A

579. Which one of the following is a fourth generation

cephalosporin?

A. Cefuroxime

B. Ceftazidime

C. Cefepime

D. Cefamandole

Key: C

580. Neutropenia is associated with:

A. Nafcillin

B. Methicillin

C. Carbencillin

D. Ampicillin

Key: A

581. Which of the following mechanism is mainly

responsible for gentamicin induced ototoxicity?

A. Direct hair cell toxicity

B. Binding to and inhibition of hair cell Na+ K+

ATPase

C. Non-cumulative toxicity

D. Bind to Ca2+ channels

Key: A

582. Tetracycline is used for the prophylaxis of?

A. Cholera

B. Brucellosis

C. Leptospirosis

D. Meningitis

Key: A

583. Which of the following should be monitored if

linezolid is given for more than 14 days?

A. Liver function tests

B. Kidney function test

C. Platelet count

D. Audiometry

Page 50: 7. Sodium valproate

Key: C

584. Erythromycin is given in intestinal hypomotility

because:

(a) It increases bacterial count

(b) It decreases bacterial count

(c) It binds to adenylyl cyclase

(d) It binds to motilin receptors

Key: D

585. True about aminoglycosides is all EXCEPT:

A. Are bacteriostatic

B. Distributed only extracellularly

C. Excreted unchanged in urine

D. Teratogenic

Key: A

586. Which of the following is not true regarding

tetracycline?

A. It is not teratogenic

B. It can cause tooth discoloration

C. It can result in superinfection

D. It can lead to pseudomembranous colitis

Key: A

587. The group of antibiotics that possesses additional anti-

inflammatory and immunomodulatory activities is:

A. Tetracyclines

B. Polypeptide antibiotics

C. Fluoroquinolones

D. Macrolides

Key: D

588. All of the following are risk factors for renal toxicity

caused by aminoglycosides EXCEPT:

A. Elderly patient

B. Hypokalemia

C. Simultaneous use of penicillin

D. Aminoglycoside administration in recent past

Key: C

589. Tetracyclines inhibit protein synthesis by:

A. Inhibition of initiation and misreading of mRNA

B. Binding to 30 S subunit and inhibiting the

binding of aminoacyl-tRNA to A site

C. Inhibiting peptidyl transferase activity

D. Inhibiting translocation

Key: B

590. The antibiotic that inhibits protein synthesis by

premature termination and which structurally

resembles amino acyl t-RNA is:

A. Tetracycline

B. Chloramphenicol

C. Puromycin

D. Erythromycin

Key: C

591. Ototoxicity of aminoglycoside is increased with

concurrent use of which of the following drug (s):

A. Cisplatin & Erythromycin

B. Furosemide & Vancomycin

C. Vincristine

D. None

Key: D

592. The following tetracycline has the potential to cause

vestibular toxicity:

A. Minocycline

B. Demeclocycline

C. Doxycycline

D. Tetracycline

Key: A

593. The most suitable tetracycline for use in a patient with

impaired renal function is:

A. Tetracycline

B. Demeclocycline

C. Oxytetracycline

D. Doxycycline

Key: D

594. The most important mechanism by which gram

negative bacilli acquire chloramphenicol resistance is:

A. Decreased permeability into the bacterial cell

B. Acquisition of a plasmid encoded for

chloramphenicol acetyl transferase

C. Lowered affinity of the bacterial ribosome for

chloramphenicol

D. Switching over from ribosomal to mitochondrial

protein synthesis

Key: B

595. Bactericidal inhibitors of protein synthesis are:

A. Tetracyclines

B. Aminoglycosides

C. Macrolides

D. Lincosamides

Key: B

Page 51: 7. Sodium valproate

596. Hepatitis with cholestatic jaundice occurs most

frequently as an adverse reaction to the following

preparation of erythromycin:

A. Erythromycin base

B. Erythromycin stearate

C. Erythromycin estolate

D. Erythromycin ethyl succinate

Key: C

597. The tetracycline with highest antileprotic activity is:

A. Minocycline

B. Doxycycline

C. Demeclocycline

D. Oxytetracycline

Key: A

598. Which of the following drugs is most effective against

an organism producing aminoglycoside inactivating

enzymes?

A. Amikacin

B. Streptomycin

C. Gentamicin

D. Tobramycin

Key: A

599. The mechanism of action of tetracyclines involves:

A. Binding of a component of the 50S ribosomal

subunits

B. Inhibition of translocase activity

C. Blockade of binding of aminoacyl-tRNA to

bacterial ribosomes

D. Selective inhibition of ribosomal peptidyl

transferase

Key: C

600. This inhibitor of bacterial protein synthesis has a

narrow spectrum of antibacterial activity. It has been

used in the management of abdominal abscess caused

by Bacteroides fragilis, but antibiotic associated

colitis has occurred. Which of the following drugs is

being described?

A. Clarithromycin

B. Clindamycin

C. Minocycline

D. Ticarcillin

Key: B

601. Which of the following drugs act by inhibiting

bacterial protein synthesis?

A. Bacitracin

B. Dapsone

C. Ethambutol

D. Streptomycin

Key: D

602. Which of the following aminoglycosides has highest

nephrotoxicity?

A. Paramomycin

B. Streptomycin

C. Amikacin

D. Neomycin

Key: D

603. Auditory toxicity is maximum with:

A. Streptomycin

B. Kanamycin

C. Tobramycin

D. Amikacin

Key: D

604. Erythromycin acts by interfering with

A. Translocation of 50S ribosome

B. Translocation of 50S ribosome

C. Transcription of 50 S ribosome

D. Signal transduction of 50 S ribosome

Key: A

605. Single dose aminoglycoside administration is more

preferable than 8 hourly dose because of:

A. MIC

B. Increase perfusion of renal cortex

C. Post antibiotic effect

D. None

Key: C

606. Linezolid is best used for

A. MRSA

B. VRSA

C. K.pneumoniae

D. coli

Key: B

607. Doxycycline is used in the treatment of following

diseases EXCEPT:

A. Leptospirosis

B. Q fever

C. Borrelliosis

D. All of the above

Key: D

Page 52: 7. Sodium valproate

608. Fluoroquinolone having longest half-life is:

A. Levofloxacin

B. Lomefloxacin

C. Ciprofloxacin

D. Moxifloxacin

Key: D

609. Cotrimoxazole can be used for the treatment of all of

the following except:

A. Chancroid

B. Lower urinary tract infections

C. Prostatitis

D. Typhoid

Key: A

610. A girl on sulphonamides developed abdominal pain

and presented to emergency with seizure. What is the

probable cause?

A. Acute intermittent porphyria

B. Congenital erythropoietic porphyria

C. Infectious mononucleosis

D. Kawasaki’s disease

Key: A

611. Folic acid metabolism is inhibited by:

A. Sulfonamides

B. Methotrexate

C. Nitrous oxide

D. Trimethoprim

E. 5-Flucytosine

Key: A

612. Which of the following blocks replication without

getting involved in the DNA strand?

A. Cytarabine & 6-Mercaptopurine

B. Nalidixic acid & Ciprofloxacin

C. 5-Fluorouracil

D. All

Key: B

613. In unconjugated hyperbilirubinemia the risk of

kernicterus increases with the use of:

A. Ceftriaxone

B. Phenobarbitone

C. Ampicillin

D. Sulfonamide

Key: D

614. All of the following are topically used sulfonamides

EXCEPT:

A. Sulfacetamide

B. Sulfasalazine

C. Silver sulfadiazine

D. Mafenide

Key: B

615. Which of the following statements is NOT true

regarding sulfonamides?

A. Sulfasalazine is absorbed well from GIT

B. Crystalluria can occur with sulfonamide

administration

C. Sulfonamide administration to newborn may

cause kernicterus

D. Sulfonamides are of value in treatment of

infections due to Norcardia species.

Key: A

616. Which of the following fluoroquinolones does not

require dose adjustment in a patient with creatinine

clearance of < 50 mg/min?

A. Ciprofloxacin

B. Trovafloxacin

C. Lomefloxacin

D. Sparfloxacin

Key: B

617. Sparfloxacin and astemizole can cause:

A. Ventricular arrhythmia

B. Myopathy

C. Electrolyte imbalance

D. Nephropathy

Key: A

618. Which of the following statements about

sulfonamides is FALSE?

A. Sulfonamides inhibit bacterial dihydrofolate

reductase

B. Dysfunction of the basal ganglia may occur in the

newborn if sulfonamides are administered late in

pregnancy

C. Sulfonamide crystalluria is most likely to occur at

low urinary pH

D. Sulfonamides are antimetabolites

Key: A

619. Which of the following statements about the

fluoroquinolones is FALSE?

A. Gonococcal resistance to fluoroquinolones may

involve changes in DNA gyrase

Page 53: 7. Sodium valproate

B. Modification of fluoroquinolones dosage is

required in patients if creatinine clearance is less

than 50 mL/ min

C. A fluoroquinolone is the drug of choice for

treatment of an uncomplicated urinary tract

infection in a 7 year-old girl

D. Fluoroquinolones inhibit relaxation of positively

supercoiled DNA

Key: C

620. Which of the following adverse effects is most likely

to occur with sulfonamides?

A. Neurologic effects including headache, dizziness,

and lethargy

B. Hematuria

C. Fanconi’s anemia

D. Skin reactions

Key: D

621. Which fluoroquinolone is highly active against

Mycobacterium leprae and is being used in alternative

multidrug therapy regimens:

A. Norfloxacin

B. Ofloxacin

C. Ciprofloxacin

D. Lomefloxacin

Key: B

622. Maximum incidence of phototoxicity is associated

with:

A. Norfloxacin

B. Sparfloxacin

C. Lomefloxacin

D. Cotrimoxazole

Key: B

623. Methanamine salts are used as urinary antiseptics.

The reason they lack systemic antibacterial action is

that they are:

A. Not absorbed into systemic circulation after oral

use

B. Rapidly metabolized by liver drug metabolizing

enzymes

C. Converted to formaldehyde at low urinary pH

D. Substrates for active tubular secretion

Key: C

624. A contraindication to the use of ciprofloxacin is a

history of:

A. Epilepsy

B. Deep vein thrombosis

C. Gout

D. G-6 PD deficiency

Key: A

625. The combination of trimethoprim and

sulphamethoxazole is effective against which of the

following opportunistic infections in the AIDS

patient?

A. Disseminated Herpes simplex

B. Cryptococcal meningitis

C. Pneumocystis jiroveci

D. Tuberculosis

Key: C

626. Ciprofloxacin should not be given to an asthmatic

using theophylline because:

A. Ciprofloxacin inhibit theophylline metabolism

B. Theophylline inhibits ciprofloxacin metabolism

C. Ciprofloxacin decreases effect of theophylline

D. Theophylline induces metabolism of

ciprofloxacin

Key: A

627. Mechanism of action of fluoroquinolones is:

A. Inhibits cell wall synthesis

B. Inhibits protein synthesis

C. Inhibits DNA gyrase

D. Interferes with intermediary metabolism

Key: C

628. Eye drops of which sulphonamide is used clinically?

(a) Sulfacetamide

(b) Sulfamethoxazole

(c) Sulfinpyrazone

(d) All

Key: A

629. Which is of the following can be used safely in renal

failure?

A. Ciprofloxacin

B. Ofloxacin

C. Lomefloxacin

D. Pefloxacin

Key: D

630. Drug-induced colitis is most frequently associated

with:

A. Neomycin

B. Vancomycin

C. Clindamycin

Page 54: 7. Sodium valproate

D. Chloramphenicol

Key: C

631. A 26 year old patient presents with suspected

pneumococcal meningitis. CSF culture is sent for

antibiotic sensitivity. Which empirical antibiotic

should be given till culture sensitivity result come?

A. Penicillin G

B. Ceftriaxone + metronidazole

C. Doxycycline

D. Cefotaxime + vancomycin

Key: D

632. A patient develops an infection of methicillin resistant

Staphylococcus aureus. All of the following can be

used to treat this infection except:

A. Cotrimoxazole

B. Cefaclor

C. Ciprofloxacin

D. Vancomycin

Key: B

633. Drug of choice for syphilis in a pregnant lady is:

A. Penicillin

B. Azithromycin

C. Tetracycline

D. Ceftriaxone

Key: A

634. A patient diagnosed as having ventilator associated

pneumonia, is on treatment with ceftriaxone and

amikacin. Culture and sensitivity turned out to be

positive for ESBL producing Klebsiella infection. The

most appropriate next action should be:

A. Continue same antibiotic but at higher dose

B. Replace ceftazidime for ceftriaxone

C. Remove amikacin and add quinolone

D. Change over to imipenem.

Key: D

635. Which of the following antibiotic is used in the

treatment of Clostridium difficile associated diarrhea?

A. Ciprofloxacin

B. Metronidazole

C. Piperacillin

D. Clindamycin

Key: B

636. Which of the following drug should not be used to

treat

Klebsiella infection?

A. Ampicillin

B. Amikacin

C. Imipenem

D. Tigecycline

Key: A

637. Drug of choice for chlamydial infection in pregnancy

is:

A. Doxycycline

B. Tetracycline

C. Erythromycin

D. Ciprofloxacin

Key: C

638. Which of the following drugs is effective against

Pseudomonas infection?

A. Ampicillin

B. Ceftriaxone

C. Colistin

D. Cefixime

Key: C

639. Drug of choice for treatment of infection caused by

methicillin resistant Staphylococcus aureus is:

A. Macrolides

B. Third generation cephalosporins

C. Carbapenems

D. Glycopeptides

Key: D

640. Methicillin resistant Staphylococcus aureus is not

expected to respond to

A. Aminoglycoside

B. Lincosamide

C. Oxazolidinone

D. Carbapenem

Key: D

641. Fixed drug eruptions can be seen more frequently

with:

A. Penicillin

B. Sulfonamide

C. Cetrizine

D. Roxithromycin

Key: B

642. Which of the following penicillins is effective against

pseudomonas?

A. Piperacillin

Page 55: 7. Sodium valproate

B. Amoxycillin

C. Ampicillin

D. Oxacillin

Key: A

643. Drug of choice for prophylaxis of diphtheria is:

A. Tetracycline

B. Erythromycin

C. Ciprofloxacin

D. Amikacin

Key: B

644. Which of the following is least nephrotoxic?

A. Streptomycin

B. Gentamicin

C. Polymixin B

D. Doxycycline

Key: D

645. Drug commonly used against enteric fever are all

EXCEPT:

A. Amikacin

B. Ciprofloxacin

C. Ceftriaxone

D. Azithromycin

Key: A

646. Which of the following is an antipseudomonal

antibiotic?

A. Ciprofloxacin

B. Vancomycin

C. Cefaclor

D. Tetracycline

Key: A

647. Which of the following drugs is not used for MRSA?

A. Cefaclor

B. Cotrimoxazole

C. Ciprofloxacin

D. Vancomycin

Key: A

648. Which of the following medications is contraindicated

in patients with allergy to sulfonamides

A. Levobunolol

B. Bimatoprost

C. Brinzolamide

D. Brimonidine

Key: C

649. A diabetic patient develops cellulitis due to

Staphylococcus aureus that was found to be

methicillin resistant on the antibiotic sensitivity

testing. All of the following antibiotics will be

appropriate EXCEPT:

A. Vancomycin

B. Imipenem

C. Teicoplanin

D. Linezolid

Key: B

650. All of the following drugs can cause renal failure

EXCEPT:

A. Cephaloridine

B. Amphotericin B

C. Cefoperazone

D. Gentamicin

Key: C

651. The treatment of contacts of meningococcal

meningitis is by:

A. Rifampicin

B. Erythromycin

C. Penicillin

D. Cephalosporin

Key: A

652. Which of the following is not an anti-pseudomonal

agent?

A. Vancomycin

B. Ticarcillin

C. Ceftazidime

D. Tobramycin

Key: A

653. A patient has hepatic encephalopathy. The drug of

choice for gut sterilization in this patient is:

A. Neomycin

B. Netilmicin

C. Bleomycin

D. None of the above

Key: A

654. Which of the following drug causes pseudotumor

cerebri?

A. Sparfloxacin

B. Tetracycline

C. Gentamicin

D. Clofazimine

Page 56: 7. Sodium valproate

Key: B

655. In a chronic alcoholic patient all of the following

drugs should be avoided EXCEPT:

A. Cefamandole

B. Metronidazole

C. Chlorpropamide

D. Beclomethasone

Key: D

656. Drugs that can be used for outpatient treatment of

community acquired pneumonia are

A. Ceftriaxone

B. Imipenem

C. Azithromycin

D. Doxycycline

Key: C & D

657. Drugs that should be avoided in a patient with seizure

disorder are:

A. Ciprofloxacin

B. Cycloserine

C. Glucocorticoids

D. All

Key: D

658. Treatment of choice for Salmonella typhi is:

A. Cephalexin

B. Gentamicin

C. Streptomycin

D. Ciprofloxacin

Key: D

659. Drug of choice for methicillin resistant staphy-

lococcus aureus (MRSA) is

A. Amoxicillin-Clavulanate

B. Vancomycin

C. Clindamycin

D. Erythromycin

Key: B

660. Which of the following drugs is most likely to cause

loss of equilibrium and auditory damage?

A. Amikacin

B. Ethambutol

C. Isoniazid

D. Rifabutin

Key: A

661. Which of the following drugs is LEAST likely to

require dosage reduction in renal dysfunction?

A. Amikacin

B. Ciprofloxacin

C. Clindamycin

D. Vancomycin

Key: C

662. Antimicrobials effective against anaerobic bacteria

include the following EXCEPT:

A. Tobramycin

B. Clindamycin

C. Chloramphenicol

D. Metronidazole

Key: A

663. Select the antimicrobial agent that can be used to treat

both methicillin resistant and vancomycin resistant

Staphylococcus aureus infections:

A. Clarithromycin

B. Clindamycin

C. Linezolid

D. Lincomycin

Key: C

664. For a 23 old pregnant female having severe

sensitivity to amoxicillin, drug used to treat

gonorrhoea in a single dose should be

A. Ceftriaxone

B. Tetracycline

C. Ciprofloxacin

D. Spectinomycin

Key: D

665. The drug that should be used for prophylaxis of close

contacts of a patient suffering from meningococcal

meningitis is:

A. Rifampicin

B. Dapsone

C. Erythromycin

D. Amikacin

Key: A

666. A 14 year old boy present with headache, fever and

cough for 2 days. Sputum is scant and non-purulent

and gram stain reveals many white cells but no

organisms.

The treatment should be initiated with:

A. Cefazolin

B. Erythromycin

C. Amikacin

Page 57: 7. Sodium valproate

D. Trovafloxacin

Key: B

667. Drugs that can be used to treat infections caused by

Bacteroides fragilis are all EXCEPT:

A. Metronidazole

B. Trovafloxacin

C. Vancomycin

D. Amikacin

Key: D

668. A patient needs antibiotic treatment for artificial

valve, culture-positive infective enterococcal

endocarditis. His medical history includes severe

anaphylactic reaction to penicillin G during the past

year. The best approach would be treatment with:

A. Amoxicillin-clavulanic acid

B. Aztreonam

C. Cefazolin plus gentamicin

D. Vancomycin

Key: D

669. In a patient with culture-positive enterococcal

endocarditis who has failed to respond to vancomycin

because of resistance, the treatment most likely to be

effective is:

A. Clarithromycin

B. Linezolid

C. Minocycline

D. Ticarcillin

Key: B

670. Select the drug which is used to treat antibiotic

associated pseudomembranous enterocolitis and is a

component of anti H. pylori triple drug regimen:

A. Amoxicillin

B. Vancomycin

C. Metronidazole

D. Clotrimazole

Key: C

671. This drug depolarizes cell membranes of aerobic gram

positive bacteria. It is effective against vancomycin

resistant enterococcal infections. It may cause

myopathy especially in patients taking statins. It is:

A. Teicoplanin

B. Daptomycin

C. Linezolid

D. Streptogramin

Key: B

672. A patient of abdominal sepsis was started on

empirical treatment with intravenous ampicillin and

gentamicin.

Regarding the treatment of this patient, which

statement is most accurate?

A. Empirical treatment of abdominal sepsis should

always include a third generation cephalosporin

B. A drug active against anaerobe should be

included in the antibiotic regimen

C. Combination of ampicillin and gentamicin

provides good coverage for all likely pathogens

D. If the patient is severely allergic to ampicillin,

then ceftriaxone should be used

Key: B

673. Red man syndrome occurs with

A. Clindamycin

B. Teicoplanin

C. Vancomycin

D. Polymyxin

Key: C

674. Which of the following antimicrobials needs dose

reduction even in mild renal failure?

A. Ciprofloxacin

B. Carbenicillin

C. Cefotaxime

D. Ethambutol

Key: D

675. Which of the following drug can cause cartilage

damage in children?

A. Cotrimoxazole

B. Penicillin

C. Ciprofloxacin

D. Metronidazole

Key: C

676. Gray baby syndrome is caused by:

A. Chlorpromazine

B. Chloramphenicol

C. Phenytoin

D. Gentamycin

Key: B

677. Macrocytic anaemia is caused by all EXCEPT:

A. Pyrimethamine

B. Methotrexate

C. Pentamidine

D. Trimethoprim

Page 58: 7. Sodium valproate

Key: C

678. Which of the following drugs is most commonly

associated with Clostridium difficile colitis?

A. Vancomycin

B. Metronidazole

C. Clindamycin

D. Erythromycin

Key: C

679. Which of the following is not nephrotoxic?

A. Tobramycin

B. Kanamycin

C. Ampicillin

D. Amphotericin B

Key: C

680. Dose of which of the following antibiotic does not

require alteration in renal failure?

A. Vancomycin

B. Ethambutol

C. Erythromycin

D. Metronidazole

Key: C

681. Drug of choice for sore throat caused by Group A

beta hemolytic streptococcus is:

A. Erythromycin

B. Penicillin

C. Ceftriaxone

D. Sulfonamides

Key: B

682. Nephrotoxicity is seen with:

A. Doxycycline

B. Aminoglycosides

C. Erythromycin

D. Rifampicin

Key: B

683. Drug which should not be given in renal disease is:

A. Gentamicin

B. Nitroprusside

C. Doxycycline

D. Ceftriaxone

Key: A

684. Drug causing megaloblastic anemia is:

A. INH

B. Chloramphenicol

C. Pyrimethamine

D. Methyldopa

Key: C

685. Jarisch-Herxheimer reaction is seen in syphilis with:

A. Tetracyclines

B. Penicillins

C. Co-trimoxazole

D. Sulfonamides

Key: B

686. Which of the following drug is not used against

Pseudomonas?

A. Piperacillin

B. Carbenicillin

C. Ticarcillin

D. Oxacillin

Key: D

687. Red cell aplasia can be caused by:

A. Aminoglycosides

B. Chloramphenicol

C. Penicillins

D. Ciprofloxacin

Key: B

688. Which of the following drug is safe during

pregnancy?

A. Aminoglycoside

B. Ampicillin

C. Chloramphenicol

D. Cotrimoxazole

Key: B

689. Which one of the following is used in the prophylaxis

of streptococcal sore throat?

A. Phenoxy methyl penicillin

B. Inj. Benzathine Penicillin

C. Crystalline penicillin

D. Both A and B are true

Key: B

690. Drug of choice for plague is:

A. Erythromycin

B. Tetracyclines

C. Ampicillin

D. Cotrimoxazole

Key: B

Page 59: 7. Sodium valproate

691. Which one of the following is primarily

bacteriostatic?

A. Ciprofloxacin

B. Chloramphenicol

C. Vancomycin

D. Rifampicin

Key: B

692. Drug of choice for prophylaxis of meningococcal

meningitis is:

A. Penicillin

B. Erythromycin

C. Septran

D. Rifampicin

Key: D

693. Drug with high degree of photosensitivity is:

A. Tetracycline

B. Doxycycline

C. Minocycline

D. Methacycline

Key: B

694. Drug used for treatment of methicillin resistant

staphylococcus aureus is:

A. Teicoplanin

B. Vancomycin

C. Both

D. None

Key: C

695. Drug of choice in pertussis is

A. Penicillin

B. Doxycycline

C. Erythromycin

D. Ciprofloxacin

Key: C

696. Drug effective against pseudomonas is:

A. Penicillin G

B. Gentamicin

C. Tetracycline

D. Chloramphenicol

Key: B

697. Treatment of choice for chancroid is:

A. Penicillin

B. Chloramphenicol

C. Tetracyclines

D. Erythromycin

Key: D

698. Pseudomembranous colitis is associated mostly with

which drug?

A. Erythromycin

B. Ampicillin

C. Vancomycin

D. Ciprofloxacin

Key: B

699. Drug of choice for primary syphilis is:

A. Ampicillin

B. Benzathine penicillin

C. Erythromycin

D. Tetracycline

Key: B

700. Drug of choice for syphilis during pregnancy is:

A. Ampicillin

B. Erythromycin

C. Benzathine penicillin

D. Tetracyclines

Key: C

701. Drug that is NOT contraindicated in G-6 PD

deficiency is:

A. Primaquine

B. Nitrofurantoin

C. Dapsone

D. INH

Key: D

702. Which of the following drug is contraindicated in

pregnancy?

A. Chloroquine

B. Erythromycin

C. Ampicillin

D. Primaquine

Key: D

703. Absorption of which of the following drug increases

with food intake?

A. Tetracycline

B. Diazepam

C. Griseofulvin

D. Ampicilin

Key: C

Page 60: 7. Sodium valproate

704. Which of the following prokinetic drug acts on

motilin receptors?

A. Erythromycin

B. Metoclopramide

C. Loxiglumide

D. Cisapride

Key: A

705. The antibiotic which can be given safely in a pregnant

women is

A. Ciprofloxacin

B. Cefuroxime

C. Metronidazole

D. Chloramphenicol

Key: B

706. Drug of choice for Mycoplasma pneumoniae infection

is:

A. Gentamicin

B. Amoxyclillin

C. Azithromycin

D. Cefotaxime

Key: C

707. Drug of choice for acute (pneumococcal) lobar

pneumonia is:

A. Amoxicillin clavulanic acid combination

B. Ciprofloxacin

C. Co-trimoxazole

D. Crystalline penicillin (Pen. G)

Key: D

708. Drug of choice for acute meningococcal pyogenic

meningitis is:

A. Crystalline penicillin (Pen. G)

B. Sulphonamides

C. Chloramphenicol

D. Amoxycillin

Key: A

709. Which of the following is not given in myasthenia

gravis:

A. Clofibrate

B. Polymixin B

C. Penicillin

D. All

Key: B

710. Which of following drug’s absorption is increased in

gastric achlorhydria?

A. Ketoconazole

B. Penicillin G

C. Chloramphenicol

D. Ciprofloxacin

Key: B

711. Which does not cause pseudomembranous

enterocolitis?

A. Vancomycin

B. Levofloxacin

C. Clindamycin

D. Ceftazidime

Key: A

712. Single dose treatment for chlamydia is?

A. Doxycycline

B. Tetracycline

C. Azithromycin

D. Erythromycin

Key: C

713. Mechanism of action of quinolones is by?

A. Inhibiting DHFRase

B. Inhibiting DNA gyrase

C. Inhibiting protein synthesis

D. Inhibiting cell wall synthesis

Key: B

714. Longest acting sulphonamide is –

A. Sulfadiazine

B. Sulfadoxine

C. Sulfamethoxazole

D. Sulfamethiazole

Key: B

715. Widest spectrum aminoglycoside is –

A. Streptomycin

B. Amikacin

C. Framycetin

D. Netilmicin

Key: B

716. Penicillinase resistant penicillins include all of the

following drugs except:

A. Flucloxacillin

B. Nafcillin

C. Oxacillin

D. Carbenicillin

Page 61: 7. Sodium valproate

Key: D

717. Ampicillin is used in:

A. Listeria

B. Pertussis

C. Atypical pneumonia

D. Gonococci

Key: A

718. Pseudomonas is resistant to:

A. Vancomycin

B. Aztreonam

C. Ciprofloxacin

D. Polymyxin B

Key: A

719. Which of the following is contra-indicated in

pregnancy?

(a) Tetracycline

(b) Erythromycin

(c) Ampicillin

(d) Chloroquine

Key: A

720. Red man syndrome is due to:

A. Vancomycin

B. Polymyxin

C. Rifampicin

D. Teicoplanin

Key: A

721. Drug of choice for syphilis is –

A. Penicillin

B. Rifampicin

C. Tetracycline

D. Erythromycin

Key: A

722. Bleeding is a risk with the use of:

A. Cefaloridine

B. Cefazolin

C. Moxalactum

D. Ceftazidime

Key: C

723. Sulphonamides act by:

A. Competitive inhibition

B. Non-competitive inhibition

C. Allosteric inhibition

D. None of these

Key: A

724. All are true about ciprofloxacin except –

A. Contra-indicated in pregnancy

B. DNA inhibition

C. Most potent first generation fluoroquinolone

D. More active at acidic pH

Key: D

725. Drug inhibiting bacterial protein synthesis are all

except:

A. Aminoglycosides

B. Chloramphenicol

C. Clindamycin

D. Sulfonamides

Key: D

726. Actinomycete is the source of which of the following

anti microbials?

A. Tetracycline

B. Polyene

C. Aztreonam

D. Colistin

Key: A

727. Drug that can cause hypertrophic pyloric stenosis is:

A. Tertacycline

B. Erythromycin

C. Ampicillin

D. Rifampicin

Key: B

728. Drug that inhibits cell wall synthesis is?

A. Tetracyclines

B. Penicillins

C. Aminoglycosides

D. Chloramphenicol

Key: B

729. Which of the following tetracycline can be used in

renal failure without dose adjustment?

A. Oxytetracycline

B. Doxycycline

C. Demeclocycline

D. Tetracycline

Key: B

730. Drug used in prophylaxis of meningococcal

meningitis is:

Page 62: 7. Sodium valproate

A. Ciprofloxacin

B. Rifampicin

C. Penicillin

D. Gentamicin

Key: B

731. Mechanism of action of quinolones is:

A. DNA gyrase inhibitors

B. Bind to 30 S unit of ribosomes and inhibit protein

synthesis

C. Bind to bacterial cell membrane

D. Bind to tetra hydro folate reductase

Key: A

732. Drug of choice in pregnant women with secondary

syphills is:

A. Doxycycline

B. Benzathine Penicillin

C. Ceftriaxone

D. Cotrimoxazole

Key: B

733. True about imipenem is:

A. It is narrow spectrum antibiotic

B. It is easily broken by beta lactamases

C. It should be used with cilastatin

D. It is used with sulbactam

Key: C

734. Which among the following is not a beta lactamase

resistant Penicillin?

A. Methicillin

B. Carbenicillin

C. Nafcillin

D. Oxacillin

Key: B

735. Drug of choice for Treponema pallidum is:

A. Penicillin G

B. Tetracycline

C. Azithromycin

D. Doxycycline

Key: A

736. All are third generation Cephalosporins except:

A. Ceftriaxone

B. Ceftazidime

C. Cefuroxime

D. Cefoperazone

Key: C

737. Treatment for clostridial myonecrosis is:

A. Amikacin

B. Penicillin

C. Ampicillin

D. Gentamicin

Key: B

738. All of the following have beta lactam ring except:

A. Penicillin

B. Linezolid

C. Cefotaxime

D. Imipenem

Key: B

739. Drug of choice for treatment of infections caused by

MRSA is:

A. Metronidazole

B. Vancomycin

C. Imipenem

D. Clindamycin

Key: B

740. The antibiotic causing pseudomembrane colitis is:

A. Clindamycin

B. Garamycin

C. Erythromycin

D. Vancomycin

Key: A

741. An aminoglycoside that is resistant to majority of

inactivating enzymes is:

A. Gentamicin

B. Amikacin

C. Tobramycin

D. Sisomicin

Key: B

742. All of the following Beta-Lactam antibiotics possess

antipseudomonal action, except:

A. Piperacillin

B. Ceftriaxone

C. Ceftazidime

D. Cefoperazone

Key: B

743. All of the following are common antimicrobial agents

used in treatment of typhoid fever except:

A. Ceftriaxone

Page 63: 7. Sodium valproate

B. Quinolones

C. Clindamycin

D. Azithromycin

Key: C

744. The drug of choice in lymphogranuloma venereum is:

(a) Penicillin

(b) Ciprofloxacin

(c) Tetracycline

(d) Erythromycin

Key: C

745. Antacid interfere with absorption of all of the

following except:

A. Ketoconazole

B. Azithromycin

C. Oxytetracycline

D. Ofloxacin

Key: B

746. All of the following are beta lactamase inhibitors

except:

A. Clavulanic acid

B. Sulbactam

C. Tazobactam

D. Aztreonam

Key: D

747. Which one of the following is true about the beta

lactam antibiotics:

A. All are based on the 6-amino-penicillanic acid

structure

B. Include amikacin

C. Are safe in pregnancy

D. Are uniformly ineffective against pseudomonas

aeruginosa

Key: C

748. Antibiotic which is effective as a single dose therapy

for trachoma is:

A. Doxycycline

B. Clarithromycin

C. Azithromycin

D. Erythromycin

Key: C

749. The prophylactic antibiotic indicated to prevent

infection in lymphoedema is:

A. Vancomycin

B. Penicillin

C. Amikacin

D. Quinolones

Key: A

750. Drug of choice in dermatitis herpetiformis is:

A. Corticosteroids

B. Colchicine

C. Dapsone

D. Chloroquine

Key: C

751. The preferred treatment option for primary syphilis is:

A. Injection Benzathine pencillin 2.4 million units

IM single dose

B. Injection Benzathine penicillin 2.4 million units

IM once a week for 3 weeks

C. Cap. Doxycycline 100 mg orally twice a day for

2 weeks

D. Tab. Azithromycin 2 gm single dose

Key: A

752. A diabetic patient developed cellulites due to S.

aureus, which was found to be methicillin resistant on

antibiotic sensitivity testing. All of the following

antibiotics would be appropriate except:

A. Linezolid

B. Vancomycin

C. Teicoplanin

D. Imipenem

Key: D

753. Which of the following antibacterial causes both

ototoxicity and nephrotoxicity:

(a) Methicillin

(b) Vancomycin

(c) Clindamycin

(d) Azithromycin

Key: B

754. Oral vancomycin can be used for treatment of:

A. Hepatic encephalopathy

B. Pseudomembranous colitis

C. Staphylococcal food poisoning

D. None of the above

Key: B

755. Which of the following is responsible for antibiotic

associated colitis:

A. Clostridium botulinum

B. Clostridium perfringens

Page 64: 7. Sodium valproate

C. Clostridium difficile

D. Actinomyces species

Key: C

756. All of the following drugs can cause cholestatic

jaundice except:

A. Ethambutol

B. Chlorpromazine

C. Erythromycin estolate

D. Estrogens

Key: A

757. Which of the following antibiotics class is not safe in

pregnancy:

(a) Quinolones

(b) Cephalosporins

(c) Penicillins

(d) Macrolides

Key: A

758. Which of the following drugs is avoided in a patient

with high serum creatinine (> 3 mg/dl)?

A. Gentamicin

B. Azithromycin

C. Moxifloxacin

D. Amlodipine

Key: A

759. Which of the following is used in the prophylactic

treatment of rheumatic heart disease:

A. Ampicillin

B. Penicillin-G

C. Bezathine penicillin

D. Phenoxy-methyl penicillin

Key: C

760. Which amongst the following antimicrobials exhibits

a long post antibiotic effect:

A. Quinolones

B. Macrolides

C. Beta-lactams

D. Oxazolidinones

Key: A

761. Which of the following is used as prophylaxis for

meningococcal meningitis?

A. Gentamicin

B. Erythromycin

C. Rifampicin

D. Chloramphenicol

Key: C

762. Which of the following drugs is most effective against

an organism producing aminoglycoside inactivating

enzymes?

A. Tobramycin

B. Gentamicin

C. Amikacin

D. Streptomycin

Key: C

763. The persistent suppression of bacterial growth that

may occur after limited exposure to some

antimicrobial drug is called?

A. Time dependent killing

B. Concentration dependent killing

C. Post antibiotic effect

D. Sequential blockade

Key: C

764. Prophylactic antibiotics to prevent surgical site

infection are best administered:

A. After commencement of surgery

B. 30 minutes before incision

C. At the end of surgery

D. With pre medication

Key: B

765. Which of the following antimicrobial agents act solely

on the gram positive bacterial cell wall:

A. Ciprofloxacin

B. Gentamicin

C. Tetracycline

D. Vancomycin

Key: D

766. Sulfonamides inhibit bacterial synthesis of folic acid

by:

A. Uncompetitive inhibition

B. Allosteric inhibition

C. Competitive inhibition

D. Noncompetitive inhibition

Key: C

767. Which of the following drugs is an anti-pseudomonal

penicillin?

A. Cephalexin

B. Dicloxacillin

C. Piperacillin

D. Cloxacillin

Page 65: 7. Sodium valproate

Key: C

768. Which can be given safely in renal failure?

A. Tetracycline

B. Gentamicin

C. Amphotericin B

D. Doxycycline

Key: D

769. Drug of choice in pneumonia caused by P carnii is:

A. Penicillin

B. Cotrimoxazole

C. Kanamycin

D. Levofloxacin

Key: B

770. Which of the following drugs is a 4th generation

cephalosporin?

A. Cefixime

B. Ceftriaxone

C. Cefpirome

D. Cefazolin

Key: C

771. Which of the following drugs should not be given in

renal failure?

A. Clindamycin

B. Methicillin

C. Amoxicillin

D. Rifampicin

Key: B

772. Beta lactam antibiotics are all except:

A. Amoxicillin

B. Aztreonam

C. Ceftriaxone

D. Vancomycin

Key

773. All of the following drugs are administered orally

except:

A. Ciprofloxacin

B. Cotrimoxazole

C. Gentamicin

D. Amoxicillin

Key: C

774. Aplastic anemia is the adverse effect of:

A. Chloramphenicol

B. Ciprofloxacin

C. Penicillin

D. Gentamicin

Key: A

775. Highest photosensitivity is seen with:

A. Pefloxacin

B. Gatifloxacin

C. Levofloxacin

D. Sparfloxacin

Key: D

776. Drug most commonly implicated in causing pseudo

membranous colitis is:

A. Clindamycin

B. Streptomycin

C. Amoxicillin

D. Metronidazole

Key: A

777. Which of the following is a side effect of

streptomycin?

A. Phototoxicity

B. Hepatotoxicity

C. Ototoxicity

D. All of the above

Key: C

778. Not true about vancomycin is:

A. 95% oral bioavailability

B. Inhibits cells wall synthesis

C. Can be used parenterally as well as orally

D. Indicated for MRSA infections

Key: A

779. Dosage of topical tobramycin eye drops:

A. 1 mg/ml

B. 2 mg/ml

C. 3 mg/ml

D. 4 mg/ml

Key: C

780. Route of administration of vancomycin in

pseudomembranous colitis is:

A. i.m.

B. oral

C. i.v.

D. s.c.

Key: B

Page 66: 7. Sodium valproate

781. Dosage of benzyl penicillin in treatment of primary

syphilis is:

A. 1.2 MU single i.m.

B. 1.2 MU single i.v.

C. 2.4 MU single i.m.

D. 4.8 MU single i.m.

Key: C

782. Treatment of non-specific urethritis is:

A. Erythromycin

B. Sulphonamides

C. Tetracycline

D. Ampicillin

Key: C

783. Hemolysis in G-6 deficiency is precipitated by all of

the following except:

A. Dapsone

B. Cotrimoxazole

C. Quinine

D. Penicillin

Key: D

784. In treatment of Pseudomonas infections, carbenicillin

is frequently combined with:

A. Penicillin

B. Gentamicin

C. Ciprofloxacin

D. Amoxycillin

Key: B

785. All of the following are true regarding cephalosporins

except:

A. Bactericidal agents

B. Active against only gram negative bacteria

C. IIIrd Generation are resistant to β-lactamases

from gram negative bacteria

D. ceftriaxone is administered parenterally

Key: B

786. Which antimalarial drug can be safely administered in

baby with glucose-6-phosphagte dehydrogenase

deficiency?

A. Chloroquinine

B. Quinine

C. Mefloquine

D. Primaquine

Key: C

787. Drug used in the treatment of resistant gonorrhoea is:

A. Penicillin

B. Cotrimoxazone

C. Spectinomycin

D. Erythromycin

Key: C

788. Penicillinase resistant penicillin is:

A. Methicillin

B. Ampicillin

C. Carbenecillin

D. Ticarcillin

Key: A

789. Which of the following drugs is most likely to be

effective against multidrug-resistant strains of M.

tuberculosis, including those resistant to

streptomycin?

A. Amikacin

B. Clarithromycin

C. Gentamicin

D. Spectinomycin

Key: A

790. Slow acetylators of isoniazid are more prone to

develop:

A. Failure of therapy

B. Peripheral neuropathy

C. Hepatotoxicity

D. Allergic reactions

Key: B

791. Prolonged treatment with INH leads to deficiency of?

A. Pyridoxine

B. Thiamine

C. Pantothenic acid

D. Niacin

Key: A

792. Pseudojaundice is an adverse effect of:

A. Phenytoin

B. Rifabutin

C. Omeprazole

D. Chlorpromazine

Key: B

793. Commonest side effect of Dapsone is:

A. Hemolytic anemia

B. Thrombocytopenia

C. Cyanosis

Page 67: 7. Sodium valproate

D. Bone marrow depression

Key: A

794. Cross resistance of isoniazid is seen with:

A. Rifampicin

B. Ethionamide

C. Cycloserine

D. Ethambutol

Key: B

795. Ethambutol causes:

A. Retrobulbar neuritis

B. Deafness

C. Red urine

D. Peripheral neuritis

Key: A

796. Which of the following antitubercular drugs is safe in

hepatitis?

A. Isoniazid

B. Rifampicin

C. Pyrazinamide

D. Ethambutol

Key: D

797. A 30 year old pregnant woman develops tuberculosis.

Which of the following antitubercular drugs should

not be used?

A. INH

B. Rifampicin

C. Streptomycin

D. Ethambutol

Key: C

798. Patients suffering from multidrug resistant

tuberculosis can be treated with all the following

drugs EXCEPT:

A. Tobramycin

B. Amikacin

C. Ciprofloxacin

D. Clarithromycin

Key: A

799. Which anti-tubercular drug is implicated in the

causation of transient memory loss?

A. Ethionamide

B. Isoniazid

C. Ethambutol

D. Pyrazinamide

Key: B

800. All of the following are true about the therapy of

tuberculosis EXCEPT:

A. Flu like syndrome is usually seen with rifampicin

being taken on daily basis

B. Ethambutol accumulates in renal failure

C. Hyperuricemia is a recognized side effect of

pyrazinamide

D. Red green color impairment is an early sign of

ethambutol induced optic neuritis

Key: A

801. In leprosy, the best bactericidal agent is:

A. Clofazimine

B. Dapsone

C. Rifampicin

D. Ethionamide

Key: C

802. A patient suffering from AIDS is on zidovudine,

lamivudine and indinavir therapy. He develops

pulmonary tuberculosis for which treatment is started.

Which of the following should be avoided in him?

A. INH

B. Ethambutol

C. Pyrazinamide

D. Rifampicin

Key: D

803. Most effective drug against extracellular

mycobacteria is:

A. Isoniazid

B. Rifampicin

C. Pyrazinamide

D. Ethambutol

Key: B

804. Hypothyroidism is caused by which of the following

anti-tubercular drug?

A. Streptomycin

B. Ethionamide

C. Thioacetazone

D. Ethambutol

Key: B

805. ATT causing orange coloured urine is:

A. Rifampicin

B. Isoniazid

C. Streptomycin

D. Pyrazinamide

Page 68: 7. Sodium valproate

Key: A

806. Which of the following antitubercular drug is not

hepatotoxic:

A. Isoniazid

B. Rifampicin

C. Ethionamide

D. Streptomycin

Key: D

807. ATT most commonly implicated in causing peripheral

neuropathy is:

A. Rifampicin

B. Pyrazinamide

C. INH

D. Ethambutol

Key: C

808. Arthralgia is commonly caused by which ATT drug?

A. INH

B. Rifampicin

C. Pyrazinamide

D. Ethambutol

Key: C

809. Which of the following antitubercular drugs can be

safely used in severe renal failure:

A. Streptomycin

B. Ethambutol

C. Capreomycin

D. Rifampicin

Key: D

810. A patient of multidrug resistant tuberculosis is on

antitubercular drugs. After a few months he develops

inability to distinguish between red and green color.

Most likely drug causing these symptoms is:

A. Rifampicin

B. Ethambutol

C. Cycloserine

D. Ethionamide

Key: B

811. Which of the following drugs is useful in the

treatment of infection by Mycobacterium avium

complex?

A. Isoniazid

B. Clarithromycin

C. Cycloserine

D. Rifampicin

Key: B

812. Which of the following antitubercular drugs can cause

psychosis?

A. Ofloxacin

B. Cycloserine

C. Capreomycin

D. Rifampicin

Key: B

813. INH induced peripheral neuropathy results from

deficiency of vitamin:

A. B1

B. B2

C. B6

D. B12

Key: C

814. Antitubercular drug which DOES NOT cross blood

brain barrier is:

A. Streptomycin

B. INH

C. Rifampicin

D. Pyrazinamide

Key: A

815. Which of the following antitubercular drugs can be

used in patients with hepatic dysfunction?

A. Streptomycin

B. INH

C. Pyrazinamide

D. Rifampicin

Key: A

816. INH can be used safely in the presence of:

A. Jaundice

B. Chronic renal failure

C. Epilepsy

D. Coronary artery disease

Key: D

817. Which of the following antitubercular drug is

preferred in severe liver disease?

A. Streptomycin + Isoniazid

B. Streptomycin + Ethambutol

C. Isoniazid + Rifampicin

D. Rifamicin + Ethambutol

Key: B

Page 69: 7. Sodium valproate

818. Which of the following ATT has maximum CSF

penetration?

A. Streptomycin

B. INH

C. Rifampicin

D. Ethambutol

Key: B

819. Common dose dependant side effects of ethambutol

is:

A. Red-urine

B. Optic neuritis

C. Nephropathy

D. Peripheral neuropathy

Key: B

820. Most common drug used in Leprosy is:

A. Dapsone

B. Clofazimine

C. Ethionamide

D. Ofloxacin

Key: A

821. The bacterial drug resistance in tuberculosis results

from:

A. Transduction

B. Transformation

C. Plasmid mediated

D. Mutation

Key: D

822. INH and pyridoxine are given together in

antituberculous chemotherapy:

A. To prevent peripheral neuritis

B. To prevent emergence of INH resistance

C. As a nutrient supplement

D. As a synergistic combination

Key: A

823. Dapsone is used in all except:

A. Dermatitis herpetiformis

B. Leprosy

C. Pneumocystis jiroveci pneumonia

D. Tuberculosis

Key: D

824. In Lepra reaction, the drug useful is:

A. Pencillins

B. Clofazimine

C. Dapsone

D. Rifampicin

Key: B

825. Antitubercular drug that can cause hyperuricemia is:

A. Rifampicin

B. INH

C. Pyrazinamide

D. Streptomycin

Key: C

826. Treatment of lepromatous leprosy is:

A. Rifampicin + Dapsone

B. Rifampicin + Clofazamine

C. Rifampicin + Dapsone + Clofazamine

D. Rifampicin + Ofloxacin + Minocycline

Key: C

827. Ethambutol should be used very cautiously in

childhood tuberculosis due to which of its side effect?

A. Ocular toxicity

B. Renal damage

C. Hepatotoxicity

D. Neurotoxicity

Key: A

828. Which of the following drugs can produce dramatic

improvement in patients with Type II lepra reaction?

A. Thalidomide

B. Steroids

C. Dapsone

D. Clofazamine

Key: A

829. Mechanism of action of rifampicin is?

A. Inhibition of mycolic acid synthesis

B. DNA dependent RNA polymerase inhibition

C. Protein synthesis inhibition

D. Inhitits synthesis of arabinogalactone

Key: B

830. Drug that crosses placenta is:

A. Isoniazid

B. Rifampicin

C. Pyrazinamide

D. All

Key: D

831. Treatment of Mycobacteria avium complex include all

except:

Page 70: 7. Sodium valproate

A. Ciprofloxacin

B. Clarithromycin

C. Rifabutin

D. Pyrazinamide

Key: D

832. Which one of the following drugs is not used in the

treatment of mycobacterium avium intercellulare

infection?

A. Clarithromycin

B. Eflornithine

C. Ethambutol

D. Rifabutin

Key: B

833. Most important side effect of ethambutol is:

A. Hepatotoxicity

B. Renal toxicity

C. Peripheral neuropathy

D. Retro bulbar neuritis

Key: D

834. The following drugs are useful in the treatment of

isoniazid poisoning:

A. Pyridoxine

B. Diazepam

C. Bicarbonate

D. All of the above

Key: D

835. A diabetic patient presents with fungal infection of

sinuses and peri-orbital region with significant visual

impairment. The best drug for treatment of this patient

is:

A. Amphotericin B

B. Itraconazole

C. Ketoconazole

D. Broad spectrum antibiotics

Key: A

836. Amphotericin B causes deficiency of?

A. Sodium

B. Calcium

C. Potassium

D. Chloride

Key: C

837. Amphotericin B toxicity can be reduced by?

A. Incorporating it in liposomal complex

B. Combining with fluconazole

C. Combining with flucytosine

D. Injecting the drug with dextrose

Key: A

838. Voriconazole is not effective against:

A. Candida albicans

B. Mucormycosis

C. Candida tropicalis

D. Aspergillosis

Key: B

839. Which drug would treat both dermatophytosis and

candidal infections?

A. Ketoconazole

B. Griseofulvin

C. Nystatin

D. Tolnaftate

Key: A

840. Which of the following is caused by amphotericin B?

A. Hypokalemia

B. Hyperkalemia

C. Hypermagnesemia

D. Hyponatremia

Key: A

841. The antimicrobial agent which inhibits the ergosterol

biosynthesis is:

A. Ketoconazole

B. Amphotericin B

C. 5-Flucytosine

D. Griseofulvin

Key: A

842. Which of the following is the treatment of choice for

cryptococcal meningitis?

A. Fluconazole

B. Itraconazole

C. Flucytosine

D. Amphotericin B

Key: D

843. Which of the following statements about fluconazole

is most accurate?

A. It is highly effective in the treatment of

aspergillosis

B. It does not penetrate the blood-brain barrier

C. Its oral bioavailability is less than that of

ketoconazole

D. It inhibits demethylation of lanosterol

Page 71: 7. Sodium valproate

Key: D

844. Which of the following statements about terbinafine is

FALSE?

A. Its activity is restricted to dermatophytes

B. It is effective in onychomycosis

C. It inhibits squalene epoxidase

D. It is used topically only

Key: D

845. Fluconazole differs from ketoconazole in that:

A. It is not active by the oral route

B. It is a more potent inhibitor of drug metabolism

C. It is not effective in cryptococcal meningitis

D. It is unlikely to produce anti-androgenic side

effects

Key: D

846. Fluconazole is more effective than itraconazole in the

following systemic fungal disease:

A. Pulmonary histoplasmosis

B. Cryptococcal meningitis

C. Non-meningeal blastomycosis

D. Disseminated sporotrichosis

Key: B

847. Resistance to acyclovir is most commonly due to

mutation in a viral gene that encodes a protein that:

A. Converts viral RNA into DNA

B. Phosphorylates acyclovir

C. Transports acyclovir into the cell

D. Transports acyclovir out of the cell

Key: B

848. A fungicidal drug that can be used orally for the

treatment of onychomycosis is:

A. Griseofulvin

B. Amphotericin B

C. Clotrimazole

D. Terbinafine

Key: D

849. Dose limiting toxicity of amphotericin B is:

A. Infusion related reaction

B. Renal tubular acidosis

C. Myelosuppression

D. Hypotension

Key: B

850. All of the following antifungal drugs inhibit

ergosterol biosynthesis EXCEPT:

A. Ketoconazole

B. Fluconazole

C. Amphotericin B

D. None of these

Key: C

851. Griseofulvin is not useful in one of the following:

A. Tinea capitis

B. Tinea cruris

C. Tinea versicolor

D. Tinea pedis

Key: C

852. Topically used antifungal agent is:

A. Ketoconazole

B. Clotrimazole

C. Amphotericin B

D. Physostigmine

Key: B

853. Which of the following is a broad spectrum systemic

antifungal agent?

A. Econazole

B. Miconazole

C. Ketoconazole

D. Clotrimazole

Key: C

854. Drug of choice for herpes simplex virus infection is:

A. Acyclovir

B. Zidovudine

C. Indinavir

D. Ribavarin

Key: A

855. Drug of choice for chronic hepatitis –B is

A. Lamivudine

B. IFN-alpha

C. Ribavirin

D. Zidovudine

Key: A

856. Drug that can cause complete histopathological

resolution in patients with hepatitis B is

A. Cyclosporin

B. Ribavarin

C. Entercavir

D. None of the above

Page 72: 7. Sodium valproate

Key: C

857. Mechanism of action of terbinafine is?

A. Binds to ergosterol

B. Prevents formation of purine

C. Inhition of microtubule formation

D. Inhibition of ergosterol synthesis

Key: D

858. Acyclovir is indicated in:

A. Candida

B. Herpes simplex

C. Mycoplasma

D. Pneumocystis

Key: B

859. Most serious adverse effect of ketoconazole is:

A. Adrenal insufficiency

B. Pellagra like skin lesion

C. Liver injury

D. Prostate cancer

Key: A

860. Induction of treatment in serious fungal infections is

mostly done by:

A. IV amphotericin B

B. Ketoconazole

C. 5 – Flucytosine

D. Fluconazole

Key: A

861. Drug of choice for Herpes simplex encephalitis is:

A. 5-Hydroxy deoxyuridine (5-HU)

B. Acyclovir

C. Gancyclovir

D. None of the above

Key: B

862. All of the following are common adverse effects of

HAART therapy except:

A. Steatosis

B. Lipodytrophy

C. Optic neuritis

D. Increased cholesterol

Key: C

863. A person is being treated for Human

Immunodeficiency Virus-1. He developed

hypertriglyceridemia and hypercholesterolem0ia.

Most likely drug implicated for these adverse effects

is:

A. Ritonavir

B. Raltegravir

C. Didanosine

D. Efavirenz

Key: A

864. Which of the following drugs is used to prevent HIV

transmission from an HIV positive pregnant mother to

child?

A. Lamivudine

B. Stavudine

C. Nevirapine

D. Didanosine

Key: C

865. Which of the following drug is a reverse transcriptase

inhibitor?

A. Indinavir

B. Ritonavir

C. Nelfinavir

D. Abacavir

Key: D

866. Maximum risk of pancreatitis is present with:

A. Didanosine

B. Lamivudine

C. Zidovudine

D. Abacavir

Key: D

867. Drug causing maximum peripheral neuropathy is:

A. Zidovudine

B. Lamivudine

C. Stavudine

D. Didanosine

Key: C

868. All of the following drugs are protease inhibitors

EXCEPT:

A. Nelfinavir

B. Saquinavir

C. Abacavir

D. Ritonavir

Key: C

869. Drug causing maximum peripheral neuropathy is:

A. Zidovudine

Page 73: 7. Sodium valproate

B. Lamivudine

C. Stavudine

D. Didanosine

Key: C

870. All of the following drugs are protease inhibitors

EXCEPT:

A. Nelfinavir

B. Saquinavir

C. Abacavir

D. Ritonavir

Key: C

871. Which of the following anti-retroviral drugs does not

cause peripheral neuropathy?

A. Lamivudine

B. Stavudine

C. Didanosine

D. Zalcitabine

Key: A

872. Which of the following is most common side effect of

zidovudine?

A. Anemia

B. Peripheral neuropathy

C. Lactic acidosis

D. All

Key: A

873. Drug of choice in a patient with severe complicated

falciparum malaria is:

A. Chloroquine

B. Quinine

C. Artesunate

D. Artemether

Key: C

874. Which of the following drugs can be used for the

treatment of chloroquine resistant malaria in children?

A. Chloroquine

B. Doxycycline

C. Tetracycline

D. Clindamycin

Key: D

875. Which of the following is best associated with

lumefantrine?

A. Antimycobacterial

B. Antifungal

C. Antimalarial

D. Antiamoebic

Key: C

876. Which of the following antimalarial drugs is a slow

acting schizonticide?

A. Artemether

B. Mefloquine

C. Pyrimethamine

D. Quinine

Key: C

877. Drug of choice for treatment of malaria due to P.

vivax in a 25 year old pregnant female is:

A. Chloroquine

B. Primaquine

C. Sulfadoxine-pyrimethamine

D. Quinine

Key: A

878. Choose the drug whose single oral dose affords

clinical cure of uncomplicated malaria caused by

Chloroquine sensitive/resistant P. falciparum as well

as P. vivax:

A. Quinine

B. Mefloquine

C. Artesunate

D. Proguanil

Key: B

879. Chloroquine is useful in:

A. Discoid lupus erythematosis

B. Rheumatoid arthritis

C. Infectious mononucleosis

D. All of the above

Key: D

880. Drug of choice for treatment of chloroquine resistant

falciparum malaria is:

A. Quinine

B. Chloroquine

C. Pyrimethamine

D. Primaquine

Key: A

881. Tissue schizontocide which prevents relapse of vivax

malaria is:

A. Quinine

B. Primaquine

C. Pyrimethamine

Page 74: 7. Sodium valproate

D. Chloroquine

Key: B

882. Chloroquine is given in high loading dose because of:

A. High volume of distribution

B. Poor GIT absorption

C. High first pass metabolism

D. All

Key: A

883. Which of the following can cause hypoglycemia in a

patient of severe cerebral malaria on treatment?

A. Quinine

B. Chloroquine

C. Halofantrine

D. Mefloquine

Key: A

884. Radical cure is required for malaria caused by:

A. P. falciparum and P. vivax

B. P. falciparum and P. malariae

C. P. vivax and P. malariae

D. P. vivax and P. ovale

Key: D

885. Which antimalarial drug is known to cause

neuropsychiatric adverse reaction?

A. Artesunate

B. Artimisnin

C. Quinine

D. Mefloquine

Key: D

886. Drawback of artesunate is:

A. Poor bioavailability

B. Rapid recrudescence of malaria

C. Hypoglycemia

D. Hemolysis

Key: B

887. Absorption of which of the following anti-malarial

drug increases with food intake?

A. Mefloquine

B. Lumefantrine

C. Chloroquine

D. Amodiaquine

Key: B

888. Patient is being admininstered i.v. quinine following

which he developed restlessness and sweating, the

most likely cause is:

A. Hypoglycaemia

B. Cinchonism

C. Arrhythmias

D. Sweating

Key: A

889. Which one of the following antimalarial drugs is safe

for use in pregnancy?

A. Atovaquone

B. Tetracycline

C. Proguanil

D. Primaquine

Key: C

890. Chloroquine is given as 600 mg loading dose because:

A. It is rapidly absorbed

B. It is rapidly metabolized

C. It has increased tissue binding

D. It is rapidly eliminated

Key: C

891. Drug of choice for chloroquine resistant malaria in

pregnancy is:

A. Quinine

B. Mefloquine

C. Artemisinin

D. Sulphadoxine+ pyrimethamine

Key: A

892. Radical cure of Plasmodium vivax is by:

A. Chloroquine

B. Tetracycline

C. Primaquine

D. Artesunate

Key: C

893. Potassium iodide is useful in the treatment of:

A. Sporotrichosis

B. Impetigo

C. Viral warts

D. Dermatitis herpetiformis

Key: A

894. The drug most likely to be responsible for acute

pancreatitis is:

A. Didanosine

B. Ketoconazole

Page 75: 7. Sodium valproate

C. Saquinavir

D. Zidovudine

Key: A

895. The drug of choice for Kala azar is:

A. Pentamidine

B. Amphotericin B

C. Sodium stibogluconate

D. Ketoconazole

Key: B

896. What is true of ivermectin?

A. It is the most effective drug for strongyloidosis

B. It is the drug of choice for onchocerciasis

C. It can be used to treat scabies

D. All of the above

Key: D

897. Metronidazole is LEAST likely to be effective in the

treatment of:

A. Hepatic amoebiasis

B. Infection caused by Bacteroides fragilis

C. Pseudomembranous colitis

D. Pneumocystosis

Key: D

898. An antihelmenthic drug that is effective against blood

fluke, liver fluke, lung fluke and cysticercus is:

A. Albendazole

B. Praziquantal

C. Ivermectin

D. Thiabendazole

Key: B

899. Drug of choice for treatment of infestation due to

Onchocerca volvolus is:

A. Albendazole

B. Ivermectin

C. Praziquantal

D. Suramin

Key: B

900. Which of the following antimalarial agents is most

commonly associated with acute hemolytic reaction in

patients with glucose-6-phosphate dehydrogenase

deficiency?

A. Chloroquine

B. Clindamycin

C. Mefloquine

D. Primaquine

Key: D

901. a 20 year old female developed antibiotic associated

pseudomembranous colitis caused by Clostridium

difficile. Which of the following drugs is most likely

to be effective in the treatment of this disease?

A. Ampicillin

B. Clindamycin

C. Metronidazole

D. Chloramphenicol

Key: C

902. More than 90% of this drug is excreted in the urine in

intact form. Because its urinary solubility is low,

patients should be well hydrated to prevent

nephrotoxicity. The drug is:

A. Indinavir

B. Zidovudine

C. Acyclovir

D. Amantadine

Key: C

903. A 25 year old male was hospitalized with liver cyst

due to Echinococcus granulosus. He refused to

undergo surgery for removal of cyst. Therefore,

albendazole was used at high dose for 3 months. This

patient should be monitored for the toxicity to:

A. Gonads

B. Kidney

C. Liver

D. Peripheral nerves

Key: C

904. A 26-year-old male, presents to OPD complaining of

hair loss and itching on scalp. Physical examination

reveals moderate patchy hair loss from the central

portion of scalp and the lesions have ring like

configuration with central clearing. Which of the

following drugs can be used to treat this patient’s

condition?

A. Local glucocorticoids

B. Progesterone

C. Finasteride

D. Terbinafine

Key: D

905. Bull’s eye retinopathy is seen in:

A. Chloroquine

B. Methanol

C. Ethambutol

D. Steroids

Page 76: 7. Sodium valproate

Key: A

906. The drug of choice for schistosomiasis is:

A. Albendazole

B. Metronidazole

C. Praziquantel

D. Triclabendazole

Key: C

907. Hepatotoxic drugs are all EXCEPT:

A. Methotrexate

B. Isoniazid

C. Cycloserine

D. Ethionamide

Key: C

908. Drug of choice for neurocysticercosis is:

A. Praziquantel

B. Albendazole

C. Levamisole

D. Piperazine

Key: B

909. Which of the following drug causes flaccid paralysis

of

ascaris?

A. Albendazole

B. Pyrantel pamoate

C. Piperazine

D. Ivermectin

Key: C

910. DEC (Di-ethyl-carbamazine) is used for the treatment

of:

A. Filariasis

B. Dracunculiasis

C. Schistosomiasis

D. Taeniasis

Key: A

911. Which of the following drug is deposited in the

retina?

A. Isoniazid

B. Chloroquine

C. Rifampicin

D. Pyrizinamide

Key: B

912. Round worm infection is best treated with:

A. Metronidazole

B. Mebendazole

C. Albendazol

D. Pyrantel pamoate

Key: C

913. Drug of choice for bacterial vaginosis is:

A. Metronidazole

B. Ampicillin

C. Ciprofloxacin

D. Fluconazole

Key: A

914. Mebendazole cannot be used for)

A. Ascariasis

B. Entrobius vermicularis

C. Onchocercosis

D. Hydatid cyst disease

Key: C

915. Drug of choice for medical treatment of hydatid cyst

of liver is:

A. Praziquantel

B. Thiabendazole

C. Ivermectin

D. Albendazole

Key: D

916. Drug commonly used in the treatment of

echinococosis is:

A. Albendazole

B. Ivermectin

C. Pyrental prermeated

D. Metronidazole

Key: A

917. Ivermectin is used for the treatment of:

A. Filariasis

B. Ascariasis

C. Teniasis

D. Hookworm infestation

Key: A

918. Drug amphotericin B is used for treatment of:

A. Sleeping sickness

B. Kala azar

C. Malaria

D. Filaria

Key: B

Page 77: 7. Sodium valproate

919. Drug that is not used in renal failure is:

A. Ethambutol

B. Rifampicin

C. Isoniazid

D. Streptomycin

Key: D

920. The antiretroviral drug which is also effective in

chronic active hepatitis-B infection is:

A. Zidovudine

B. Nelfinavir

C. Efavirenz

D. Lamivudine

Key: D

921. Drug of choice for hookworm infestation is:

A. Piperazine citrate

B. Bephenium hydroxynaphthoate

C. Mebendazole

D. Albendazole

Key: D

922. Drug of choice for ascariasis is:

A. Piperazine citrate

B. Bephenium hydroxynaphthoate

C. Mebendazole

D. Albendazole

Key: D

923. Drugs of choice for the treatment of

neurocysticercosis are:

A. Hydroquinone and metronidazole

B. Metronidazole and pyrental palmoate

C. Albendazole and praziquantal

D. Cyclophosphamide

Key: C

924. Oral contraceptive (OCP) failure by rifampicin is due

to:

A. Decreased absorption of OCPs

B. Increased binding of OCPs by rifampicin and

reduced free drug concentration

C. Increased metabolism of OCPs

D. Increased chances of ovulation due to rifampicin

Key: C

925. Drug causing icthyosis and hyperpigmentation, when

used in leprosy is:

A. Rifampicin

B. Dapsone

C. Clofazimine

D. Ethionamide

Key: C

926. Along with INH, which vitamin is given ?

A. Riboflavin

B. Pyridoxine

C. Niacin

D. Cyanocobalamin

Key: B

927. Drug of choice for acyclovir resistant herpes is:

A. Cidofovir

B. Gancyclovir

C. Valacyclovir

D. Foscarnet

Key: D

928. Drug of choice for exo-erythrocystic stage of malaria

is:

A. Chloroquine

B. Primaquine

C. Proguanil

D. Mefloquine

Key: B

929. Side effects of isoniazid are all except:

A. Hepatitis

B. Optic neuritis

C. Peripheral neuropathy

D. Thrombocytopenia

Key: D

930. Rifampicin acts by inhibiting:

A. DNA Dependent RNA polymerase

B. RNA dependent DNA polymerase

C. Mycolic acid inhibition

D. Mycolic acid incorporation defects

Key: A

931. Antifungal drug used for systemic fungal infection is:

A. Griseofulvin

B. Clotrimazole

C. Amphotericin B

D. Econazole

Key: C

932. Drug that can cause hyperuricemia is:

Page 78: 7. Sodium valproate

A. Pyrazinamide

B. INH

C. Rifampicin

D. None of the above

Key: A

933. Optic neuritis is caused by:

A. Ethambutol

B. INH

C. Rifampicin

D. Chlormycetin

Key: A

934. Mechanism of action of paclitaxel is:

A. Topoisomerase inhibition

B. Increases the polymerization of tubulin

C. Inhibits protein synthesis

D. Alkylation of DNA

Key: B

935. Bleomycin toxicity affects which type of cells:

A. Type-I pneumocytes

B. Type-II pneumocytes

C. Endothelial cells

D. Pulmonary alveolar macrophages

Key: A

936. Sustained neutropenia is seen with?

A. Vinblastine

B. Cisplatin

C. Carmustine

D. Cyclophosphamide

Key: C

937. Cyclophosphamide/Ifosfamide belongs to which

group of anticancer drugs?

A. Alkylating agents

B. Antimetabolites

C. Mitotic inhibitors

D. Topoisomerase inhibitor

Key: A

938. Anticancer drug causing SIADH as an adverse effect

is:

A. Vincristine

B. Paclitaxel

C. Dacarbazine

D. Cyclophosphamide

Key: A

939. Which of the following drugs is topoisomerase 1

inhibitor?

A. Doxorubicin

B. Irinotecan

C. Etoposide

D. Vincristine

Key: B

940. Which of the following drugs is associated with

untoward side effect of renal tubular damage?

A. Cisplatin

B. Streptozocin

C. Methysergide

D. Cyclophosphamide

Key: A

941. Which of the following chemotherapeutic agents is

associated with secondary leukemia?

A. Vinblastine

B. Paclitaxel

C. Cisplatin

D. Bleomycin

Key: C

942. Sterile hemorrhagic cystitis is caused by:

A. Busulfan

B. Ketoprofen

C. Methicillin

D. Cyclophosphamide

Key: D

943. The mechanism of anticancer action of fluorouracil is:

A. Cross linking of double stranded DNA and the

resulting inhibition of DNA replication and

transcription

B. Cytotoxicity resulting from a metabolite that

interferes with the production of dTMP

C. Irreversible inhibition of dihydrofolic acid

reductase

D. Selective action on DNA polymerase

Key: B

944. Mechanism of action of vincristine in the treatment of

ALL is:

A. Inhibition of topoisomerase II to cause breaks in

DNA strands

B. Alkylation and cross linking DNA strands

C. Inhibition of DNA mediated RNA synthesis

D. Inhibition of polymerization of tubulin to form

microtubules

Page 79: 7. Sodium valproate

Key: D

945. Hand and foot syndrome is an adverse effect of:

A. 5-Fluorouracil

B. Bleomycin

C. Etoposide

D. Actinomycin D

Key: A

946. Most common side effect of 5-fluorouracil is:

A. G.I. toxicity

B. Bone marrow depression

C. Cardiotoxicity

D. Neurotoxicity

Key: A

947. Which of the following is a common side effect of

cisplatin?

A. Diarrhea

B. Vomiting

C. Pulmonary fibrosis

D. Alopecia

Key: B

948. Which of the following anticancer drugs can cross

blood brain barrier?

A. Cisplatin

B. Nitrosourea

C. Vincristine

D. Vinblastine

Key: B

949. Which of the following drugs produce significant

nephrotoxicity?

A. Cisplatin

B. Carboplatin

C. Vinblastine

D. Vincristine

Key: A

950. Folinic acid counteracts the toxicity of:

A. Doxorubicin

B. Methotrexate

C. Cyclophosphamide

D. Fluorouracil

Key: B

951. The antimalignancy drug which is potentially

cardiotoxic is:

A. Doxorubicin

B. Bleomycin

C. Fluorouracil

D. Dacarbazine

Key: A

952. “Stocking and glove” neuropathy is seen in:

A. Vinblastine

B. Paclitaxel

C. Etoposide

D. Mitoxantrone

Key: B

953. Which of the following causes peripheral neuritis?

A. Methotrexete

B. Vincristine

C. Busulfan

D. Cyclophosphamide

Key: B

954. Mechanism of action of imatinib mesylate is:

A. Increase in metabolism of P glycoprotein

B. Blocking the action of P glycoprotein

C. Blocks the action of chimeric fusion protein of

bcrabl

D. Non-competitive inhibition of ATP binding site

Key: C

955. The drug Imatinib acts by the inhibition of:

A. Tyrosine kinase

B. Glutathione reductase

C. Thymidylate synthetase

D. Protein kinase

Key: A

956. Treatment of choice for chronic myeloid leukemia is:

A. Imatinib

B. Hydroxyl-urea

C. Interferon-alpha

D. Cytarabine

Key: A

957. Allopurinol potentiates action of:

A. Azathioprine

B. Busulfan

C. Actinomycin

D. Procarbazine

Key: A

Page 80: 7. Sodium valproate

958. Finasteride is a:

A. 5-a reductase inhibitor

B. Phosphodiesterase inhibitor

C. Alpha blocker

D. Androgen receptor blocker

Key: A

959. The drug of choice in choriocarcinoma is:

A. Methotrexate

B. Actinomycin-D

C. Vincristine

D. 6-Thioguanine

Key: A

960. Drug of choice for neutropenia due to cancer

chemotherapy is:

A. Vitamin B-12

B. IL-11

C. Filgrastim

D. Erythropoietin

Key: C

961. Pulmonary fibrosis is caused by

A. Methotrexate

B. Cyclophosphamide

C. Mercaptopurine

D. Busulfan

Key: D

962. Thalidomide acts through:

A. Inhibiting angiogenesis

B. Inhibiting thymidylate synthase

C. Inhibition of Topo-isomerase I

D. Inhibition of Topo-isomerase II

Key: A

963. Which of the following is not an adverse effect of

growth hormone therapy?

A. Carpal tunnel syndrome

B. Hypoglycemia

C. Intracranial hypertension

D. Slipped femoral epiphysis

Key: B

964. Which of the following is used in the treatment of

hyperprolactinemia?

A. Cimetidine

B. Methysergide

C. Bromocriptine

D. Ondansetron

Key: C

965. A 22 year old female, presented to you with

complaints of headache and vomiting since 2 months.

She is having amenorrhea but urine pregnancy test is

negative. She also complained of secretion of milk

from the breasts. A provisional diagnosis of hyper

prolactinemia was made and MRI was suggested.

MRI confirmed the presence of a large pituitary

adenoma. Patient was advised surgery, however, she

is not willing to undergo surgery. Which of the

following medications is most likely to be prescribed?

A. Sumatriptan

B. Bromocriptine

C. Ergotamine

D. Allopurinol

Key: B

966. All of the following agents are useful in acromegaly

EXCEPT:

A. Bromocriptine

B. Somatostatin

C. Octreotide

D. Nafarelin

Key: D

967. Mechanism of action of bromocriptine is:

A. Agonism at D2 receptors

B. Antagonism at D2 receptors

C. Antagonism at D1 receptors

D. Antagonism at α receptors

Key: A

968. Which of the following drugs DO NOT cause

hyperprolactinemia?

A. Bromocriptine

B. Haloperidol

C. Reserpine

D. Chlorpromazine

Key: A

969. A young female with amenorrhea, infertility and

galactorrhoea was treated with a drug that

successfully restored ovulation and menstruation.

Before being given the drug, the woman was carefully

questioned about previous mental health problems,

which she did not have. She was advised to take the

drug orally. The drug used to treat this patient was

probably:

A. Bromocriptine

B. Desmopressin

Page 81: 7. Sodium valproate

C. Human gonadotropin hormone

D. Leuprolide

Key: A

970. A 47-year old male exhibited signs and symptoms of

acromegaly. Radiologic studies showed the presence

of a large pituitary tumor. Surgical treatment of the

tumor was only partially effective in controlling the

disease. At this point, which of the following drugs is

most likely to be used as pharmacological therapy?

A. Desmopressin

B. Leuprolide

C. Octreotide

D. Somatropin

Key: C

971. A 7-year old boy underwent successful chemotherapy

and cranial radiation for the treatment of acute

lymphocytic leukemia. One month after the

completion of therapy, the patient presented with

excessive thirst and urination plus hypernatremia.

Laboratory testing revealed pituitary diabetes

insipidus. To correct these problems, this patient is

likely to be treated with:

A. Corticotropin

B. Desmopressin

C. hCG

D. Menotropins

Key: B

972. Long acting dopamine agonist is:

A. Bromocriptine

B. Lisuride

C. Cabergoline

D. Apomorphine

Key: C

973. Bromocriptine is useful in all except:

A. Parkinsonism

B. Prolactinoma

C. Endogenous depression

D. Infertility

Key: C

974. Drugs used for treatment of acute variceal bleeding

are all except:

A. Octreotide

B. Somatostatin

C. Desmopressin

D. Terlipressin

Key: C

975. GnRH analogue used in hormonal treatment of

carcinoma prostate is?

A. Goserelin

B. Nilutamide

C. Cyproterone acetate

D. Finasteride

Key: A

976. Which of the following is given at intervals as a

pulsatile therapy?

A. GnRH

B. GH

C. FSH

D. Estrogen

Key: A

977. True regarding use of bromocriptine for suppression

of lactation includes:

A. It can cause deep vein thrombosis

B. It can cause hypotension

C. Metoclopramide potentiates the action of

bromocriptine

D. It is given for 1 week only

Key: B

978. Which of the following statements about iodine is

false?

A. Contraindicated in hyperthyroidism

B. Causes iodism

C. Inhibits the release of thyroxine

D. Inhibits the synthesis of iodo thyroxine and iodo

thyronine

Key: A

979. Conversion of T4 to T3 is inhibited by all except:

A. Propanolol

B. Propylthiouracil

C. Amiodarone

D. Methimazole

Key: D

980. A pregnant female is taking carbimazole. Which of

the following is not seen in the neonate ?

A. Choanal atresia

B. Scalp defects

C. Cleft lip/palate

D. Fetal goiter

Key: C

Page 82: 7. Sodium valproate

981. The drug of choice for the treatment of thyrotoxicosis

during pregnancy is:

A. Carbimazole

B. Iodine therapy

C. Propylthiouracil

D. Methimazole

Key: C

982. The antithyroid drug with the most rapid onset of

antithyroid action is:

A. Iiodine131

B. Sodium iodide

C. Methimazole

D. Propylthiouracil

Key: B

983. Triiodothyronine (T3) as compared to T4:

A. Is more plasma protein bound

B. Is shorter acting

C. Is less potent

D. Has delayed action

Key: B

984. Mechanism of action of propylthiouracil in

hyperthyroidism is:

A. Inhibition of organification of iodine

B. Inhibition of oxidation of iodine

C. Inhibition of coupling of two DIT residues

D. All of the above

Key: D

985. Which of the following drugs inhibit 5’-deiodinase?

A. Propylthiouracil

B. Methimazole

C. Lugol’s iodine

D. Radioactive iodine

Key: A

986. Carbimazole as compared to propylthiouracil:

A. Is less potent

B. Is shorter acting

C. Does not produce an active metabolite

D. Does not inhibit peripheral conversion of T4 to T3

Key: D

987. Fastest acting anti-thyroid drug is:

A. Lugol’s iodine

B. Radioactive iodine

C. Propylthiouracil

D. Sodium thiocyanate

Key: A

988. Beta blockers are used in hyperthyroidism:

A. As short term symptomatic therapy till effect of

B. carbimazole develops

C. As long term therapy after subtotal

thyroidectomy

D. In patient not responding to carbimazole

E. To potentiate the effect of radioactive iodine

Key: A

989. A pregnant female with thyrotoxicosis is planned for

surgery. Before surgery can be done, her gland should

be reduced in size and vascularity by administering:

A. Iodide ion

B. Propanolol

C. Propylthiouracil

D. Radioactive iodine

Key: C

990. A patient of hypothyroidism was prescribed

levothyroxine. Which of the following is the most

reliable guide for adjustment of thyroxine dose in

him?

A. Pulse rate

B. Body weight

C. Serum thyroxine level

D. Serum TSH level

Key: D

991. Which of the following is not used in the management

of thyroid storm?

A. Potassium iodide

B. Reserpine

C. Propanolol

D. Calcium channel blockers

Key: B

992. Plasma half-life of carbimazole is:

A. 4 hours

B. 8 hours

C. 16 hours

D. 24 hours

Key: B

993. All are antithyroid drugs except:

A. Propylthiouracil

B. Methimazole

C. Carbimazole

Page 83: 7. Sodium valproate

D. Carbamazepine

Key: D

994. L-Thyroxine is used in:

A. Thyroid storm

B. Cretinism

C. Endemic goiter

D. Grave’s disease

Key: B

995. Safest treatment of hyperthyroidism in pregnant

women is:

A. Radioactive iodine

B. Methimazole

C. Carbimazole

D. Propylthiouracil

Key: D

996. All are causing hypothyroidism except:

A. PAS

B. Captopril

C. Lithium

D. Amiodarone

Key: B

997. Conversion of T 4 to T3 inhibition is associated with:

A. Propylthiouracil

B. Ampicillin

C. Lithium

D. Carbimazole

Key: A

998. Which of the following anti-diabetic drugs can cause

vitamin B 12 deficiency?

A. Glipizide

B. Acarbose

C. Metformin

D. Pioglitazone

Key: C

999. Which of the following drugs does not cause

hypoglycemia:

A. Acarbose

B. Insulin

C. Glimepride

D. Nateglinide

Key: A

1000. A patient is receiving insulin and acarbose for

diabetes mellitus and developed hypoglycemia.

Which of the following should be used for treatment

of hypoglycemia in this patient?

A. Sucrose

B. Galactose

C. Glucose

D. Starch

Key: C

1001. True about pioglitazone are all except:

A. Metabolized in the liver by CYP3A4

B. Selective agonist for the nuclear peroxisome

profilerator activated receptor gamma

C. It causes transcription of gene for carbohydrate

and fat metabolism in the absence of insulin

D. It should be avoided in a patient with cardio-

vascular disease

Key: C

1002. Insulin causes all of the following except:

A. Glycogenesis

B. Glycolysis

C. Lipogenesis

D. Ketogenesis

Key: D

1003. What will happen if insulin alone is given rapidly in

Diabetic Ketoacidosis?

A. Hypokalemia

B. Hypernatremia

C. Hyperkalemia

D. Hypocalcemia

Key: A

1004. All of the following preparations of insulin are rapid

and short acting EXCEPT:

A. Lispro

B. Aspart

C. Glargine

D. NPH

Key: C

1005. Which of the following statements about biguanides is

NOT true?

A. Do not stimulate insulin release

B. Decrease hepatic glucose production

C. Renal dysfunction is not a contraindication for

their use

D. Can be combined with sulfonylureas

Page 84: 7. Sodium valproate

Key: C

1006. If a diabetic patient being treated with an oral

hypoglycemic agent develops dilutional

hyponatremia, which one of the following could be

responsible for this effect?

A. Chlorpropamide

B. Tolbutamide

C. Glyburide

D. Glimepride

Key: A

1007. All of the following are true about metformin

EXCEPT:

A. Causes little or no hypoglycemia in non-diabetic

patients

B. Acts by increased insulin secretion

C. Increases peripheral utilization of glucose and

decreases absorption of glucose from intestine

D. When given with alcohol, increases risk of lactic

acidosis

Key: B

1008. True about lispro-insulin is:

A. Action is faster and short in duration than regular

insulin

B. It is given 15 minutes prior to meal

C. Source is lamb

D. Action is faster and of longer duration than

regular insulin

Key: A

1009. Antidiabetic drugs that can be used safely in renal

failure are:

A. Metformin

B. Glimepiride

C. Phenformin

D. Rosiglitazone

Key: B

1010. The most likely complication of insulin therapy in

ketoacidosis is:

A. Dilutional hyponatremia

B. Hypoglycemia

C. Increased bleeding tendency

D. Pancreatitis

Key: B

1011. Which of the following drugs promotes the release of

endogenous insulin?

A. Acarbose

B. Glipizide

C. Metformin

D. Pioglitazone

Key: B

1012. Which of the following drugs is taken during the first

part of the meal for the purpose of delaying

absorption of dietary carbohydrates?

A. Acarbose

B. Glipizide

C. Nateglinide

D. Pioglitazone

Key: A

1013. Which of the following patients is most likely to be

treated with intravenous glucagon?

A. A young man who took cocaine and has a blood

pressure of 190/110 mm Hg

B. A middle aged man with type II diabetes who

has not taken his regular dose of glipizide for

last 4 days

C. An old man with severe bradycardia and

hypotension resulting from ingestion of

overdose of atenolol

D. An old woman with lactic acidosis as a

complication of severe infection and shock

Key: C

1014. Insulin acts by stimulation of:

A. Ionotropic receptor

B. Enzymatic receptor

C. Metabotropic receptor

D. Nuclear receptor

Key: B

1015. The most common route of administration of insulin

is:

A. Intradermal

B. Subcutaneous

C. Intramuscular

D. Intravenous

Key: B

1016. A 15 year old girl with type 1 diabetes is brought to

emergency complaining of dizziness. Laboratory

findings include severe hyperglycemia, ketoacidosis

and blood pH of 7.15. To achieve rapid control of

severe ketoacidosis, appropriate drug is:

A. Crystalline insulin (regular)

B. NPH insulin

C. Tolbutamide

Page 85: 7. Sodium valproate

D. Ultra lente insulin

Key: A

1017. A 54- year old obese patient with type 2 diabetes

mellitusnband a history of alcoholism probably

should not receive metformin because it can increase

the risk of:

A. Disulfiram like reaction

B. Hypoglycemia

C. Lactic acidosis

D. Severe hepatic toxicity

Key: C

1018. Insulin causes:

A. Na+ entry into cells

B. K+ exit from cells

C. Na+ exit/K+ entry

D. K+ entry into cells

Key: D

1019. Indications of newer insulins include all EXCEPT:

A. Insulin resistance

B. Lipodystrophy

C. Pregnancy

D. Diabetic kidney disease

Key: D

1020. Which of the following is not used for the treatment

of insulin induced hypoglycemia?

A. Intravenous glucose

B. Glucagon

C. Adrenaline

D. Oral carbohydrates

Key: C

1021. Sulfonylureas act by:

A. Decreasing glucagon secretion from pancreas

B. Decreasing insulin secretion from pancreas

C. Increasing gluconeogenesis

D. Increasing insulin secretion from pancreas

Key: D

1022. Flushing is common in patient taking which of the

following oral hypoglycemic drug with alcohol:

A. Chlorprompamide

B. Phenformin

C. Glibenclamide

D. Tolazamide

Key: A

1023. Anti-diabetic effect of sulfonylureas is by reducing:

A. Glucagon production

B. Insulin secretion

C. Tissue sensitivity to insulin

D. Tissue sensitivity to glycogen

Key: A

1024. Lactic acidosis is commonly seen in:

A. Metformin

B. Repaglinide

C. Rosiglitazone

D. All

Key: A

1025. Tolbutamide acts by increasing:

A. Insulin receptors

B. Glucose entry

C. Glucose absorption

D. Insulin secretion

Key: D

1026. Adverse effects of insulin include all of the following

except:

A. Edema

B. Hyperglycaemia

C. Lipodystrophy

D. Allergy

Key: B

1027. Long acting insulin is:

A. Lente

B. Semilente

C. Ultralente

D. Lispro insulin

Key: C

1028. Monocomponent insulin has all the following

advantages except:

A. Can be used in pregnancy

B. Less hypoglycemic episodes

C. Longer duration of action

D. Less chances of lipodystrophy

Key: C

1029. Oral hypoglycemic drug that is less likely to cause

hypoglycemia is:

A. Repaglinide

B. Gliclazide

C. Rosiglitazone

Page 86: 7. Sodium valproate

D. Glimipiride

Key: C

1030. Common side effect of thiazolidinediones is:

A. Dysguesia

B. Hypoglycemia

C. Water retention with weight gain

D. Anemia

Key: C

1031. Long acting insulin preparations are frequently

administered by:

A. Oral route

B. Intramuscular route

C. Intradermal route

D. Subcutaneous route

Key: D

1032. Drug of choice for pregnant female suspected of

having a baby with congenital adrenal hyperplasia is:

A. Dexamethasone

B. Betamethasone

C. Hydrocortisone

D. Predinsolone

Key: A

1033. Hyperaldosteronism causes all except:

A. Hypernatremia

B. Hypokalemia

C. Hypertension

D. Metabolic acidosis

Key: D

1034. The most potent topical corticosteroid is:

A. Hydrocortisone butyrate cream 0.1%

B. Betamethasone valerate cream 0.5%

C. Clobetasol propionate cream 0.5%

D. Clobetasone butyrate cream 0.5%

Key: B

1035. All of the following are correct about steroids

EXCEPT:

A. Inhibit the release of arachidonic acid from

vessel wall through action on phospholipase A

B. Binds to the plasma membrane receptors and

following internalization influence nuclear

changes

C. Inhibit vascular membrane permeability

D. Increase glucose synthesis in liver

Key: B

1036. Glucocorticoids act in inflammation by:

A. ↓ Lipocortin

B. ↑ IL-2

C. ↑ Lipocortin

D. ↑ CRP

Key: C

1037. Drugs causing Addison’s disease are:

A. Ketoconazole & Aminoglutethimide

B. Cyclosporine & Glucocorticoids

C. ACTH

D. None

Key: A

1038. Glucocorticoids have proved useful in the treatment

of:

A. Chemotherapy induced vomiting

B. Hyperprolactinemia

C. Parkinson’s disease

D. Type II diabetes

Key: A

1039. The drug prednisolone is known to be a powerful

antiinflammatory agent. This is true due to the action

of the drug on which of the following enzymes?

A. Cyclooxygenase

B. Lipoxygenase

C. Phospholipase A

D. Phosphodiesterase

Key: C

1040. Hydrocortisone acts as an anti-inflammatory agent

because of induction of the synthesis of which of the

following protein?

A. Heat shock protein 90

B. Inhibin

C. Transcortin

D. Lipocortin

Key: D

1041. Which of the following steroids is preferred for high

dose intravenous corticosteroid pulse therapy?

A. Cortisone

B. Hydrocortisone

C. Triamcinolone

D. Methyl-prednisolone

Key: D

Page 87: 7. Sodium valproate

1042. Select the regime of corticosteroids which has the

maximum adverse effect potential?

A. Prednisolone 20 mg/day oral for one year

B. Prednisolone 60 mg/day oral for 7 days

C. Dexamethasone 4 mg intravenous daily for 3

days

D. Methyl-prednisolone 1000 mg intravenous twice

single dose

Key: A

1043. Which of the following disorders is NOT aggravated

by corticosteroid therapy?

A. Congenital adrenal hyperplasia

B. Diabetes mellitus

C. Hypertension

D. Peptic ulcer

Key: A

1044. Toxic effects of long term administration of a

glucocortocoid include:

A. Hepatotoxicity

B. Osteoporosis

C. Precocious puberty

D. Lupus like syndrome

Key: B

1045. A patient has been diagnosed to have bronchial

asthma and is maintained on oral prednisolone 20 mg

daily and inhaled salbutamol as required. The patient

develops chest infection. Which of the following

measures would you like to take?

A. Stop prednisolone

B. Reduce prednisolone dose to 5 mg/day

C. No change/increase in prednisolone dose

D. Substitute prednisolone with inhaled budesonide

Key: C

1046. Salma has been diagnosed to have brain tumor. You

would prefer to give her

betamethasone/dexamethasone over hydrocortisone as

steroids to decrease her cerebral edema because:

A. They do not cause Na+ and water retention

B. They are more potent

C. They can be administered intravenously

D. They inhibit brain tumours

Key: A

1047. A 50 year old man with advanced tuberculosis has

developed signs of severe acute adrenal insufficiency.

The patient should be treated immediately with a

combination of:

A. Aldosterone and fludrocortisone

B. Triamcinolone and dexamethasone

C. Cortisol and fludrocortisone

D. Dexamethasone and metyrapone

Key: C

1048. Long term steroid therapy can lead to suppression of

hypothalamic-pituitary-adrenal axis. It can be

overcome by using alternate day therapy with

corticosteroids. Which of the following steroids are

unsuitable for alternate day therapy for such purpose?

A. Cortisol

B. Prednisolone

C. Betamethasone

D. Hydrocortisone

Key: C

1049. Which is not true about beclomethasone?

A. Indicated for chronic use

B. Inhalational steroid

C. Effective in acute asthma

D. Predispose to fungal infections

Key: C

1050. Most potent mineralocorticoid is:

A. Aldosterone

B. Fludrocortisone

C. Triamcinolone

D. None

Key: A

1051. All are side effects of steroids EXCEPT:

A. Skin atrophy

B. Telengectasia

C. Folliculitis

D. Photosensitivity

Key: D

1052. Systemic steroids can cause all of the following

EXCEPT:

A. Hypertension

B. Glaucoma

C. Cataract

D. Osteoporosis

Key: B

1053. Compared to hydrocortisone maximum glucocorticoid

action is found in:

A. Dexamethasone

B. Prednisolone

Page 88: 7. Sodium valproate

C. Methyl prednisolone

D. Cortisone

Key: A

1054. Which of the following antifungal drug can be used in

the treatment of Cushing syndrome?

A. Ketoconazole

B. Fluconazole

C. Itraconazole

D. Miconazole

Key: A

1055. Steroids cause:

A. Increased TSH

B. Increased FSH

C. Prevent de-iodination

D. All of the above

Key: C

1056. Bisphosphonates act by:

A. Increasing the osteoid formation

B. Increasing the mineralization of osteoid

C. Decreasing the osteoclast mediated resorption of

bone

D. Decreasing the parathyroid hormone secretion

Key: C

1057. Bisphosphonates are used in all EXPECT:

A. Paget's disease

B. Vitamin D excess

C. Postmenopausal osteoporosis

D. Hypercalcemia of malignancy

Key: B

1058. A child has been diagnosed to be having vitamin D

dependent rickets. The most appropriate vitamin D

preparation for him is:

A. Calciferol

B. Cholecalciferol

C. Calcifediol

D. Calcitriol

Key: D

1059. A patient began taking alendronate and she was

advised to take large amount of water and remain in

the standing position for at least half an hour till she

had the first meal of the day. These instructions were

given to reduce the risk of:

A. Cholelithiasis

B. Constipation

C. Erosive esophagitis

D. Osteonecrosis

Key: C

1060. Calcitonin causes hypocalcemia by:

A. Inhibiting bone resorption

B. Promoting osteolysis

C. Decreasing renal tubular reabsorption of

calcium

D. Decreasing absorption of phosphorus

Key: A

1061. Prevention or treatment of osteoporosis in

postmenopausal women may be achieved by all

EXCEPT:

A. Estrogen and progesterne hormone replacement

therapy

B. Calcium and vitamin D supplementation

C. Bisphosphonates

D. Multivitamins

Key: D

1062. Contraceptive that should be avoided in epilepsy is:

A. Oral contraceptives

B. Condoms

C. Intrauterine contraceptive devices

D. Post-coital pills

Key: A

1063. Which of the following progesterone is used in

emergency contraception?

A. Levonorgesterol

B. Micronised Progesterone

C. Norgesterone

D. Depot Medroxyprogesterone acetate

Key: A

1064. In a patient taking oral contraceptive, the chance of

pregnancy increases after taking any of the following

drugs EXCEPT

A. Phenytoin

B. Griseofulvin

C. Ampicillin

D. Cimetidine

Key: D

1065. Oral contraceptives are not given with:

A. Streptomycin

B. Rifampicin

C. Pyrazinamide

Page 89: 7. Sodium valproate

D. Ethambutol

Key: B

1066. Which of the following is an aromatase inhibitor?

A. Tamoxifen

B. Letrozole

C. Danazol

D. Taxane

Key: B

1067. Diethylstilbesterol should never be used in pregnant

women because it is associated with:

A. Development of deep vein thrombosis in the

pregnant woman

B. Feminization of the external genitalia of male

offspring

C. Infertility and development of vaginal cancer in

female offspring

D. Virilization of the external genitalia of female

offspring

Key: C

1068. Dr. Anam decides to give estrogen therapy in a

postmenopausal woman. The risk of which of the

following will not be increased?

A. Gall stones

B. Osteoporosis

C. Endometrial carcinoma

D. Breast cancer

Key: B

1069. Clomiphene citrate is not known to produce which of

the following effects in a young female of 30 years of

age (child bearing age group)?

A. Hot flushes

B. Ovulation

C. Decreased FSH and LH secretion

D. Polycystic ovaries

Key: C

1070. The combined estrogen-progestin oral contraceptive

pill act mainly by:

A. Production of cervical mucus hostile to sperm

penetration

B. Suppression of FSH and LH release

C. Making endometrium unsuitable for

implantation

D. Enhancing uterine contraction to dislodge the

fertilized ovum

Key: B

1071. One of the health benefits of the use of combined oral

contraceptives in pre-menopausal women is that these

contraceptives reduce the risk of:

A. Deep vein thrombosis

B. Migraine

C. Ovarian cancer

D. Ischemic stroke

Key: C

1072. A 23-year old woman desires a combined oral

contraceptive for pregnancy protection. A factor that

would lead a health professional to recommend an

alternative form of contraception is that the woman:

A. Has an evidence of hirsutism

B. Has a history of gastroesophageal reflux disease

and is currently taking omeprazole

C. Has a history of pelvic inflammatory disease

D. Has a history of migraine headache that is well

controlled by sumatriptan

Key: D

1073. A young female comes to you in the gynaecology

OPD and gives the history that she had intercourse

with her boyfriend 5 hours back. Select the drug that

can act as a single dose postcoital contraceptive for

her:

A. Clomiphene citrate

B. Mifepristone

C. Danazol

D. Medroxyprogesterone acetate

Key: B

1074. A patient Parul gives you the history that she has

missed a single dose of her combined oral

contraceptive pill. Which of the following will you

advise her?

A. Continue with the course without regard to the

missed dose

B. Take 2 pills the next day and continue with the

course

C. Take 2 pills everyday for the remaining part of

the course

D. Discontinue the course and use alternative

method of contraception

Key: B

1075. Oral contraceptive pills can cause all except:

A. Mastalgia

B. Dysmenorrhea

C. Chloasma

D. Breakthrough bleeding

Page 90: 7. Sodium valproate

Key: B

1076. Side effects of oral contraceptives are all EXCEPT:

A. Irregular bleeding

B. Headache

C. Thrombosis

D. Increased risk of ovarian cancer

Key: D

1077. All of the following are natural estrogens EXCEPT:

A. Estradiol

B. Ethinylestradiol

C. Estriol

D. Estrone

Key: B

1078. Mechanism of action of tamoxifen is:

A. Has androgenic receptor blocking action

B. Inhibits enzyme 5 α-reductase

C. Has partial agonist and antagonist action on

estrogen receptors

D. Inhibition of FSH and LH release from the

pituitary

Key: C

1079. The progestogenic emergency contraceptive pills act

by:

A. Altered cervical secretion

B. Inhibition of ovulation

C. Anti-implantation effect

D. Inhibition of LH secretion

Key: C

1080. The drug used for first trimester abortion is:

A. Oral mifepristone

B. Extra-amniotic ethacrydine lactate

C. Oxytocin infusion

D. Chloroquine

Key: A

1081. “Oral contraceptive pills” protect against:

A. Thrombosis

B. Ovarian cancer

C. Cancer cervix

D. Hepatocellular adenoma

Key: B

1082. Mifepristone is a:

A. Progesterone antagonist

B. Oestrogen antagonist

C. Intra-amniotic saline

D. None

Key: A

1083. Oral contraceptive failure occurs with:

A. Phenytoin

B. Phenobarbitone

C. Rifampicin

D. All

Key: D

1084. Clomiphene citrate is used for:

A. Mania

B. Induction of ovulation

C. Depression

D. Psychosis

Key: B

1085. All of the following are recognized effects of

combined oral contraceptive except:

A. Breakthrough bleeding

B. Decreased risk of endometrial cancer

C. Increased risk of ischemic stroke

D. Increased risk of ovarian cancer

Key: D

1086. In spider nevi, dilatation of blood vessels is due to:

A. Testosterone

B. Estrogen

C. Hepatotoxins

D. FSH

Key: B

1087. Oxytocin causes all except:

A. Lactogenesis

B. Milk ejection

C. Contraction of uterine muscle

D. Myoepithelial cell contraction

Key: A

1088. Drug of choice for polycystic ovarian disease is:

A. Metformin

B. Estrogen

C. Estrogen and progesterone combination pill

D. Dopamine antagonist

Key: A

1089. Hirsutism producing drugs include all except:

Page 91: 7. Sodium valproate

A. Methyldopa

B. Corticosteroids

C. Phenytoin

D. Minoxidil

Key: A

1090. Hypoglycemia is caused by:

A. Alcohol intoxication

B. Thiazide

C. Diazoxide

D. Metoclopramide

Key: A

1091. Mechanism of Calcitriol is:

A. Decreased calcium resorption calcium from

bone

B. Increase calcium absorption from intestine

C. Decreased calcium absorption from kidney

D. Decrease calcium absorption from intestine

Key: B

1092. Which of the following drug is a dopamine receptor

agonist ?

A. Methyl dopa

B. Bromocriptine

C. Haloperidol

D. Morphine

Key: B

1093. Which of the following is an anabolic steroid ?

A. Methyltestosterone

B. Fluoxymesterone

C. Nandrolone

D. Danazol

Key: C

1094. Tibolone is a :

A. SSRI

B. SPRM

C. STEAR

D. SERM

Key: C, "Tibolone: a selective tissue estrogenic activity

regulator (STEAR)

1095. Drug which inhibits conversion of T4 to T3 is:

A. Carbimazole

B. Methimazole

C. Propylthiouracil

D. Lugol's iodine

Key: C

1096. Which of the following agents has the least

glucocorticoid action?

A. Fludrocortisone

B. Cortisone

C. Dexamethasone

D. Betamethasone

Key: C

1097. Which is an intermediate acting insulin?

A. Insulin lispro

B. Regular insulin

C. NPH insulin

D. Insulin glargine

Key: C

1098. Side effects of oxytocin are all except:

A. Placental abruption

B. Fetal distress

C. Peripheral vascular disease

D. Water intoxication

Key: C

1099. Intranasal calcitonin is used for?

A. Paget’s disease

B. MEN Syndrome

C. Hypercalcemia

D. Postmenopausal osteoporosis

Key: D

1100. Which insulin is never mixed with other insulins?

A. Lente

B. Aspart

C. Lispro

D. Glargine

Key: D

1101. Drug of choice for acute adrenal insufficiency is?

A. Oral prednisone

B. IV hydrocortisone

C. IV betamethasone

D. IV dexamethasone

Key: B

1102. Insulin release due to closure of K+ channels is seen

with:

A. Nateglinde

B. Acarbose

C. Exenatide

Page 92: 7. Sodium valproate

D. Sitagliptin

Key: A

1103. Drug used to control postprandial hyperglycemia is:

A. Acarbose

B. Biguanides

C. Sulfonylurea

D. Repaglinide

Key: D

1104. Bisphosphonate-induced osteomalacia is commonly

seen with:

A. Alendronate

B. Pamidronate

C. Zolendronate

D. Etidronate

Key: D

1105. Long term administration of glucocorticoids can

cause all of the following except:

A. Proximal myopathy

B. Hyperkalemia

C. Hypertension

D. Cataract

Key: B

1106. Absolute contraindication of combined oral

contraceptive pill is:

A. Epilepsy

B. Obesity

C. Smoking 10 cigars/day

D. Active liver disease

Key: D

1107. The managment of thyrotoxic crisis includes all the

following except:

A. Propanolol

B. Hydrocortisone

C. Oral I131

D. Propylthiouracil

Key: C

1108. Which one of the following is an adverse effect

associated with combined oral contraceptives:

A. Cerebral stroke

B. Aggravation of asthma

C. Peripheral neuropathy

D. Nephrotic syndrome

Key: A

1109. Sulphonylureas act by:

A. Reducing the absorption of carbohydrate from the

gut

B. Stimulating the beta islet cells of pancreas to

release insulin

C. Increasing the uptake of glucose in peripheral

tissue

D. Reducing the hepatic gluconeogenesis

Key: B

1110. Which of the following drugs used to treat type II

diabetes mellitus causes weight loss:

A. Metformin

B. Glimepiride

C. Repaglinide

D. Gliclazide

Key: A

1111. Most important step in management of diabetic

ketoacidosis is administration of:

A. Insulin

B. Intravenous fluids (saline)

C. Soda-bicarbonate

D. Potassium

Key: B

1112. Deaths from lactic acidosis in diabetes mellitus is

associated with therapy with which one of the

following:

A. Metformin

B. Tolbutamide

C. Phenformin

D. Glipizide

Key: A, Phenformin > Metformin

1113. Lugol’s iodine is given to the patient:

A. Before surgery

B. After surgery

C. During surgery

D. Adjuvant therapy

Key: A

1114. Somatostatin secreted by which type of cells in

pancreas?

A. Gamma cells

B. Delta cells

C. Alpha cells

D. Beta cells

Key: B

Page 93: 7. Sodium valproate

1115. The following insulin can be given intravenously:

A. Protamine zinc insulin

B. Ultra lente insulin

C. Semi lente insulin

D. Regular insulin

Key: D

1116. Drug of choice for bleeding oesophageal varices is:

A. Ethanolamine oleate

B. Octreotide

C. Propanolol

D. Phytonadione

Key: B

1117. All of the following reduce T4 absorption except:

A. Metformin

B. Iron salts

C. Raloxifene

D. Colsevelam

Key: A

1118. Drug of choice in gestational diabetes is

A. Metformin

B. Glimepiride

C. Repaglinide

D. Regular insulin

Key: D

1119. Drug of choice in diabetes in pregnancy is

A. Metformin

B. Glimepiride

C. Regular insulin

D. Gliclazide

Key: B

1120. Drug of choice of type II diabetes mellitus in obese

patient:

A. Metformin

B. Glimepiride

C. Repaglinide

D. Gliclazide

Key: A

1121. Drug of choice in patient with insulin resistance

A. Metformin

B. Glimepiride

C. Repaglinide

D. Gliclazide

Key: A

1122. Insulin secretion increase by

A. Beta blocker

B. Secretin

C. Citalopram

D. Beta agonist

Key: A

1123. Insulin secretion inhibited by

A. Epinephrine

B. B blocker

C. Somatostatin

D. Beta agonist

Key: C > B

1124. Effect of insulin on adipose tissue:

A. Stimulate hormone sensitive lipase

B. Increase Lipoprotein lipase

C. Inhibits lipoprotein lipase

D. Activate hormone sensitive lipase

Key: B

1125. A diabetic patient is suffering from upper respiratory

tract infection. Which fluoroquinolones in this patient

is contraindicated?

A. Levofloxacin

B. Moxifloxacin

C. Gatifloxacin

D. Gemifloxacin

Key: C, All above are Respiratory fluoroquinolones,

Gatifloxacin causes dysglycemia. Dysglycemia is

a broad term that refers to an abnormality in

blood sugar stability. This can include

hypoglycemia (low blood sugar) or

hyperglycemia (high blood sugar). Thiazide also

causes dysglycemia